National Law Class Quizzes w/answers&topic

अब Quizwiz के साथ अपने होमवर्क और परीक्षाओं को एस करें!

Question 10 A physician who was photographically documenting the deterioration of a terminally ill patient came into his room to take more photos as the patient's wife was saying her final goodbye. Ignoring the wife's protests, the physician lifted the patients head and slid a sheet of blue cardboard underneath for a better background, and then took some photos as the patient expired. The wife is the patient's sole heir. The physician may be liable for what tort(s)? A Battery and intrusion on seclusion. B Battery only, because a privacy ton does not survive the death of the plaintiff. C Intrusion on seclusion only, because the physician did not cause a harmful or offensive contact with the patient. D Neither battery nor intrusion on seclusion.

A (Intrusion on Seclusion & Battery)

Question 1 A defendant is on trial for violating a statute forbidding possession of a concealed weapon within 100 yards of a government building. The prosecution presents evidence that the defendant was arrested on a street corner with a handgun in his pocket. The building housing the local city hall occupies the entire block on the north and east sides of the tm streets where the defendant was apprehended. Which of the following statements is most accurate regarding judicial notice of the location of the city hall? A The judge may take judicial notice of this fact without resort to a map, and should instruct the jury that it may, but need not, accept this fact as evidence of an element of the offense. B The judge may take judicial notice of this fact only upon reference to an official street map of the city. C The judge may not take judicial notice of this type of fact in a criminal case without a request by the prosecution. D If the judge properly takes judicial notice of this fact, a presumption is created tat shifts the burden of persuasion to the defendant to disprove this fact

A Judicial Notice- Intermediate

Question 2 While driving east on a two-lane highway, a pickup truck collided with a westbound motorcycle. The accident was observed by a cab driver. Because the cab driver had a passenger, she did not stay at the scene of the accident but made some notes shortly thereafter. She made her statement to the police the next day, in which she stated that just before the accident, the pickup swerved into the westbound lane, hitting the motorcycle. At the trial of the motorcyclist's personal injury action against the truck driver, the cab driver was having difficulty remembering some of the facts. The motorcyclist's attorney sought to let her review the notes she had made Should the court permit the cab driver to review her notes? A Yes, because it is a present recollection refreshed. B Yes, because it is a past recollection recorded. C No, because the cab driver is required to report all accidents to her employer, and the record is thus privileged. D No, because there is no showing that the cab driver had used her notes when she gave her statement to the police officer.

A Present Recollection Refreshed - Intermediate

Question 4 A property owner sued a developer in federal court, alleging breach of an oral contract. According to the property owner, three persons witnessed the conversation in which the contract was allegedly made. At the final pretrial conference, the parties and the judge agreed that it would be sufficient to call only two of the witnesses, who were subsequently named in the pretrial conference order. The next day, five days before the scheduled trial, the property owner's attorney decided he ought to call all three witnesses to enhance the credibility of his case. He immediately notified the developer of his change in plans. May the property owner call the additional witness? A Yes, if the court modifies the pretrial order. B Yes, because the trial has not yet begun. C No, because the final pretrial order controls the subsequent course of the trial. D No, if the jury has already been selected.

A Pretrial Order-Intermediate

Question 1 The defendant is charged with the burglary of a home. Evidence presented at the defendant's trial indicates that he talked another person into assisting him. The jury is instructed on burglary, solicitation, conspiracy, and attempt. If the defendant is found by the jury to be guilty of burglary, which of the following is true? A He also may be found guilty of conspiracy, but not of solicitation or attempt. B He also may be found guilty of conspiracy and solicitation, but not of attempt. C He also may be found guilty of conspiracy or solicitation but not both, and he may not be found guilty of attempt. D He also may be found guilty of solicitation or attempt but not both, and he may not be found guilty of conspiracy

A Solicitation, Attempt, Conspiracy and Crime - Intermediate

Question 4 The President of the United States and the king of a foreign nation entered into a treaty agreeing that citizens of the foreign nation who reside in the United States would not be taxed by the United States and that United States citizens who reside in the foreign nation would not be taxed by it. The treaty was ratified by the United States Senate and the royal council of the foreign nation. One year after the treaty became effective, the foreign nation began to tax United States citizens within its borders. The President immediately declared the tax treaty to be void and ordered the Internal Revenue Service to tax citizens of the foreign nation living in the United States. Is the President's action constitutional? A Yes, because the President has emergency powers to protect United States citizens. B Yes, under the foreign policy powers of the President. C No, because the treaty is the supreme law of the land, on par with federal legislation, and the President is not free to ignore it. D No, unless the President receives the advice and consent of the United States Senate

B Executive Powers & Treaty- Intermediate

Question 2 An impatient driver who was fed up with jaywalking pedestrians drove straight at one of them, leaning on the horn and intending to make her jump. She did not hear him or change her pace however, because her music player was turned to full volume. A bystander on the curb rushed out to pull her to safety. She tripped as she was being pulled to the curb, fracturing her kneecap. If the pedestrian sues the driver for assault, what will be the likely result? A The driver wins, because the pedestrian did not know at the time that she was in danger from the driver. B The driver wins, because he did not intend for the pedestrian to be injured by his conduct. C The pedestrian wins, because the driver intended to create in her an apprehension of immediate harmful contact. D The pedestrian wins, because the driver's conduct was a substantial factor in causing her injury.

A (Assault)

Question 2 A nervous man was persuaded by his girlfriend to go with her to a haunted house. He saw the signs in front of the haunted house warning that this attraction has live "monsters" who will be trying to scare people and is not for the faint of heart and he also noticed the same warning printed on the tickets. He paid for his ticket and reluctantly went into the darkened house with his girlfriend. In the first room, an actor dressed as a large monster came at them with a shriek, and the man dove through one of the plate glass windows to the outside, severely lacerating his arms and face in the process. If the man brings an action against the actor, will he recover? A No, because the man expressly assumed the risk of injury. B No, unless the jury determines that the actor was negligent in trying to scare the man. C Yes, because the actor intended to cause apprehension on the part of the man. D Yes, but the man's recovery will be reduced by a certain percentage if the trier of fact determines that he was also at fault.

A (Assumption of the Risk)

Question 9 The superintendent of a waste management company that operated a landfill noted that some children who lived in a nearby residential development had taken to sledding down the snowy-covered mounds of dirt that were piled on the site. A construction company needing dirt fill for a highway project had offered to remove the mounds of dirt at minimal cost but the superintendent had not yet arranged for their removal. In the meantime, he posted numerous signs around the landfill site that stated in bold letters, "NO TRESPASSING-NO SLEDDING." Despite the signs, which he saw and read, a 10-year-old neighborhood child sledded down one of the mounds of dirt and was propelled onto a busy highway adjacent to the landfill, where he was struck by a car and seriously injured. In a jurisdiction that retains traditional contributory negligence rules, is the waste management company liable for the child's injuries? A Yes, because the company could have had the piles of fill removed at minimal cost. B Yes, because the company created a public nuisance. C No, because the child was a trespasser. D No, because the child read and understood the warning signs and appreciated the danger

A (Attractive Nuisance, Child. Trespass)

Question 7 A plumber working for a company providing plumbing services to commercial and industrial establishments was required to be "on call" for emergency plumbing services 24 hours a day, and was required to drive his company van home each night so he would have all of his tools and equipment at hand for any calls. However, he was not permitted to use the company van for personal errands. On his way home one afternoon, he took a detour toward a supermarket a few blocks away to pick up some items for dinner. While entering the supermarket parking lot, he drove negligently and struck a pedestrian, seriously injuring him. The pedestrian filed suit against the plumber's company in a jurisdiction that maintains traditional common law rules regarding contribution and indemnity, and the jury awarded him $100,000 in damages, which the company paid. If the company sues the plumber to recoup its loss in the lawsuit, which party will prevail? A The company can recover 100% of the judgment as an indemnity, because the plumber was negligent, not the company. B The company will prevail, because the company had a rule against using company vehicles for personal errands. C The company will not prevail, because the company has already been found liable under principles of vicarious liability in the lawsuit by the pedestrian. D The company will not prevail, because the company required the plumber to be "on call" 24 hours a day.

A (Contribution and Indemnification/Respondeat Superior)

Question 6 A columnist for a major metropolitan newspaper had a very antagonistic relationship with the city's mayor. When a restaurant owned by the columnist's family was shut down by city health inspectors, the columnist responded with a column publicizing the shutdown and asserting that it was in retaliation for his prior columns in which he had criticized the mayor. In fact, the mayor had nothing to do with the action by the city health inspectors. While the columnist had no evidence of the mayor's involvement, he believed that there was a connection because "that's how the city works." Can the mayor recover against the columnist for defamation? A No, because the columnist did not act with actual malice. B No, because the columnist had a qualified privilege to explain why he believed his family's business was shut down. C Yes, because the columnist's hostility toward the mayor establishes malice so as to overcome any qualified privilege the columnist had. D Yes, because the columnist should have investigated the accuracy of his claims before publishing the column

A (Defamation Public Figure)

Question 1 A new homeowner had two dogs that frequently barked at birds and squirrels in the yard, especially during the day while the homeowner was at work. A neighbor who worked nights was aggravated by the barking, which disturbed his sleep, and decided to let the homeowner know how he felt. One evening, upon learning that the homeowner was entertaining her boss and several clients, the neighbor came to her front door with a boombox and stalled playing a recording of the dogs barking, putting it at full volume. When the homeowner came to the door, he began yelling at her and berating her in front of her guests for having no consideration for her neighbors, while continuing to play the recording. The homeowner was very upset, especially because her guests decided that they had better leave, and she ended up losing a bonus that her boss was going to give her at the end of the evening. If the homeowner asserts a claim based on intentional infliction of emotional distress against the neighbor, what will be the probable result? A The homeowner will prevail because the neighbors conduct was extreme and outrageous. B The homeowner will prevail because she suffered pecuniary harm from the neighbor's conduct. C The neighbor will prevail because the homeowner suffered no physical harm. D The neighbor will prevail if the barking from the homeowner's dogs is judged to constitute a nuisance

A (IIED)

Question 7 A pedestrian walking on the sidewalk was struck by a car backing out of a driveway. The driver did not see the pedestrian because her neighbor's bushes obscured her view of the sidewalk. The pedestrian was seriously injured and brought suit against the driver and the neighbor. The pedestrian also included the city in his lawsuit, alleging that the city failed to enforce its ordinance requiring homeowners to provide a clear view of sidewalks where they intersect with driveways. The trier of fact determined that the driver was 60% at fault, the neighbor was 30% at fault, and the city was 10% at fault. The jurisdiction has adopted comparative contribution in cases applying joint and several liability. Which of the following is a correct statement regarding liability? A The city is liable to the pedestrian for the full amount of the damage award. B Both the driver and the neighbor are liable to the pedestrian for 90% of the damage award. C Each of the three defendants are liable to the pedestrian for one-third of the damage award. D The driver is liable to the pedestrian for 60% of the damage award, the neighbor is liable for 30% of the damage award, and the city is liable for 10% of the damage award.

A (Intermediate Indivisible Injury)

8. An interior decorator asked a woodworker she met at a crafts fair to build a curly maple armoire. They entered into a written contract, with a contract price of $6,500 to be paid upon the decorator's receipt of the armoire. When the work was completed, the woodworker shipped the armoire to the decorator. After inspecting it, the decorator felt that it was not of the same high level of workmanship as she was expecting, given the other furniture that the woodworker had showcased at the fair, and a good faith dispute arose between the parties as to the workmanship. The decorator sent the a check for $4,000 marked "payment in full." The woodworker indorsed and cashed the check, then sued the decorator to recover the $2,500 balance. What would most courts likely hold? (A) The woodworker's cashing of the check constituted an accord and satisfaction, discharging the decorator's duty to pay the balance. (B) The woodworker can recover the $2,500 balance from the decorator. (C) The woodworker is estopped to sue for the balance because he cashed the check knowing that it was being tendered in full settlement. (D) The woodworker's indorsing a check so marked constituted a written release, thereby discharging the contract.

A - Accord & Satisfaction

2. A buyer for a chain of shoe stores ordered 1,000 pairs of shoes from a shoe manufacturer. The shoes cost $50 per pair, so the total contract price was $50,000. It happened that the manufacturer owed $50,000 to a trucking company. The manufacturer assigned, in writing, "all proceeds from the contract with the buyer" to the trucking company. The manufacturer notified the buyer that he had assigned the proceeds of the contract to the trucking company and then shipped the 1,000 pairs of shoes to the buyer. Upon receipt of the shoes, the buyer discovered that 10% of the shoes were defective. He sent a check for of the contract price ($45,000) to the manufacturer, who deposited the check. Shortly thereafter, the manufacturer closed down its business and disappeared without a trace. The trucking company, meanwhile, demanded payment from the buyer, to no avail. If the trucking company sues the buyer for the $45,000 that the buyer paid on the contract, will the trucking company prevail? (A) Yes, because the buyer had notice from the manufacturer that the contract had been assigned to the trucking company. (B) No, because the manufacturer wrongfully took the money that was assigned to the trucking company and is solely liable to the trucking company. (C) No, because the buyer fulfilled his obligations under the contract by paying the manufacturer. (D) No, because the trucking company could not have performed the other side of the contract by furnishing the shoes.

A - Assignment

6. The owner of a house put the property up for sale. A surgeon entered into negotiations with the owner to purchase the house, and the parties agreed upon a sale price of $200,000 and a closing date. The owner told the surgeon that she would drop a contract in the mail and have her attorney draw up a deed. The owner signed a land sale contract, which included the property's address but did not contain a metes and bounds legal description. She mailed the contract to the surgeon that afternoon, although it was mailed too late for the last mail pickup of the day. The owner's attorney promptly drew up a deed and dropped it in the mail to his client. The surgeon received the contract the next day. After she mailed the contract, she received an offer of $250,000 for her property from her next-door neighbor, who wanted to expand beyond his property line. The owner called her attorney and told him to inform the surgeon that the deal was off. The attorney e-mailed the surgeon, stating that his client had found another purchaser for the property, and that all matters regarding the surgeon's offer for the property were rescinded. The surgeon signed the contract anyway and returned it to the homeowner by registered mail. If the surgeon brings an action to compel the owner to convey the property to him for $200,000, is he likely to prevail? (A) Yes, because the owner signed the land sale contract. (B) Yes, because the contract was effective when the owner placed the document in the mail. (C) No, because the surgeon did not mail the signed contract until after he received the revocation by e-mail. (D) No, because the land sale contract does not contain the complete legal description of the property.

A - Common Law Statute Of Frauds

9. A dairy farmer hired a local company to assemble milking machines that the farmer had purchased. The written contract between the parties provided that the company would assemble and install the milking machines in the farmer's dairy barn within 30 days, in time for the arrival of some additional cows, and the farmer agreed to pay the company $10,000. Three weeks into the job, the company realized that it would lose $2,500 on the job, due to a new wage agreement forced on the company by its employees' union after the contract was executed. The company approached the farmer and told him that the job could not be completed for less than $12,500. After some discussion, the farmer and the company executed an agreement obligating the farmer to pay an additional $2,500 upon completion of the job. The company completed the work on time, but the farmer now refuses to pay the additional $2,500. In a suit by the company against the farmer, which of the following would be the farmer's best defense? (A) This was a contract for services and the modification was not supported by consideration. (B) The modification is voidable because the company knew that the farmer needed the machines up and running in 30 days and took advantage of his duress. (C) A contract cannot be modified once performance has begun, and the attempted modification occurred three weeks into the job. (D) During initial contract negotiations, the company assured the farmer that the milking machines would be assembled and installed for no more than $10,000.

A - Consideration; Legal Value Element

7. A homeowner was postponing shoveling the snow off her driveway one Saturday morning because it was so cold outside. When she heard a scream and ran outside, she found the neighborhood newspaper delivery boy injured, lying by his bicycle in her driveway. Feeling terribly guilty, she took the boy to a local doctor and wrote a note to the doctor stating that, if the doctor would treat the boy, the homeowner would cover all of the resulting costs. The boy later admitted that he was attempting to ride the bicycle with his eyes closed, and that his fall had nothing to do with the snow in the homeowner's driveway. After treating the boy for his injuries until he recovered, the doctor sent the homeowner a bill for his services. When the homeowner refused to pay, the doctor brought an action to recover the amount of his bill. In the doctor's suit against the homeowner, which party is likely to prevail? (A) The doctor, because he gave medical treatment to the boy after receiving the homeowner's note. (B) The doctor, because the homeowner's promise to pay the boy's medical expenses was in writing. (C) The homeowner, because there was no consideration for her promise to the doctor. (D) The homeowner, because she derived no benefit from the medical services rendered to the boy.

A - Consideration; Promissory Estoppel or Detrimental Reliance

6. A landlord owned a three-story apartment building that had a beautiful view of the ocean over a neighbor's vacant lot. The neighbor assured the landlord that he had no plans to develop the vacant lot. Subsequently, the neighbor was killed in an automobile accident, and his will left the lot to his son. Over the landlord's objections, the son began building a ID-story office structure. Once the construction began, the contractor's workers continually swung large girders suspended from a crane over the landlord's building. The landlord complained incessantly because the tenants complained to her that the girders frightened them, but the son did nothing about it. Even when the office structure was only partially completed it put the landlord's apartment building in shadows for the greater part of the afternoon. Her apartments not only no longer had an ocean view, but were dark and devoid of sunlight. The landlord slashed her rents by 40%, but still had difficulty keeping the building fully occupied. The landlord brings a suit against the son seeking all available relief. What relief will a court most likely grant? (A) Damages or an injunction, because the girders repeatedly pass over the landlord's air space. (B) Damages, because of the loss of the ocean view. (C) Damages or an injunction, because of the loss of sunlight. (D) None.

A - Easements

2. A landlord who owned a strip mall entered into a written five-year lease of one of the units with a discount retail perfumery. The lease provided for a monthly rent of $1,000, payable on or before the first day of each month. The perfumery dutifully paid its rent on time for two years and three months. At that time, with the oral permission of the landlord, the perfumery transferred its interest in the remainder of the lease to a dry cleaner in writing, and added a clause requiring the dry cleaner to get permission from the perfumery for any subsequent assignments. The dry cleaner promptly paid rent to the landlord for 14 months, and then asked the landlord to approve a transfer of its interest in the lease to a video rental store. The landlord gave her oral assent. To obtain the perfumery's approval of the transfer to the video store, the dry cleaner wrote a letter to the perfumery, promising that if any problems arose and anyone tried to go after the perfumery for money, the dry cleaner would "make it good." After the perfumery sent a letter back to the dry cleaner agreeing to the transfer, the dry cleaner executed a written transfer of its interest to the video store. The video store promptly paid rent for three months. Having failed to make any profits, the video store ceased paying any rent to the landlord and cannot be located. The landlord has been unable to find anyone interested in the unit. Given that any judgment against the video store would be worthless, from whom can the landlord collect the unpaid rent owed on the lease? (A) Either the perfumery or the dry cleaner. (B) The perfumery only, but the perfumery may recover in turn from the dry cleaner. (C) The perfumery only, and the perfumery has no recourse against the dry cleaner. (D) Neither the perfumery nor the dry cleaner.

A - Landlord Tenant (Difficult)

1. A gallery owner offered to sell a piano to an acquaintance for $400. The acquaintance had been to the gallery owner's house a few weeks prior to the offer and had seen a Steinberg piano in his living room, so she accepted. Unbeknownst to the acquaintance, the gallery owner also owned a Haiwin piano, which he kept at the gallery, and that was the piano that he intended to sell. The acquaintance had never been to the gallery, and the gallery owner was aware that the acquaintance had never seen the Haiwin. If the acquaintance sues the gallery owner to obtain the Steinberg, which party is likely to prevail? (A) The acquaintance, because the gallery owner knew of the ambiguity. (B) The acquaintance, because that was her objective intent. (C) The gallery owner, because there was a mutual mistake. (D) The gallery owner, because he subjectively intended to sell the Haiwin instead of the Steinberg.

A - Misunderstanding; Ambiguous Contract Language

4. To encourage her neighbor to continue pursuing her avocation of photography, an art patron promised to buy her first photographic work that wins a prize in a juried art show. To show that she was serious, the art patron gave her neighbor a contract to sign which stated that she agreed to buy and the neighbor agreed to sell the neighbor's first framed photograph that wins a ribbon (first, second, or third prize) in a juried art show, with payment due within 14 days of delivery. Price was not mentioned in the contract, which was enforceable in all other respects. Both parties signed the contract. Some months later, the neighbor won first prize for one of her photos set in an expensive frame. The parties made arrangements for the art patron to take possession of the piece. The neighbor insisted on a price of $1,200, pointing to the expensive frame and the prestige of the art show that awarded the prize. The art patron instead tendered $700, a good faith approximation of the value of the photo display, and sued when the neighbor refused to deliver it. If the neighbor defends on the ground that no price was fixed in the contract and the parties have been unable to agree on a price, how should the court rule? (A) The art patron is entitled to purchase the photograph at whatever price the court determines to be reasonable. (B) An arbitrator must be appointed to set the price. (C) The art patron is entitled to purchase the photograph for $700. (D) The art patron will have to pay $1,200 for the photograph.

A - Mutual Assent; Offer and Acceptance; Definite and Certain Terms; Definiteness of Subject Matter; Missing Terms.

9. An advertising agency specializing in aerial banners and skywriting signed a contract with a film production company that was premiering a new blockbuster film. The contract provided that the agency would advertise the film by flying over the city towing a giant streamer belonging to the film company heralding the film's catch phrase and title in large letters. This contract specified that the flight was to be conducted on the first Saturday in June at noon (the day of the local premier), and the film company was to pay the advertising agency $500 for the flight. On the designated Saturday, the advertising agency was unable to fly because of a defective fuel pump. The defective condition was entirely unforeseeable and did not occur through any negligence or fault of the agency. The film company did not pay the agency, and each of the parties has sued the other for damages. Which of the following best states the rights and liabilities of the parties? (A) The film company is entitled to recover damages from the advertising agency on account of the agency's failure to fly. (B) The advertising agency is entitled to recover from the film company the $500 contract price, as the incapacity of the airplane was not the agency's fault. (C) Neither party is entitled to recover against the other, because the advertising agency's duty to fly was discharged by impossibility, and the film company's duty to pay was contingent on the agency's flight. (D) Neither party is entitled to recover against the other, because the film company's offer to pay $500 for the flight was in effect an offer for an act, and because the act was not performed, there was no valid acceptance.

A - Objective and subjective impossibility

7. A young woman went to her local shoe shop and selected a pair of shoes. She gave the salesperson cash for the shoes. As the salesperson was putting the shoes into a bag, a robber brandishing a gun entered the store, forced the salesperson to put all of the money in the register into the bag with the shoes, and fled with the bag, the money, and the shoes. After the police had come, the young woman asked the salesperson to get her another pair of shoes. He told the young woman that she would have to pay for them again. The young woman refused. If the young woman sues the shoe shop for another pair of shoes, who will prevail? (A) The young woman, because she did not yet have possession of the shoes. (B) The young woman, because the purpose of the contract had been made impossible by an unforeseen event. (C) The shoe shop, because title to the shoes had already passed to the young woman. (D) The shoe shop, because the contract goods had already been identified.

A - Risk of Loss, Noncarrier

9. A farmer owned a large tract of land. She divided the tract into two parcels: Parcel 1 comprised the northern half on which the farmer built her home, and Parcel 2 comprised the southern portion containing a large orchard of fruit trees. First, the farmer conveyed Parcel 1 by grant deed to her friend "for life, and then to his widow for her life, remainder to his children then alive." Later, the farmer conveyed Parcel 2 to the orchard manager. Subsequently, the farmer died intestate, leaving her sister as her sole heir at law. The house on Parcel 1 is old and in need of repair. The friend proposes to tear down the house and plant fruit trees. The property would be worth substantially more as a fruit orchard than in its present condition. However, the sister feels that the farmer's old house has sentimental value and wants the friend to leave the land as it is. At the time of the deed and at present, the friend is married and has two children. The common law Rule Against Perpetuities is unmodified by statute in the jurisdiction. Does the sister have standing to sue to enjoin the friend from tearing down the house? (A) Yes, because she holds a reversion by operation of law that will take effect on the death of the friend's widow. (B) Yes, because she holds an executory interest that will become possessory if the friend dies without surviving children. (C) No, because the sister has no interest in Parcel 1. (D) No, because the friend was married at the time of the farmer's conveyance and had children.

A - Rule Against Perpetuities

Question 8 An acquaintance asked the defendant to give him a lift downtown because he did not have bus fare. While riding on the defendant's motorcycle, the acquaintance asked to stop at a convenience store to get a bottle of wine, showing the defendant a tire iron in his backpack that he was going to use. The defendant nodded in acknowledgment of what the acquaintance was planning to do. The defendant stopped at the store and waited in the parking lot while the acquaintance went in. He demanded money from the clerk, brandishing the tire iron. The clerk tried to grab a gun under the counter while he was filling a bag with money, and a struggle ensued. The gun discharged, killing the clerk. The defendant heard the gunshot and raced off, but was eventually apprehended. The jurisdiction's criminal code provides that a death caused during the commission of certain felonies, including robbery, is first degree felony murder, for which the death penalty is permitted. The code also permits cumulative penalties for first degree felony murder and for the underlying felony. The defendant was charged and convicted of both robbery and felony murder. After appropriate consideration of all relevant circumstances, the jury imposed the death penalty. On appeal, the defendant challenged both the convictions and the sentence. Assuming the above facts were properly admitted into evidence, how should the appellate court rule? A The defendant's conviction for both offenses should be upheld, but imposition of the death penalty was not proper. B The defendant's conviction for both offenses should be upheld, and imposition of the death penalty was proper. C The defendant's conviction should be overturned under double jeopardy principles because robbery is a lesser included offense of felony murder. D The defendant's conviction for felony murder should be overturned because the circumstances do not establish the necessary degree of culpability

A 8th Amendment - Advanced

Question 7 A manufacturer sued a department store for breach of contract, alleging that the department store failed to pay fora shipment of watches delivered by the manufacturer to the department store in July of that year. At trial, the manufacturer introduced evidence regarding the department store's failure to pay for the watches. In addition, it introduced evidence that the department store failed to pay for a shipment of necklaces, also delivered that July pursuant to a separate contract. At the close of the manufacturer's case, the department store introduced evidence showing that both the watches and the jewelry were defective. The jury returned a verdict for the manufacturer, awarding the manufacturer damages for breach of both contracts. The manufacturer then immediately moved to amend its complaint to conform to the evidence introduced at trial. Should the court allow the amendment? A Yes, because the department store impliedly consented to the introduction of the evidence. B Yes, because the parties entered into the contracts at approximately the same time. C No, because the motion should have been made before the verdict was rendered. D No, because the defendant must have had knowledge of the claim when the complaint was served.

A Amend Complaint- Intermediate

Question 10 Criminal statutes in the state define murder as "the unlawful killing of another human being with malice aforethought, either express or implied," and define voluntary manslaughter as "the unlawful killing of another human being under an extreme emotional disturbance for which there was reasonable explanation or excuse, without express or implied malice aforethought." Another statute provides that "it shall be an affirmative defense to the charge of murder if the defendant proves by clear and convincing evidence that the defendant was unable to control his actions or conform his conduct to the law." The defendant was charged with murder and tried in state court. At trial, he introduced evidence regarding his state of mind at the time of the homicide, including testimony from a psychiatrist. At the conclusion of the case, the court instructed the jury as follows: There are two kinds of homicide in our state: murder and manslaughter. The common elements of both are that the homicide be unlawful-i.e. neither justifiable nor excusable - and that it be intentional. Malice aforethought is an indispensable element of the crime of murder. However, if the prosecution can establish that the killing was both intentional and unlawful, malice aforethought may be presumed unless the defendant proves by a preponderance of the evidence that he acted under extreme emotional disturbance for which there was reasonable explanation or excuse, in which case he shall be liable only for voluntary manslaughter, because extreme emotional disturbance is inconsistent with, and negates the existence of, malice aforethought. The judge further instructed the jury that it could return an acquittal by reason of insanity "if the defendant established by clear and convincing evidence that he was unable to control his actions or conform his conduct to the law." The jury found the defendant guilty of murder. The defendant appealed, claiming that his constitutional rights were violated by the courts instructions as to homicide and by the requirement that he prove insanity by clear and convincing evidence. Were the defendant's rights violated? A Yes, by the courts instructions as to homicide, but not the requirement of clear and convincing evidence for insanity. B Yes, by the requirement of clear and convincing evidence for insanity, but not the instructions as to homicide. C Yes, by both the homicide instructions and the clear and convincing evidence requirement. D No, neither the homicide instructions nor the clear and convincing evidence requirement violated the defendant's rights

A Burden of Proof & Jury Instructions- Advanced

Question 9 A plaintiff sued a defendant over a claimed debt. At the trial, the plaintiff established the existence of the debt and testified that he never received payment. In response, the defendant presents evidence sufficient to establish that she took her check to the post office and sent it to the plaintiff's proper address by certified mail. The defendant offers a certified mail receipt with an illegible signature, which she claims is the plaintiff's signature. The defendant also presents evidence that her basement flooded on March 28, and she claims that she cannot produce a canceled check because her box of canceled checks was destroyed from the water damage. Evidence is also presented that, due to a computer glitch, the defendant's bank cannot reproduce her checking account records for the months of February and March. After the defendant's testimony, which of the following is correct? A The burden of persuasion and the burden of going forward with the evidence are on the plaintiff. B The burden of persuasion is on the plaintiff, but he has no burden of going forward with the evidence. C The plaintiff has satisfied his burden of persuasion, but he has a burden of going forward with the evidence. D The plaintiff has satisfied both his burden of persuasion and his burden of going forward with the evidence

A Burdens- Advanced

Question 2 The state of Blue enacted a statute to protect its faltering lobster industry. The statute provides that no lobster shall be taken from lobster beds lying within three miles of the state shoreline unless the lobster is at least one pound in weight. The statute's one-pound limitation is intended to enable young lobsters to reproduce before being caught. At the same time, Congress enacted a lobster conservation act that provides $5 million for research funds to develop and improve breeding grounds for lobsters. The federal act imposes a special excise tax of $1,000 on each lobster caught in violation of state law if later shipped in interstate commerce. A lobsterman who lives in the state of Green, which is just south of the state of Blue, crossed over into waters lying within three miles of the Blue coastline. He was arrested by state Blue law enforcement officers for taking lobsters that weighed less than one pound. The man defended the charge by challenging the constitutionality of the state Blue statute. Which of the following results is most likely? A The statute will be upheld because it does not violate the Constitution in any way. B The statute will be upheld because the Commerce Clause does not apply when a state is seeking to protect natural resources. C The statute will be held invalid because it violates the Interstate Privileges and Immunities Clause of Article IV, Section 2. D The statute will be held invalid because it is preempted by the Federal Lobster Conservation Act

A Commerce Clause—Proper State Statute - Intermediate

Question 5 An electrician from State A was hired by a commercial builder from State B to install all the electricity in a new strip mall located in State A. Upon completion, the builder refused to pay the electrician, claiming the installation was not up to the building code. The electrician filed a diversity action in federal court sitting in State A against the builder for breach of contract. Although the action was filed within the applicable statute of limitations period, the electrician failed to serve the summons and complaint on the builder until after the statute of limitations on the claim expired. Under the law in State A, an action is "commenced" for purposes of the state's statute of limitations when the defendant is served with the summons and complaint. However, under the Federal Rules of Civil Procedure, an action is "commenced" in federal court when the plaintiff files the complaint in federal court. The builder filed a motion to dismiss the complaint for exceeding the statute of limitations period, arguing the state law governs. How should the court rule? A Grant the motion, because statutes of limitations and rules for tolling statutes of limitations are considered substantive rules, and therefore the state law will apply. B Grant the motion, because the federal rule will change the outcome of the case, and when applying a federal rule that will change the outcome, the federal court must apply state law to discourage forum shopping. C Deny the motion, because the Federal Rules of Civil Procedure control over conflicting state law so long as they are procedural and not unconstitutional. D Deny the motion, because the statutes of limitations and rules for tolling statutes of limitations are considered procedural rules, and therefore, the federal law will apply.

A Erie- Intermediate

Question 6 An attorney was employed by the United States Department of Health and Human Services in a regional office located in a tobacco-growing state. A labor contract between the agency and the clerical workers union contained a policy providing for termination of union employees only for certain specified grounds. The attorney, however, was not a member of the union and not covered by such a policy. The attorney was angered by the regional director's refusal to adopt a no-smoking policy for employees and visitors in the office. She posted a notice in the employee cafeteria ridiculing what she called the hypocrisy of an agency promoting health issues and nonsmoking programs while refusing to provide its employees with those same opportunities. The notice prompted a great deal of debate among the employees and was brought to the attention of the regional director, who was very displeased. Which of the following statements is most accurate regarding the director's right to dismiss the attorney? A The attorney has a liberty interest in the exercise of her First Amendment rights that entitles her to a hearing to contest the grounds of her dismissal. B The attorney has a property interest as a public employee that precludes her from being fired without notice and an opportunity to respond. C The attorney has no right to a hearing because her statements were not an expression of views on public issues. D The attorney has both a liberty interest and a property interest that entitles her to a pre-termination evidentiary hearing

A Government employee-speech related termination- Advanced

Question 4 A victim and his former business partner, the defendant, had a bitter falling out after the victim accused the defendant of embezzling company funds. The defendant threatened to get even. Shortly thereafter, while driving on the expressway, a car swerved suddenly in front of the victim's car. Although the victim applied the brakes immediately, his car failed to stop. To avoid colliding with the car ahead of him, he swerved to the right and smashed into a concrete retaining wall. A passing motorist stopped and came to the aid of the victim. Bleeding profusely from a head wound, and rapidly losing consciousness, the victim said, "l don't think I'm going to make it tried to slow down, but my brakes didn't work. My former partner must have tampered with them to get back at me." With that, the victim lapsed into unconsciousness, and has been in a coma and on life support ever since. A personal injury suit has been filed on his behalf by a court-appointed guardian against the defendant. At trial, can the motorist testify as to the statement made by the victim? A No, because the victim did not know that the defendant tampered with the brakes. B No, because the victim is still alive. C Yes, because the victim thought he was about to die. D Yes, because this is a civil case

A Hearsay Dying Declaration Advanced

Question 3 A plaintiff sued an airline company for negligence for back injuries she sustained when the airplane in which she was a passenger was involved in an accident. The defendant has answered the complaint with a general denial of negligence as well as of personal injuries. Immediately after the accident, the plaintiff was examined and treated by her physician. The physician made an affidavit stating that he had examined the plaintiff the day after the accident and found her to be suffering from a back injury. The physician is now dead. The plaintiff's counsel seeks to introduce the affidavit. How should the judge rule on the affidavit? A Inadmissible, because it is hearsay not within any exception. B Inadmissible, because the affidavit does not state that the injury occurred from the accident. C Admissible, as the prior recorded testimony of the physician. D Admissible, as a statement of present bodily condition made to a physician

A Hearsay- Advanced

Question 5 A decedent's executor filed a wrongful death suit against a railroad company whose freight train collided with the decedents automobile, killing him instantly. After the accident it was impossible for experts to tell whether one of the crossing gates was broken because a vehicle drove through it, or if it broke while being lowered suddenly on top of a vehicle entering the grade crossing. A motorist who did not see the accident but who arrived on the scene just seconds later heard a pedestrian standing near the grade crossing exclaim, "That gate didn't come down on time!" The pedestrian collapsed after making his statement. An emergency medical technician tended to the pedestrian. She told the pedestrian that he had probably suffered a mild heart attack. The pedestrian told the medical technician, "Well guess I'm a lot luckier than that poor fool who rammed his car right through that crossing barrier into the train." Two months later, the pedestrian suffered another heart attack and died. Four months later, the executor's suit came to trial. The plaintiff called the motorist to the stand, and the court allowed him to testify to what he heard the pedestrian say after the accident. The railroad's attorney wishes to call the medical technician to the stand to testify as to the pedestrian's statement when she was treating him. How should the court rule on the medical technician's testimony? A Admissible, but solely for the purpose of impeachment. B Admissible, both for impeachment purposes and as evidence of the positioning of the crossing gate at the time of the accident. C Inadmissible, because the pedestrian is not available to be questioned about the inconsistent Statements. D Inadmissible, as hearsay not within any recognized exception to the hearsay rule

A Impeachment - Advanced

Question 10 A group of parents advocating abstinence among teenagers developed a seminar that it presents to middle school students throughout the country. The seminar includes lectures by trained professionals accompanied by slides, some of which depict explicit sexual activity between nude males and females. Parents are required to give their consent before any child may participate. A school administrator invited the group to present its program. However, a law in the state in which the school is located provides in its entirety, "It is unlawful to sell, give, or display to any person under the age of 17 any lewd or obscene article, picture, or depiction." The local district attorney learned of the planned presentation and threatened to prosecute the school's administrative board and program's presenter under the state statute if they carried out the planned program. The school's administrators seek relief in federal court. Which of the following statements regarding the likely result of the case is correct? A The federal court has power to grant a declaratory judgment that the statute is unconstitutional, either on its face or as applied to the program. B The federal court has power to enjoin the district attorney from prosecuting the administrators and presenter only if there is diversity of citizenship between them and the district attorney. C The federal court is more likely to grant an injunction or declaratory relief after the state criminal prosecution has commenced than beforehand. D Under no circumstances will the federal court enjoin a state criminal prosecution

A Judicial review - Advanced

Question 5 A defendant was involved in an accident in which her car struck the rear end of the car driven by the plaintiff. The police issued tickets to the defendant, charging her with reckless driving and speeding. When the defendant's case came before the traffic court, her attorney entered into a plea bargain with the prosecutor under the plea bargain, the defendant agreed to plead guilty to speeding and to pay a fine of $ 100, and the prosecution agreed to drop the reckless driving charge. Accordingly, the defendant pleaded guilty and the court fined her $100. In the later civil suit, where the plaintiff is seeking damages from the defendant for personal injuries, is the guilty plea before the traffic court admissible? A Yes, because it is a statement by an opposing party. B Yes, because it is a statement against interest. C No, because there is a public policy in favor of plea bargaining to promote court efficiency. D No, because no felony was involved

A Nonhearsay Party Opponent-Advanced

Question 8 A union filed suit against a corporation, known for its anti-union management, asserting that its members were being discharged in retaliation for membership in the union rather than for any failure to perform their jobs properly. Under the pretrial discovery orders, a union employee was allowed to examine all of the records held in the corporation's files concerning discharge of employees for a seven-year period prior to the instigation of suit by the union. The employee sorted through this large volume of material and discovered that persons who were union activists usually had "lack of corporate spirit" listed as their reason for discharge, while other fired workers tended to have more specific grounds for discharge listed, e.g. persistent lateness. The employee developed a chart showing grounds for dismissal of union members versus nonmembers based on the data in the files. At the trial, the union placed the employee on the stand. She testified in some detail regarding how she had conducted her research. The employee brought out the chart and the union's lawyer asked that the chart be admitted into evidence. The corporation's attorney objected. How should the court rule on the admissibility of the chart? A Admissible, because copies of the original documents upon which the chart was based were available to the corporation prior to trial. B Admissible, because the chart is helpful to the trier of fact. C Inadmissible, because it is hearsay not within any exception. D Inadmissible, in the absence of the underlying records having been first introduced into evidence

A Presentation of Evidence Best Evidence Rule - Chart- Intermediate

Question 3 A student activist suggested to her boyfriend that they put a drug in the sugar bowls at their college cafeteria so that students would become sick and the activist's antinuclear group could claim that toxic emissions from a nearby nuclear power plant were the cause. The boyfriend told the student that people could become very ill, or even die, if they consumed too large a quantity of the drug. The student promised that they would only put a small amount of the drug in each bowl and asked the boyfriend to get the drug from his chemistry lab during his chemistry class, which he did. After class, the boyfriend gave the bottle to the student. Before the boyfriend awoke the next day, the student went to the cafeteria and put small quantities of the drug in several sugar bowls. She accidentally put a much larger amount in the sugar bowl at one table. Four persons who sat at that table became extremely ill, and one of the four eventually died. In a jurisdiction applying the common law, how will the boyfriend be liable for the death of the student? A As an accessory before the fact. B As a principal in the first degree. C As a principal in the second degree. D As an accessory after the fact

A Principles and Accessories Before and After - Intermediate

6. A buyer agreed to purchase a portion of a seller's parcel for $25,000. The buyer and the seller orally agreed that the property included in the purchase would be the westerly third of the parcel, and the eastern boundary would be a stone fence that ran from the northern border of the parcel to the southern boundary. Due to a clerical error by the seller's secretary, when the agreement was reduced to writing, the eastern boundary was stated to be the picket fence, which is 275 yards east of the stone fence. If the buyer sues for specific performance of the contract conveying the additional strip of land, will the buyer likely prevail? (A) No, because the court will reform the contract. (B) No, because natural monuments prevail over artificial monuments. (C) Yes, because the language in the contract controls. (D) Yes, because evidence of the oral agreement is inadmissible under the Statute of Frauds.

A Reformation

Question 5 A man who belonged to an ancient religion whose rituals require the use of bald eagle feathers traveled to an area where bald eagles were known to roost. After searching the area, he found a fallen eagle feather and returned home. A few weeks later, the man showed the feather to an acquaintance, who happened to be a park ranger, and explained how the feather was obtained. The ranger informed the man that a state anti-poaching law makes any possession of a bald eagle feather without a special permit a crime. The ranger then cited the man for possession of the feather and confiscated it. At the man's trial for violating the state bald eagle feather possession statute, which of the following constitutional arguments is most appropriate for the prosecution to make? A The statute is a neutral law that only incidentally burdens the man's rights under the First Amendment. B The Free Exercise Clause applies only to belief and not to conduct. C The government has a substantial and important interest in protecting bald eagles and there is no other feasible way to achieve the legislative purpose. D Making an exception for the man on religious grounds would violate the Establishment Clause of the First Amendment

A Religion - Intermediate

Question 2 A city entered into a contract with a cable and Internet provider allowing the provider to be the exclusive cable and Internet provider in the city in exchange for certain rights. To facilitate installation of the provider's fiber optic cables for all of the city's residents, the city passed an ordinance requiring all apartment owners to allow the provider to install cables in their buildings. Installation of the cables involved drilling a hole in one exterior wall of the building and running the cables between the interior walls of the buildings. The owner of an apartment building within the city did not want to allow new cables to be installed in his building, as he had gone through a similar process with another company three years earlier and he believed that the other company had damaged his property. To deter the installation, the owner filed suit against the city, claiming that the ordinance amounts to a taking under federal law. How should the court rule? A For the owner, because the ordinance amounts to a taking without just compensation. B For the owner, because the government does not have a right to require owners of private property to allow private installation of cable lines. C For the city, because the ordinance is a regulation and not a taking. D For the city, because any taking here is de minimis

A Taking - Intermediate

10. An owner of a 240-acre tract of land entered into an agreement to sell 20 acres of the land to his neighbor. The land the neighbor wanted was located in the northwest quarter of the land. The owner wrote out the following statement: "The owner agrees to sell to the neighbor, for $50,000, 20 acres in the northwest quarter of the owner's land, the owner's land beginning at a point in the northeast quadrant of the county on the 16th county survey line, six minutes west of the 98th meridian, and then proceeding due east 1,320 yards and from that point due south to the creek, and then westward along the creek to the point at which it intersects the public road, and then northward along the eastern edge of the public road until the road ends, and then due northward from that point to the original starting point of the survey." The owner had copied the language describing the land from his own deed. The owner then signed the paper and gave it to the neighbor. On the date set for closing, the neighbor tendered $50,000 to the owner. The owner refused to accept the money and refused to convey 20 acres to the neighbor. If the neighbor sues the owner for specific performance, what is the likely result? (A) The owner will prevail, because the writing did not adequately describe which 20 acres were to be sold. (B) The owner will prevail, because the writing was not signed by both parties. (C) The neighbor will prevail, because metes and bounds are a sufficient way to legally describe property. (D) The neighbor will prevail, because the writing satisfies the Statute of Frauds.

A SOF

5. A developer and an investor had been in the real estate business for many years. Because of their long-standing relationship, the developer and the investor, neither of whom was an attorney, often dispensed with certain legal formalities when dealing with each other, thus saving the costs of lawyers' fees and other attendant expenses. The investor owned a parcel of land that the developer was interested in, and she offered to buy it from him for $50,000. The investor accepted the developer's offer, and the parties agreed on June 15 as the closing date. The developer handed the investor a check for $2,500 with "earnest money" written in the memo, and they shook hands on their deal. A few weeks before closing, the developer called the investor and told him she had changed her mind about purchasing the land because of a sudden economic downturn in the area. The investor appeared at the developer's office on June 15 with the deed to the land in his hand. The developer refused to tender the balance due, and the investor sued the developer for specific performance. Will the investor prevail? (A) No, because the agreement does not comply with the Statute of Frauds and is, therefore, unenforceable. (B) No, but the court will allow the investor to keep the $2,500 earnest money as damages. (C) Yes, because the $2,500 payment constituted part performance of the contract. (D) Yes, because the developer and the investor had established a course of dealing.

A SOF

Question 10 A driver took her car to a garage to have her squeaking brakes repaired. The garage was owned by a corporation in which she held shares of stock. Unfortunately, rather than fixing the brakes, the garage made them worse. As a result, the driver had a bad accident when her brakes failed as she was navigating a tight curve. She brought a negligence action against the garage as a corporate entity in federal court, asserting diversity jurisdiction. She filed suit one day before the statute of limitations expired and sewed the garage with process one day after the statute of limitations expired. Eight months later the driver decides to add a shareholder's derivative claim based on corporate misconduct, but the statute of limitations has long expired. May she add the shareholder's derivative claim? A No, because the statute of limitations on the derivative claim has run. B No, because the corporation was not served until after the statute of limitations had run. C No, because the other shareholders did not receive notice of the suit within the statute of limitations period. D Yes, because the original complaint and service of process in the first action were both timely.

A Shareholder Derivative Claim/Amended Complaint-Advanced

Question 5 To stabilize state corn prices, a state purchased large quantities of corn from resident farmers and convened the corn into biodegradable plastics. The state then sold the plastics to state residents at cost and to out-of-state residents at cost plus 25%. An out-of-state corporation purchased biodegradable plastics from the state at a cost substantially below the price other companies charge. Nevertheless, the corporation believes that it is unconstitutional for the state to charge out-of-state purchasers more than resident purchasers. The out-of-state corporation, therefore, brings suit in federal court challenging the state pricing scheme. Assuming that the court has jurisdiction, should it uphold the constitutionality of the pricing scheme? A Yes, because as a market participant the state is free to charge nonresidents more than residents. B Yes, because the state is selling plastics to nonresidents at prices substantially below that of other companies. C No, because the scheme discriminates against nonresidents in violation of the Commerce Clause. D No, because charging nonresidents more for plastics than residents pay violates the Privileges and Immunities Clause guaranteeing benefits of state citizenship

A State as Market Participant -Intermediate

Question 7 The federal accounting office issued a call for competitive bids for a contract to supply 3,000 four-wheel drive utility vehicles without antipollution devices and with engines with a displacement of 4,000 cubic centimeters. A supplier in a state won the contract as low bidder and began manufacture of the vehicles. However, the state's statutes require that automobiles manufactured in that state be equipped with antipollution devices and have a maximum displacement of 2,500 cubic centimeters. The supplier files suit in state court, seeking a judicial declaration that the state statute may not be enforced as to it. How should the court rule? A The statute may not constitutionally be applied to the supplier because to do so would violate the Supremacy Clause. B The statute may not constitutionally be applied to the supplier because to do so would violate the Contracts Clause. C The statute may not constitutionally be applied to the supplier because to do so would violate the Commerce Clause. D The statute may constitutionally be applied to the supplier

A Supremacy Clause- Advanced

Question 1 A pedestrian was injured in a car accident involving two cars. The pedestrian filed a negligence action in federal district court against the first driver, seeking $100,000 in damages. The pedestrian is a citizen of State A and the first driver is a citizen of State B. The first driver then filed a third-party claim against the second driver, claiming that the second driver is responsible for half of the harm caused to the pedestrian and seeking to recover half of any liability the first driver is found to have to the pedestrian. The second driver is a citizen of State A. Does the federal court have subject matter jurisdiction over the third-party claim asserted by the first driver against the second driver? A Yes, because the court has supplemental jurisdiction over the third-party claim. B Yes, because the court has diversity of citizenship jurisdiction over the third-party claim. C No, because the amount in controversy in the third-party claim is too small. D No, because complete diversity of citizenship is lacking.

A Third party claim-Intermediate

Question 3 A pedestrian was injured in a car accident involving two cars. The pedestrian filed a negligence action in federal district court against the first driver, seeking $100,000 in damages. The pedestrian is a citizen of State A and the first driver is a citizen of State B. The first driver then filed a third-party claim against the second driver, claiming that the second driver is responsible for half of the harm caused to the pedestrian and seeking to recover half of any liability the first driver is found to have to the pedestrian. The second driver is a citizen of State A. Does the federal court have subject matter jurisdiction over the third-party claim asserted by the first driver against the second driver? A Yes, because the court has supplemental jurisdiction over the third-party claim. B Yes, because the court has diversity of citizenship jurisdiction over the third-party claim. C No, because the amount in controversy in the third- party claim is too small. D No, because complete diversity of citizenship is lacking.

A Third-Party Claim, Supplemental Jurisdiction- Intermediate

Question 8 A plaintiff sued a city for damages to his motorcycle that occurred when a bolt on the brush of a street sweeper came loose, allowing the brush to become detached and roll onto his driveway. The city has a rule that any employees must file an accident report with the city within 12 hours of any accident. At trial, counsel for the plaintiff demands that the city produce the original handwritten report of the street sweeper operator, which the city does. The plaintiff then calls the operator to the stand as an adverse witness. The operator identifies the report as his and testifies to the existence of the rule requiring such reports and that he routinely fills them out only a few minutes after the accident. However, the operator does not recall all the details in his report. Because the report states that a faulty bumper brush bolt caused the accident, the plaintiff offers it into evidence. The city objects on appropriate grounds. Is the report admissible? A Yes, as nonhearsay. B Yes, because the operator cannot remember the details in the report. C No, because the record was not prepared in the course of the city's primary business. D No, because only statements of fact in business records are admissible, and the conclusion that the bolt was "faulty" and that it "caused" the accident are not statements of fact

A Vicarious Admission Nonhearsay Internal Company Accident Report-Advanced

10. A driller owned a large tract of land on which she began to drill for oil, but all of her exploratory wells were nonproductive "dry holes." The driller was certain that there was oil in the area, and she asked the neighboring landowner to grant her a lease to drill on his land. The landowner turned down the driller's offer. After the landowner's refusal, the driller drilled an exploratory well on her property. However, the driller drilled the well on a slanted angle, so that she was actually drilling under the landowner's land, even though her rig was located on her property. The driller struck oil, but shortly thereafter the landowner discovered that the oil was coming from underneath his land. Does the landowner have an action for damages against the driller? (A) Yes, because the driller has invaded the landowner's subterranean rights. (B) Yes, but only if the driller's drilling interferes with the landowner's use and enjoyment of his land. (C) No, because oil is a free-flowing liquid and may be captured wherever it flows. (D) No, because the driller's action does not interfere with the landowner's right to drill for oil on his land.

A, Right to exclude, trespass, private nuisance

5. A landowner inherited a parcel of land, free of encumbrances, and promptly recorded his deed. The landowner took out a $100,000 mortgage on the land with a bank to pay for improvements to the property. The instrument was properly recorded. The landowner regularly made the scheduled payments on the mortgage. Subsequently, the landowner decided to further improve the property and took out another mortgage with a financing company for $50,000. The instrument was properly recorded. The landowner then sold the land to a buyer subject to both mortgages. The buyer procured another loan of $100,000 secured by a mortgage on the land from a lender. The lender knew about the bank's mortgage, but the buyer did not inform her of the financing company's mortgage. The lender lent the money to the buyer on the understanding that the buyer would use the money to pay off the bank's mortgage, placing the lender in first priority. The buyer promptly paid off the bank but made no further payments to the financing company. The financing company initiated steps to foreclose on the land. The lender brings an appropriate action seeking to have her rights declared superior to those of the financing company. The state in which the land is located has the following statute: "Any conveyance of an estate in land, other than a lease for less than one year, shall not be valid against any subsequent purchaser whose conveyance is first recorded." If the court rules in the lender's favor, what is the likely reason? (A) The lender could reactivate the first mortgage and step into the shoes of the bank. (B) The balancing of equities clearly favors the lender. (C) The lender lacked actual knowledge of the second mortgage. (D) The lender received no benefit from the financing company, and therefore cannot be burdened by the financing company's demands

A, foreclosure, priority of first vs second mortgage

3. A music promoter planned to open a discotheque and bar on the outskirts of town. He hired a builder to build it, with a completion date of September 15. Although the promoter was very optimistic that the disco would be a big success, its profitability could not be determined with certainty. First year profits were estimated to be about $1,000 per day. To encourage the builder to work in a timely manner, the contract included a liquidated damages clause, providing that the builder would pay the promoter $10,000 per day for each day the contract ran over its completion date. The builder's work progressed smoothly and would have been finished on time, except that the builder failed to place in a timely manner his order for the disco's specially manufactured dance floor lighting. Consequently, the work was not completed until October 15. The promoter sued the builder for breach of contract. The builder called a witness who testified that the disco would have received $30,000 in income during that 30-day period and would have expended $20,000, leaving a profit of $10,000. How much should the builder be required to pay to the promoter in damages? (A) $10,000, representing the disco's lost profits. (B) $30,000, representing the disco's lost income. (C) $300,000, representing damages provided in the contract. (D) $310,000, representing damages provided in the contract, plus lost profits.

A- Monetary Remedies

3. A bulk retailer of accessories for musical instruments placed an advertisement in a trade magazine popular with those in the music business, offering for sale 50-count boxes of a particular type of mouthpiece for use with the French horn, minimum purchase 10 boxes, at $100 per box. In response to the advertisement, the owner of a large store that sold brass and woodwind instruments in its shop and over the Internet sent a written order to the bulk retailer for 12 boxes (50-count) of the mouthpiece. In his letter that accompanied the order, the store owner stated that he would send the bulk retailer his payment of $1,200 upon delivery. The letter also said that the mouthpieces must fit onto three specified models of French horn. The day after receiving the written order and letter from the store owner, the bulk retailer shipped 12 boxes (50-count) of the mouthpiece to him. Accompanying the invoice on the boxes was a letter from the bulk retailer stating that the mouthpieces are compatible with two of the models of French horn, but that the retailer makes no warranties as to the compatibility of the mouthpieces with any other model of French horn. Shortly after accepting shipment of the boxes, the store owner realized that the mouthpieces did not fit onto the third model of French horn that it had specified and instituted an action against the bulk retailer. Which of the following statements would offer the strongest support in favor of the store owner's position? (A) The store owner's letter was an offer, and shipment of the units was an acceptance. (B) The store owner's letter was an offer, and the bulk retailer's letter accompanying the invoice was an acceptance. (C) The bulk retailer's letter was an offer, and acceptance of the units by the store owner was an acceptance of the offer. (D) Shipment of the units was a counteroffer, and acceptance of the units by the store owner was an acceptance of the counteroffer.

A- Mutual Assent; Offer and Acceptance; Acceptance Under Article 2

Question 3 A pedestrian was struck and seriously injured by a car driven by an intoxicated driver. The driver had been served several alcoholic drinks by a bartender at a local bar. The pedestrian sued the bartender in a jurisdiction that does not have a dramshop act. Is the bartender vicariously liable for the pedestrian's injuries? A No, because the driver acted recklessly by driving while intoxicated. B No, because there is no dramshop act in the jurisdiction to impose liability. C Yes, because there is no dramshop act in the jurisdiction to limit liability. D Yes, because the intoxicated driver caused the pedestrian to suffer personal injuries

B (Dramshop Act)

Question 9 A driver and his passenger were involved in an automobile accident when the driver ran a red light and crashed into another car. Due to a manufacturing defect in the automobile's airbag system, the passenger side airbag did not deploy. The passenger was killed on impact. The passenger's estate brought suit against the driver and the airbag's manufacturer. At trial it is established that the driver was negligent in running the red light. What effect would such proof have on the claim of the passenger's estate against the airbag manufacturer? A It would reduce recovery by the estate if the action against the manufacturer is based on negligence. B It would bar recovery by the estate if the trier of fact finds that the driver was the sole legal cause of the passenger's death. C It would bar recovery by the estate if it is shown that the driver is the sole legal heir of the passenger's estate. D It would have no effect on recovery by the estate as long as the action against the manufacturer is based on strict liability.

B (Actual and Proximate Cause)

Question 5 The owner of a corner lot allowed a hedge on his property to become overgrown, obstructing the view of motorists at that corner. Two motorists were driving inattentively and each ran a stop sign at the intersection bordering the lot. Their cars collided in the intersection and one of the motorists was injured. She sued the owner of the lot. The jury determined that the lot owner was 10% at fault and each of the motorists was 45% at fault. Will the injured motorist recover damages from the lot owner? A Yes, because she was not more than 50% at fault. B Yes, because it was foreseeable that motorists could be injured if the hedge was not cut back. C No, because the other motorist's negligence was a superseding cause of her injuries. D No, because the lot owner's fault was slight compared with the motorist's fault

B (Apportionment)

Question 4 A sailor steering his sailboat through a channel was nearly swamped by a large cabin cruiser. The sailor made an obscene gesture and shouted epithets at the captain of the larger boat, who responded by swinging his boat around and heading at high speed directly at the sailboat's bow. The sailor was convinced that the boats would collide, so he steered close to the edge of the channel and abruptly ran aground on a shallow sand bar. The sailor was extremely upset but otherwise uninjured. His boat was not damaged by hitting the sand bar. If the sailor brings an appropriate action against the captain for damages, what is the probable outcome? A The sailor will win, because he suffered severe emotional distress from the captain's conduct. B The sailor will win, because he believed that the captain's maneuvers threatened imminent danger of harm to him. C The captain will win, because the sailor suffered no physical injury or property damage. D The captain will win, because the sailor was responsible for provoking the captain during the relevant events

B (Assault Damages)

Question 3 A backgammon player was upset after losing a match against the club champion. Rushing out of the club, he inadvertently grabbed the champion's board, which looked very much like his own but which was much more expensive. The player left the backgammon board in the trunk of his car, as was his usual practice. During the night, the car was stolen and along with it, the champion's expensive backgammon board. In an action by the champion against the player to recover the value of the backgammon board, is the champion likely to recover? A Yes, because when the player took the backgammon board he committed a trespass to the champion's chattel. B Yes, because when the backgammon board was stolen along with the car, the player became liable for conversion of the champion's chattel. C No, because the player believed in good faith that the board was his when he took it from the backgammon club. D No, because the backgammon board was lost through no fault of the players

B (Conversion)

Question 8 A former college baseball player who played in the College World Series 20 years ago was named in a radio broadcast as committing a key error that cost his team the championship. In fact, it was another ballplayer who committed the error. If the ballplayer brings a defamation action against the broadcaster and the court finds that he was defamed, to what damages would he be entitled? A Nominal damages only, unless the ballplayer can show actual pecuniary loss. B General damages, even without proof of actual injury. C Only damages based on competent evidence of actual injury. D No damages, unless the ballplayer can prove actual malice on the part of the broadcaster.

B (Defamation Damages)

Question 8 At the request of local police, airport security officials made random searches of passenger luggage for contraband as it was being sent to the baggage claim area, although they did not have legal authority to search bags without a warrant. The searches were conducted so that there was no delay in the luggage being released to those claiming their bags. A traveler went to the baggage claim area but his luggage, which contained a number of valuables but no contraband, did not appear. It had been selected for a search but the X-ray machine had malfunctioned and security officers were having difficulty getting it unlocked to search it by hand. When the traveler inquired about the luggage, he was told that it was being inspected and that he would have to remain in the area if he wanted to claim it when it was released, and that its return could not be guaranteed if he was not around when it was released. About 30 minutes later, the luggage was returned to the traveler with an apology for the delay. The delay caused the traveler to miss his train to the city, so he had to pay for cab fare. Assuming there are no issues of governmental immunity, can the traveler bring an action against the airport security officials for false imprisonment? A Yes, because the traveler suffered harm as a result of the delay in releasing his luggage. B Yes, because the traveler reasonably believed that he would not get his luggage back if he left the airport. C No, because the traveler was not restrained from leaving the airport. D No, because the delay in releasing the luggage was not done for the purpose of restraining the traveler

B (False Imprisonment)

Question 7 An investor who owned several thriving shopping malls was negotiating to purchase a local mall from the company that currently owned it. A staff attorney for the state transportation department who shopped at the mall regularly learned of the negotiations and contacted the investor. The mall had deteriorated noticeably during the time the current company had owned it and the attorney believed that new ownership would revitalize the mall considerably. Although the attorney had no information to support this, she told the investor that the state was currently planning to construct a new interchange for the turnpike only three blocks from the mall. The investor went ahead with the purchase, believing that the new interchange would boost sales. In fact, no interchange was being considered by the state at that time, and nothing that the investor did after he purchased the mall could stem the decline in sales. He ended up selling the property at a substantial loss several years after the purchase. Does the investor have a cause of action against the attorney for his losses? A. Yes, for negligent misrepresentation, because the owner made a business transaction in reliance on the attorney's statements. B. Yes, for intentional misrepresentation, because the attorney was aware that she did not know whether the state was planning an interchange. C. No, because the attorney's statement pertained to a future event that may not be justifiably relied upon. D. No, because the attorney made her statement to the owner gratuitously.

B (Intentional Misrepresentation)

Question 2 The owner of a boat took two friends out on a lake near his home. One of his friends was driving the boat when it struck a partially submerged rock that the owner of the boat had forgotten to tell him about. The owner of the boat and the other passenger were injured; the driver of the boat was not hurt. In a jurisdiction that applies joint and several liability with comparative contribution, the passenger brought suit against both the boat owner and the driver, and the boat owner also sued the driver. The jury determined that the boat owner was 55% at fault and suffered $10,000 in damages, the driver of the boat was 45% at fault, and the injured passenger suffered $100,000 in damages. After entry of judgment, the boat owner paid the passenger her total damages of $100,000, while the driver of the boat has paid nothing. How much, if anything, can the boat owner recover from the driver? A $45,000, because the driver was 45% at fault. B $49,500, because the driver was 45% at fault and the boat owner suffered $10,000 in damages. C $50,000, because the boat owner and the driver are jointly liable. D Nothing, because the boat owner was more at fault than the driver.

B (Joint and Several Liability and Comparative Contribution)

Question 10 A homeowner invited guests over for an evening pool party. One of the guests jumped into the pool without seeing that a swimmer was in the water right below him. He struck the swimmer on the head, knocking her unconscious. Other guests pulled the swimmer out of the water and attempted to revive her. She survived but suffered permanent injury. The swimmer brought an action against the homeowner for her injuries in a jurisdiction that applies the traditional rules for landowners and possessors of land. At trial, evidence established that when the guest jumped into the pool, he did not see the swimmer in part because a light in the pool near the point of impact was not working at the time, but the homeowner was not aware that it was not working. No other evidence was presented regarding the condition of the pool. Other evidence established that the rescue efforts of the other guests caused greater injury to the swimmer than the initial impact. At the end of the presentation of the evidence, the homeowner moved for a directed verdict. Should the homeowners motion be granted? A Yes, because the swimmer's injuries were caused by the conduct of third persons. B Yes, because the homeowner was not aware that the light in the pool was not working. C No, because the jury could determine that the homeowner failed to exercise reasonable care in activities on the property. D No, because the jury could find that any negligence on the part of the rescuers was foreseeable and therefore does not cut off the homeowner's liability.

B (Landowner Duties)

Question 9 The owner of a small fleet of taxicabs had his cabs serviced by a national chain of auto service centers. One of his cabs went through a stop sign when its brakes failed without warning. The ensuing collision seriously injured the passenger. An investigation revealed that brake repairs had been made on the cab a week before, but the service center's mechanic had used the wrong parts and had made numerous errors in reassembling the brakes. If the passenger sues the cab company owner for her injuries, who should prevail? A The passenger should prevail, unless the jury determines that the owner exercised a high degree of care in selecting the service center for maintenance of his cabs. B The passenger should prevail, because the owner breached his duty to her to provide a safe vehicle in which to ride. C The owner should prevail, because he had no reason to know that the service centers mechanic would be negligent. D The owner should prevail, because he is not vicariously liable for the negligence of an independent contractor

B (Nondelegable duty and Vicarious Liability)

Question 10 A driver struck a 10-year-old boy who had darted into the road during a game of tag. After the accident, the boy's mother refused to take him to a physician because of moral scruples against the medical profession. As a result, the boy's injuries were more severe than they would otherwise have been. What is the boy's best argument for recovering all of his injuries in an action against the driver? A The doctrine of avoidable consequences at most bars recovery for the aggravation of but not for the original injury itself. B Any negligence on the mother's part is not to be imputed to her child. C Victims have no duty to take steps for their own safety after the accident. D Defendants must take their victims as they find them, including their mothers' attitudes toward physicians.

B (Parent/Child and Imputed Negligence)

Question 5 A salesman in a highly visible and competitive field went to the police station to post bond for his son, who had been arrested for possession of a small quantity of narcotics. A photographer for the local newspaper who was at the police station took a picture of the salesman flanked by two bulky police officers. The photo, which looked like the pictures of alleged criminals being taken into custody, ran on a quarter of the front page because it was a slow news day. The photo was accompanied by a very small caption giving the salesman's name and stating that his son had been arrested for possession of narcotics. The salesman's boss was hypersensitive about the reputation of his company and fired the salesman after he saw the picture in the newspaper. If the salesman brings an invasion of privacy action against the newspaper, what is the most likely basis? A. Intrusion upon seclusion B. False light publicity C. Public disclosure of private facts D. Appropriation of plaintiff's picture for commercial purposes

B (Privacy Torts - Invasion of Privacy)

Question 4 A man purchased a large flat screen plasma television and decided to mount it on the ceiling over his bed. The manual that came with the product included detailed instructions and illustrations on how to mount the television on different types of walls, along with all the required hardware, but contained neither instructions nor warnings regarding mounting on the ceiling. The man carefully followed the wall-mounting instructions and was satisfied that it would hold. In fact, however, the mounting was not appropriate for ceilings. The next night, a woman who was the man's overnight guest was seriously injured when the television came loose and fell on the bed Will the woman prevail in a suit against the company that manufactured the television? A Yes, because the manufacturer had a duty to include warnings for all potential placements of its product. B Yes, if the manufacturer knew that its television was sometimes mounted on ceilings rather than walls. C No, if the manufacturer's manual had all of the customary warnings for this type of product. D No, because the man was negligent in mounting the television on the ceiling

B (Products Liability)

Question 5 A gardener purchased a new riding lawnmower from a gardening store. The gardener used the lawnmower that afternoon to mow her lawn. Although the instructions recommended against it, she ran the mower at full throttle around her yard because she was in a hurry to finish. As she was nearing the edge of her property, she attempted to turn the mower. However, the steering wheel locked and she was unable to turn, hitting into a low wall on the border of her property. The collision caused the mower to overturn and the gardener was injured. The gardener asserts a claim against the gardening store based on strict liability in ton in a jurisdiction that follows traditional contributory negligence rules. At trial, the gardener presents evidence that the steering locked because of a defect present when the mower left the manufacturer's factory. The gardening store presented evidence that it carefully inspected the mower before selling it. Will the gardener likely prevail? A Yes, because the steering locked while the gardener was riding the mower. B Yes, because the steering locked as a result of a defect present when the mower left the manufacturer's factory. C No, because the gardener contributed to her own injury by running the mower at full throttle. D No, because the gardening store carefully inspected the mower before selling it

B (Strict Liability)

1. The proprietor of a food brokerage entered into oral negotiations with a manufacturer of gourmet food products for restaurants and select retail outlets. The proprietor wished to secure an exclusive distributorship for the manufacturer's products in the six New England states. At the end of the first stage of oral negotiations, the parties had come to an agreement on the major points, and only a few minor points of disagreement remained. Both, however, were anxious to begin distribution of the food products in New England, and the manufacturer assured the proprietor, "Don't worry about it; we'll work these things out." Assuming from this that he would be the New England distributor for the food products, the proprietor leased larger facilities, bought a number of trucks, and hired new workers. Shortly thereafter, the manufacturer informed the proprietor that another distributor, and not the proprietor, would receive the New England distributorship. If the proprietor prevails in a suit against the manufacturer, it will most likely be because the court applies which of the following theories? (A) Implied-in-fact contract. (B) Promissory estoppel. (C) Unjust enrichment. (D) Quasi-contract.

B - Consideration; Promissory Estoppel or Detrimental Reliance

1. A landowner conveyed his land to his wife, son, and daughter "as joint tenants with right of survivorship." The daughter then conveyed her interest to a friend. The wife subsequently executed a will devising her interest to the daughter. Then the son mortgaged his interest to a lender, who promptly and properly recorded the mortgage. The wife died, then the daughter's friend died, leaving a will that bequeathed her entire estate to the daughter. The daughter and the son survived. If the jurisdiction follows the title theory, who owns what interest in the land? (A) The lender owns the fee simple. (B) The lender and the daughter own unequal shares as tenants in common. (C) The son and the daughter own unequal shares as tenants in common. (D) The son and the daughter own equal shares as joint tenants.

B - Conveyance, title theory, tenants in common, right of survivorship

Question 4 A gang member threatened to kill the defendant unless he robbed a convenience store and gave the proceeds to the gang member. The gang member also demanded at gunpoint that the defendant kill the clerk to prevent identification. In abject fear of his life, the defendant did everything that the gang member requested. If the defendant is arrested and charged with murder and robbery in a common law jurisdiction, what result? A The defendant should be convicted of murder and robbery. B The defendant should be acquitted of the robbery and convicted of murder. C The defendant should be convicted of robbery, and the killing will be reduced to voluntary manslaughter. D The defendant should be acquitted of the robbery, and the killing should be reduced to voluntary manslaughter.

B Coercion/Duress - Intermediate

9. An environmentalist divided her 25-acre property into 100 quarter-acre residential lots. At the time the environmentalist sold her lots, there was a recycling center about one mile from the western boundary of the development. She included in the deed of all 100 grantees the following provision: Grantee covenants for herself and her heirs and assigns that all aluminum cans, glass bottles, and grass clippings of Grantee and her heirs and assigns shall be recycled. This covenant runs with the land and shall remain in effect as long as there is a recycling center within five statute miles of the development. A buyer purchased a lot in the development. Her deed contained the recycling clause. Two years later, the buyer decided to give the property to her niece as a gift. The niece's deed to the property contained the recycling covenant. Shortly after the niece took possession of the house, the recycling center moved its location to a new site about four and a half miles from the development. When the niece put the house up for sale, she said nothing to prospective buyers about recycling. The house was purchased by a veteran who had lost the use of his legs. The veteran's deed did not contain the recycling clause, and he hired a local disposal service to carry away his garbage and a landscaper to maintain the yard. The landscaper bagged the grass clippings and they were removed by the disposal service, which put all the trash and clippings in a landfill. When the veteran's neighbors informed him of his duty to recycle, he told them that he knew nothing of the covenant and that it would be difficult for a person in his physical condition to haul cans, bottles, and clippings to the recycling center. Unfazed, the neighbors filed suit to require the veteran to comply with the covenant or pay damages. The veteran's best defense is which of the following? (A) The veteran's deed did not contain the covenant. (B) The covenant does not touch and concern the land. (C) An intelligent inspection of the neighborhood would raise no inference that the covenant existed. (D) The veteran's physical condition requires a balancing of hardships by the court.

B - Covenants, touch and concern the land

7. A homeowner and a builder entered into a written contract to build a sauna in a spare room in the homeowner's home at a cost of $3,000. The contract contained a clause stating that the builder will not begin construction without prior approval of the plans by the homeowner's certified public accountant. The builder submitted his designs to both the homeowner and the accountant. The homeowner liked the plans, but the accountant did not and withheld his approval. The builder asked the homeowner whether she wanted him to submit new designs. The homeowner told the builder orally, "No! Your designs are great! My accountant is crazy! You go right ahead and construct the sauna." The builder constructed the sauna. The homeowner now refuses to pay the builder, citing the clause requiring approval by the accountant. If the builder sues the homeowner, what will the builder likely recover? (A) The full contract price, because the accountant's approval was not a condition precedent for the contract to take effect. (B) The full contract price, because once the builder began building the sauna after speaking to the homeowner, the homeowner did nothing to stop the builder. (C) The reasonable value of the builder's service and materials, because otherwise the homeowner would be unjustly enriched. (D) Nothing, because the homeowner's oral statement will be excluded by the parol evidence rule.

B - Excuse of Condition by Waiver or Estoppel

10. A new college graduate entered into an oral agreement with a freshman to lease the freshman her mini-refrigerator for a term of four years. The freshman was to pay the graduate $20 a month, of which $10 of the monthly charge was to be paid directly to the graduate's parents, in satisfaction of a debt the graduate owed her parents. While the graduate was putting the agreement into writing she accidentally did not include the agreement to pay her parents directly. The freshman also failed to notice that the direct payment provision was missing before she signed the contract, which the graduate signed. If the parents bring an action against the freshman, which of the following will have the greatest effect on the outcome? (A) Whether the parents were a party to the agreement between the graduate and the student. (B) Whether the agreement between the graduate and the freshman was completely integrated. (C) Whether the graduate was negligent in not discovering that the agreement omitted mention of the payment of money directly to her parents. (D) Whether the freshman was negligent in not discovering that the agreement omitted mention of the payment of money directly to the parents.

B - Is the Writing an "Integration"

10. A brother and a sister held real property as joint tenants. The sister was involved in an automobile accident and was sued by a motorist who had received serious bodily injuries. The jury ruled against the sister and assessed a large damages award that the sister was unable to pay in full. Therefore, the motorist went back into court and secured a statutory lien on the property. Shortly thereafter, the sister died. What are the respective interests of the brother and the motorist in the property? (A) The brother is the sole owner of the property, but the property is subject to the motorist's statutory lien. (B) The brother is the sole owner of the property, and the property is not subject to the motorist's statutory lien. (C) The brother and the motorist own the property as tenants in common. (D) The brother and the motorist own the property in joint tenancy.

B - Joint Tenancy

2. In answer to a radio advertisement, a teenager two months shy of his 18th birthday contracted to buy a late model car from a car dealership. The agreement required a $1,500 down payment with the remainder of the $7,200 price to be paid in monthly installments to a local finance company. The teenager's first eight payments were made regularly until his driver's license was suspended. He then informed the company that no further payments would be forthcoming. The finance company sued for the remaining payments. The age of majority in the teenager's state is 18 years. Would the teenager be liable for the balance of the payments? (A) Yes, because the car dealership was liable on the contract from the outset, notwithstanding his minority. (B) Yes, because he kept the car for six months after reaching the age of majority. (C) No, because he was a minor at the time of contracting, and the contract was voidable by him. (D) No, because he informed the finance company in a timely manner after his driver's license was suspended.

B - Minority; Legal Incapacity to Contract

Question 6 The defendant is on trial for assault with a deadly weapon. The sole prosecution witness is the victim, who testifies as to his version of the events leading up to and including the charged assault. The defense's first witness contradicts the victim's testimony that the defendant engaged in an unprovoked attack. The witness testifies that the victim pulled a knife on the defendant and that the defendant, in defending himself, wrested the knife away and accidentally stabbed the victim. The defense's next and final witness intends to testify that the defendant's reputation in the community for honesty and veracity is very good. Aware of the intended testimony, the prosecutor moves in limine to exclude it. A For the state, because the defendant may not introduce evidence of his character to prove that he acted in conformity therewith. B For the state, because the testimony as to the defendant's honesty and veracity is irrelevant. C For the defendant, because a criminal defendant may put his character in issue. D For the defendant, because a criminal defendant's reputation for honesty and veracity is always at issue

B Character Evidence- Intermediate

6. A print cartridge company sent a letter to a business office offering to supply a free premium printer if the business office would agree to purchase all the print cartridges the office would need from the print cartridge company. This letter arrived in the hands of the business owner on the same day the office printer failed. The business office had experienced a slow month and the business owner was trying to decide whether to pay the office rent for the month or fix the printer. Based on the offer, the owner paid the rent. A week after she put the rent check in the mail, the owner received a second letter from the print cartridge company indicating that the printer program was being canceled due to a lack of printers. The next day, before she read the second letter, the owner mailed her acceptance letter to the print cartridge company. The print cartridge company refused to supply the business owner with a printer. If the business owner brings a breach of contract action against the print cartridge company, how should the court rule? (A) For the print cartridge company, because its offer to the business office was not sufficiently definite. (B) For the print cartridge company, because its revocation letter was received by the business owner before she dispatched the acceptance letter. (C) For the business owner, because she mailed her acceptance letter without being aware that the print cartridge company had revoked its offer. (D) For the business owner, because the owner used funds to pay the office rent in reliance on the print cartridge company's offer.

B - Mutual Assent; Offer and Acceptance; Termination of Offer by Offeror, revocation

9. A widow wanting to give her daughter a special wedding gift entered into a written agreement with a contractor to build a house for $300,000 on property she owned. Detailed specifications regarding the plans and materials to be used were included in the written agreement between the widow and the contractor. Just as the contractor was about to begin construction of the house, he discovered that an underground river bisected the widow's property, leaving insufficient subterranean support to construct the house as planned. Given this discovery, the contractor refused to build the house for $300,000. If the widow files suit demanding specific performance or damages from the contractor, which of the following additional facts, if proven, would most favor her case? (A) It is physically possible to build the house according to the original specifications, but it would add $1 million to the contractor's costs. (B) The detailed specifications in the agreement had been drawn up by the contractor, as were other blueprints and plans for the house. (C) Neither the widow nor the contractor had reason to know of the underground river before the contract was signed. (D) The daughter knew of the contract between the widow and the contractor and turned down other opportunities to purchase suitable housing in the area in reliance on the contract.

B - Mutual Mistake: Assumption of Risk

4. On November 5, an athletic shoe manufacturer entered into a written contract with a shoe store to carry its running shoes. The contract included a provision that "5% of the proceeds attributable to the sale of the manufacturer's shoes by the shoe store during the month of February (American Heart Month) each year would be donated to the local hospital's cardiovascular wing." The day after the parties signed their contract, the store owner informed the hospital of the planned donation and indicated that the hospital could expect to receive about $1,500 in early March. In anticipation of the donation, the hospital purchased a new heart monitor on January 5. Because the Christmas sales season had been poor, on January 15 the manufacturer and the shoe store agreed to modify their contract to eliminate the provision for payments to the hospital's cardiovascular wing. When the hospital did not receive the payment in March, it discovered that the parties had modified the agreement. If the hospital brings an action against the shoe store to recover 5% of the proceeds from the sale of the shoes, is it likely to prevail? (A) Yes, because the hospital's rights vested when it learned of the original agreement. (B) Yes, because the hospital detrimentally relied on the parties' agreement. (C) No, because the hospital's rights had not vested when the modification was made. (D) No, because the payment is a gift, and the hospital is a donee-beneficiary.

B - Third party beneficiary, incidental/intended, vesting

3. An insurer offered a plan to cover an insured's catastrophic illnesses for the remainder of the insured's life in exchange for a large one-time payment at the inception of coverage. Because the program was experimental, the insurer would accept only a fixed number of applications during the enrollment period. A recent retiree in good health was one of the applicants accepted, and he enrolled in the program. He paid the one-time premium of $30,000 a few days before coverage began. The day after his coverage started, he was struck by a bus and killed. The executor of the retiree's estate reviewed the policy and immediately notified the bank to stop payment on it. The insurer then filed suit against the retiree's estate. Will the court compel the estate to pay the premium to the insurer? (A) Yes, because the insurer necessarily declined to take another applicant during the enrollment period because of the retiree's promise to buy the policy. (B) Yes, because the risk of the timing of the retiree's death was assumed by both parties and built into the cost of the contract. (C) No, because the purpose of the contract between the retiree and the insurer had been frustrated. (D) No, because it is unconscionable for the insurer to have charged the retiree so much for so little value received.

B -Impossibility, frustration, assumption of risk

4. A mother purchased 80 acres of desert land over 30 years ago. The deed was properly recorded. Although her family had never even visited the land, the mother had described to them the little two-room cabin that sat in the middle of the parcel near the dry streambed. Ten years ago, the mother's son found what he was certain was the little cabin, and over the next few years he built a barn, a greenhouse, and some corrals, all enclosed by a sturdy wire mesh fence. The area bounded by the fence, containing all the structures, occupied about two acres of the 80 owned by the mother. Three years ago, the mother died, validly devising the 80 acres to the son. The son entered into a contract for sale of the two acres, describing it in detail with reference to the structures and nearby landmarks. The purchaser's surveyor discovered that the son had settled onto a completely different parcel from the one owned by the mother. The purchaser immediately announced that he would not proceed with the sale contract. The state's statutory period for establishing adverse possession is five years. If the son brings an action for specific performance of the sale contract, for whom should the court rule? (A) The purchaser, because the son does not own the land he is purporting to sell. (B) The purchaser, because the son does not have marketable title to the land he is purporting to sell. (C) The son, if he conveys by quitclaim deed. (D) The son, because the description in the contract of sale is sufficient to identify the property and need not be as accurate as one contained in a deed conveying land.

B Adverse Possession, Conveyance, marketable title

Question 5 A Congressional committee was formed to conduct an investigation. The committee subpoenaed a man to appear before it to answer certain questions. The man appeared before the committee but refused to answer any questions. The committee notified the Speaker of the House of the man's refusal to cooperate. The Speaker called a special session of the House. At the special session, a majority of the members of the House voted to order the Attorney General of the United States to prosecute the man pursuant to a federal statute that establishes the penalties for contempt of Congress. Is the Attorney General constitutionally obligated to prosecute the man pursuant to the congressional order? A No, if the man is an appointive official of the executive branch, because he would then be immune from prosecution for acts performed in the course of his duties. B No, because the decision to prosecute is exclusively within the discretion of the executive branch. C Yes, because the Attorney General may not lawfully disobey a directive from Congress to punish a contempt. D Yes, because the Attorney General may not refuse to prosecute one who has violated federal law

B Congressional and Attorney General - Intermediate

3. An accountant borrowed $30,000 from a bank to help set up a small business. There was an acceleration clause in the loan agreement, which the bank could exercise any time after six months unless the accountant provided security for the loan. A few years later, the bank invoked the acceleration clause. The accountant did not have the cash on hand to repay the loan, so he offered the bank a $30,000 mortgage on property he owned. The bank accepted the mortgage and duly recorded it. There were no other mortgages on the property, which the accountant had inherited from his parents. The following year, a buyer offered the accountant $200,000 for the property, and the accountant promptly accepted. They entered into a written land sale contract, setting the closing date as May 24. The buyer contracted to tender the $200,000 on that date and the accountant contracted to convey "marketable title, free of encumbrances." On May 15, the buyer's title search in the county recorder's office revealed the mortgage on the property. The buyer immediately contacted the accountant, who said that he planned to use the money he obtained at closing to pay off the mortgage. The buyer found this unacceptable and failed to appear at the appointed time and place of closing, although the accountant was there on time with a deed of conveyance in hand. If the accountant files an appropriate suit against the buyer demanding specific performance and wins, what is the most likely reason? (A) A mortgage used to secure a debt does not constitute a legal encumbrance. (B) The vendor of real property need not have marketable title until the time of the closing. (C) The mortgage was unenforceable ab initio, because a preexisting debt is not adequate consideration. (D) A mortgage of the type described does not follow the land, so the buyer would have taken the land free of the mortgage even if the accountant did not pay it off.

B Conveyance, Marketable Title

Question 3 A teenager stole a car from the homeowner's driveway after she had left the keys in the car overnight. He was stopped for speeding and a license plate check indicated that the car had been reported stolen. The teenager was charged with common law larceny. At trial, his defense was that he intended to return the car to the owner's home the following morning before anyone realized it had been taken. The trial judge instructed the jury that while the state must prove the case beyond a reasonable doubt, the defendant has the responsibility to prove his defense by a preponderance of the evidence. The court further instructed the jury that if it found by a preponderance of the evidence that the teenager intended to return the car, it should find him not guilty. The teenager was convicted; he appealed on the ground that the jury instructions were erroneous. Should the teenager's conviction be reversed? A Yes, because in a criminal case the state must prove all disputed issues beyond a reasonable doubt. B Yes, because the instruction placed an improper burden of proof on the defendant. C No, because intent to return is not a defense to the charge. D No, because the jury instructions were correct

B Defendant Burden of Proof at Trial, Jury Instructions- Intermediate

Question 10 While fleeing from an armed robbery he had just committed, a man struck a pedestrian with his car, seriously injuring the pedestrian. The robber was soon apprehended and charged with armed robbery and reckless driving, both felonies dust prior to trial, the pedestrian died from his injuries. The trial on the robbery and driving charges proceeded, and the robber was convicted of the armed robbery charge and acquitted of the reckless driving charge. The robber was then indicted under the jurisdiction's felony murder statute for causing the death of the pedestrian during the course of committing an armed robbery. The robber moved to dismiss the indictment on the ground that a second trial would violate double jeopardy. Is the robber's claim correct? A Yes, because he was acquitted of the reckless driving charge. B Yes, because the pedestrian died before the robber's first trial had begun. C No, because he was convicted of the armed robbery charge. D No, because felony murder requires proof of an additional element not required by the felony itself

B Double Jeopardy - Advanced

Question 3 An officer went to an apartment to execute a properly obtained search warrant during an investigation of an operation making counterfeit watches. When he arrived he shouted, "Police, open up," but he did not wait before entering the apartment through the unlocked front door. The officer found the defendant half-asleep in a back room, with a workbench for assembling counterfeit watches nearby. Along with some completed counterfeit watches, the officer found a toolset used for making watches and receipts for various watch components in a of the workbench, all of which the officer seized as evidence. The defendant was charged with illegal counterfeiting. At a preliminary hearing, his attorney moves to suppress the evidence obtained during the search. Should the court grant his motion? A No, because regardless of whether the search warrant itself was valid, the evidence was in "plain view' upon entry. B No, because the exclusionary rule does not apply to the officer's Fourth Amendment violation. C Yes, because the officer failed to wait long enough prior to entering the apartment. D Yes, because an unlocked door is not an invitation to enter

B Exclusionary Rule & Knock and Announce Rule - Intermediate

Question 8 The beneficiaries of a trust filed suit to remove the trustee and recover damages, alleging that the trustee improvidently invested trust funds. At trial, the trustee calls a financial expert to testify in his defense. To qualify the witness as an expert, the trustee asks the witness about her education, training, and experience in finance. The witness testifies that she has received a BA and PhD in economics and business finance. On cross-examination concerning qualifications, the beneficiaries' attorney asks, "Isn't it a fact that you flunked out of college at the end of your freshman year?" to which the witness replies, "No." The beneficiaries' attorney then offers the registrar of the college that the witness attended to introduce a transcript of the college's records to show that the witness studied only one year and was awarded no degrees. The trustee objects. Is the transcript admissible? A Yes, but the judge should instruct the jury that they are to disregard the witness's testimony if they conclude that she is not an expert. B Yes, but the judge should allow the witness to testify further only if he believes the witness to be an expert. C No, because denying the witness's claim that she obtained a BA and PhD is impeachment on a collateral matter, which cannot be proved by extrinsic evidence. D No, because whether the witness was awarded a BA and PhD is irrelevant to the main issues

B Expert Qualifications - Advanced

Question 10 Several members of a small terrorist group are on trial in federal court for conspiring to bomb a military installation. The prosecution would like to introduce the testimony of a military guard at one of the installation's gates. The guard had been present when a bomb that was being planted by a member of the group had exploded prematurely. The guard will testify that she ran over to administer first aid to the member, who in great pain told her that his group was in the process of planting three other bombs in other areas of the military installation and was going to detonate them all at the same time to get publicity for their cause. The guard will also testify that the member disclosed the locations of the other bombs and the names of tm other members of the group. The authorities were able to prevent the other bombings and arrest the other members of the group. The member died from his injuries. What is the best basis for allowing the guard to testify as to the member's statements? A As a vicarious admission of a co-conspirator. B As a statement against interest. C As a statement of present state of mind. D As a dying declaration

B Hearsay, Statement Against Interest- Advanced

Question 6 A citizen of State A purchased life insurance by mail from a State B insurance company. The policy was the only one that the company had ever sold in State A. The purchaser mailed premiums from State A to State B for five years, and then died. The insurance company refused to pay the policy benefits. The purchaser's administrator sued the company in State A state court. The state has a long arm statute that grants a state court in personam jurisdiction over a defendant who "contract(s) to insure any person, property or risk located within this State at the time of the contracting!' The insurance company argued that its only contact with State A since it began its business was the purchaser's insurance policy, and that this single contact does not meet the minimum required for the exercise of in personam jurisdiction under International Shoe. How should the court rule on the minimum contacts issue? A For the purchaser's administrator, because the State A statute authorizes jurisdiction. B For the purchaser's administrator, because of the close connection between the contact and the case. C For the insurance company, because the exercise of jurisdiction would not be constitutional. D For the insurance company, because suit must be brought in State B.

B International Shoe Minimum Contacts - Advanced

Question 4 You are an in-house counsel for an insurance company. One of its life insurance policyholders has recently passed away and each of his three children claims the right to the proceeds from the policy. Which type of action would you bring on the insurance company's behalf? A An impleader action. B An interpleader action. C An action for an injunction. D A motion to dismiss.

B Interpleader -Intermediate

Question 4 A woman had a history of epileptic seizures which came without warning over the last five years. One day, as she was driving to town she had a seizure. She lost control of her car and struck a pedestrian who was lawfully crossing the street. The pedestrian died immediately thereafter as a result of his injuries. The woman is charged with involuntary manslaughter in a jurisdiction that follows common law principles. Will she most likely be found guilty? A Yes, because the law presumes that a person intends the natural and probable consequences of her acts. B Yes, because she knowingly and negligently chose to drive her car. C No, because her failure to control the car was not a voluntary act. D No, because she had no intent to harm anyone

B Involuntary Manslaughter- Seizures- Intermediate

Question 1 A civilian contract employee working for the United States Army was suspected of copying classified army documents onto a flash drive and selling the files to foreign governments. After a short investigation, the employee was arrested by military police. The employee was brought before a court martial, convicted of espionage, and sentenced by the court to 20 years' hard labor. The employee appeals his conviction and sentence on constitutional grounds. How should the court rule? A The sentence is unconstitutional because 20 years' hard labor is cruel and unusual punishment. B The conviction is unconstitutional because the court martial did not have jurisdiction to try the employee. C The sentence will be upheld because 20 years' hard labor is neither cruel nor unusual given the charges here. D The conviction will be upheld because the court had jurisdiction over the employee

B Judicial review - Intermediate

Question 8 A recent law school graduate was offered a job as an aide by a state legislator. The legislator told the graduate that before she could begin working, she had to take the following loyalty oath: "l swear to uphold our state and federal Constitutions; to show respect for the state and federal flags; and to oppose the overthrow of the government by violent, illegal, or unconstitutional means." The graduate told the legislator that the oath is unconstitutional and refused to take the oath. Is the graduate correct? A Yes, as to the promise to uphold the state and federal Constitutions. B Yes, as to the promise to respect the flag. C Yes, as to the promise to oppose the overthrow of the government. D No, as to all three provisions

B Loyalty to Constitution Oath for Employment - Advanced

Question 9 While at a party, the defendant ran into an acquaintance. The acquaintance proceeded to ridicule the defendant about his looks. After an hour of verbal abuse by the acquaintance, the defendant suddenly took a champagne bottle that was on a nearby table and struck the acquaintance over the head, killing him instantly. At his arrest, the defendant told the police that voices inside his head told him to shut the acquaintance up, permanently. The defendant was tried in a jurisdiction that follows the Model Penal Code test for insanity. At trial, the defendant's lawyer introduced psychiatric testimony indicating that the defendant suffered from a mental illness. Which of the following, if proved by the defense, would most likely relieve the defendant of criminal responsibility? A The defendant's actions were a product of his mental illness. B The defendant could not appreciate the criminality of killing the acquaintance, or he could not conform his conduct to the requirements of the law. C The defendant did not know that killing the acquaintance was wrong, or he could not understand the nature and quality of his actions. D The defendant was unable to control himself or conform his conduct to the law

B Model Penal Code Insanity - Intermediate

Question 1 After learning that she was the sole beneficiary in her uncle's will, a niece broke into his house while he was away one evening. She brought with her a vial of poison that she believed would be lethal but undetectable. She poured it into a glass of water that her uncle had placed on the bedside table. During the night, the uncle took a drink of water and the poison started to affect his ability to breathe. Thinking he was having a heart attack, the uncle started to go downstairs to call his doctor. On the way, he slipped and fell down the stairs, breaking his neck. He was found dead the next day. An autopsy revealed that the uncle had ingested poison, but not in a sufficient quantity to have caused death. Further investigation led the police to the niece, who was arrested and charged with burglary and murder. Based on the above facts, of what crime(s) should the niece be convicted? A Burglary only. B Burglary and murder. C Burglary and attempted murder. D Attempted burglary and attempted murder

B Murder and Burglary- Intermediate

Question 5 The state has the following homicide statutes: Murder is the unlawful killing of a human being with malice aforethought. Such malice may be express or implied. It is express when there is manifested a deliberate intention to unlawfully take away the life of a fellow creature. It is implied when no considerable provocation appears or when the circumstances attending the killing show an abandoned and malignant heart. All murder that is perpetrated by willful, deliberate, or premeditated killing or committed in the perpetration of or attempt to perpetrate arson, rape, robbery, or burglary is murder of the first degree All other kinds of murders are of the second degree. The defendant and her associate entered a jewelry store to shoplift a diamond bracelet dust as the defendant put the bracelet into her pocket, a sales clerk saw her and grabbed her by the wrist. The associate grabbed a knife from one of the silver displays and lunged at the sales clerk, but then a store guard shot and killed her. The defendant is charged with the first degree murder of her associate. Which of the following is the defendant's strongest argument? A The defendant cannot be convicted of murder because when they went into the store they were not carrying any weapons; therefore, there was no felony on which the felony murder rule may arise. B The defendant cannot be convicted of murder because the associate's death was not murder but justifiable homicide. C The defendant cannot be convicted of murder because she and her associate had an agreement never to use violence when they stole anything. D The associate did not intend to hurt the sales clerk, but just wanted to scare him so that the defendant could run

B Murder of Co-Criminal - Intermediate

Question 8 The police set up a sting operation targeting a resale shop that had a reputation for selling stolen goods. An undercover police officer approached the owner of the shop posing as a truck driver who was down on his luck and looking for a way to earn some extra money. The shop owner suggested that on his next load the officer should take a box of goods from the back of his truck, bring them to the shop owner to sell in exchange for cash, and then report the box lost to the trucking company. The next day the officer gave the shop owner an empty box in exchange for money. Immediately after the exchange, the shop owner was arrested. Which of the following crimes did the owner commit? A Receipt of stolen property. B Attempted receipt of stolen property. C Conspiracy and receipt of stolen property. D No crime

B Receipt of Stolen Property/Undercover Police- Advanced

Question 7 A defendant is on trial for allegedly burning down her business establishment because it was losing money. Before the defendant takes the stand in her defense, the prosecution seeks to introduce testimony from an insurance agent that the defendant purchased two insurance policies for the building within a month before the fire. Each policy had been purchased from a different insurance carrier and each policy was in the amount of the full value of the business. The defendant's attorney objects to the introduction of this testimony. How should the court rule on the admissibility of the testimony? A Admissible only for purposes of impeachment when the defendant takes the stand in her own defense. B Admissible as substantive evidence against the defendant. C Inadmissible, because the Federal Rules ban using evidence that a party carried insurance to prove that the party acted wrongfully. D Inadmissible, because the policies themselves are required to be introduced under the original document rule

B Relevance - Intermediate

Question 3 One night when a man was very drunk, he took one of his rifles, loaded it, and fired a bullet through his front door. Unbeknownst to him, at the time he fired the rifle, someone was driving by the house. The bullet went through the front door, through the window of the car, and killed the driver. The shooter was convicted of murder and appeals. He contends that there was insufficient evidence to support a finding of murder. How should the court of appeals rule? A That the evidence is sufficient to prove that the killing was intentional. B That the evidence is sufficient to prove that the killing was done with malice aforethought. C That the evidence is insufficient, because the shooter did not know that the driver was driving by his house and therefore he could not have acted intentionally. D That the evidence is insufficient, because at most the shooter's conduct constituted gross negligence and involuntary manslaughter

B Second Degree Murder - Intermediate

Question 1 A state statute requires that a person who is suspected of committing a crime must be informed of the nature of that crime before questioning may begin. The state supreme court has held that statements obtained in violation of a suspect's statutory rights may not be admitted into evidence. The defendant, who was arrested on suspicion of committing arson, was told, "You have the right to remain silent; anything you say can and will be used against you in a court of law; you have the right to the presence of an attorney during questioning; and if you cannot afford an attorney, one will be appointed for you." In response to questioning, the defendant gave a statement implicating himself in the arson. He was charged and brought to trial in state court for arson. At trial, should the statement be excluded from evidence? A Yes, because the Miranda warnings were not proper. B Yes, because the questioning violated state law. C No, because proper Miranda warnings were given in compliance with federal constitutional requirements. D No, because the requirement of informing the suspect of the nature of the charges against him is not a state constitutional requirement

B Self-Incrimination - Intermediate

Question 7 A man and a woman were arrested and charged with a series of armed robberies. Each suspect was given Miranda warnings, and different interrogation teams questioned each suspect separately upon being questioned, the man told the police, "I'm not going to talk until see a lawyer." An officer responded, "You might want to reconsider, because your partner has already confessed, and she's implicated you in the crimes." The man then told the police that he wanted to talk to the woman privately. The police escorted the man to the woman's cell, locked him in with her, and left. Unbeknownst to either of them, the police had bugged the woman's cell and recorded both the man and the woman making self-incriminating statements during their meeting. The man made no further statements to the police on advice of counsel whom he called immediately after his conversation with the woman. The man was put on trial first, and the prosecution sought to introduce into evidence tapes of the bugged conversation between the man and the woman. The defense made a motion to suppress the evidence. Should the court grant the motion to suppress? A Yes, because the evidence is the fruit of a wiretap that violated the Fourth Amendment. B Yes, because the police created a situation likely to induce the defendant to make an incriminating statement. C No, because there is no expectation of privacy in a jail cell. D No, because the conversation constituted a waiver of the man's Miranda rights

B Sixth Amendment Right to Counsel -Advanced

Question 10 A boy and his parents sued a driver for $75,000 for injuries they claim were caused when the driver's car hit the boy one night when the boy was out delivering papers. The boy was knocked unconscious in the accident, and the driver claims that it was not his car that hit the boy. Except for damages, the main issue in the suit is whether it was the driver's car that hit the boy. The driver's own attorney asks him, "Could the boy have mistaken your car for another?" Is this question objectionable? A Yes, because the answer would be hearsay. B Yes, because the answer would be an opinion. C No, because the answer would be relevant to the issue of whose car hit the boy. D No, if a proper foundation has been laid

B Speculation- Advanced

Question 1 A defendant was visiting with his girlfriend in his apartment when a visitor came to see him. The defendant and the visitor engaged in a conversation relating to the distribution of illegal narcotics in the girlfriend's presence. Two months later, the defendant and his girlfriend married. Subsequent to the marriage, the defendant was arrested and charged under federal law with the sale and distribution of drugs. The prosecutor wants the defendant's wife to testify about the conversation between the defendant and the visitor, but the defendant forbids it. May the defendant's wife testify about the conversation? A Yes, because the conversation occurred prior to their marriage. B Yes, but only if she chooses to do so. C No, because the defendant forbids it. D No, unless both the defendant and his wife agree that she may testify.

B Spousal Immunity - Intermediate

Question 6 A comprehensive federal health-care reform statute created a Federal Health Policy Board, which was directed to monitor the fees charged for various medical procedures covered by insurance. The board also had the power to subpoena records to determine whether fee increases were a true reflection of cost increases. Nothing in the statute provided for caps on fee increases. Because of the continuing escalation of health-care costs while the statute was being debated, several states had passed health-care legislation on their own. One state passed legislation that prohibited most fee increases of 10% or more per year for specified health-care services covered by insurance, and created a health-care review board to regulate these costs and impose monetary penalties on health-care providers or insurers that tried to circumvent the cap. Which of the following would be the best basis for finding the state provision unconstitutional? A The federal legislation was passed after the state legislation and therefore supersedes it. B The Federal Health Policy Board was constituted with many of the same powers as the state board but was not given the power to impose sanctions. C The state provision impairs existing contracts between health-care providers and insurers in violation of the Contract Clause. D Health-care fee caps create an undue burden on interstate commerce even in the absence of federal regulation

B Supremacy Clause/Preemption - Advanced

Question 7 A state study indicated that an inordinately high percentage of homeless in the state were afflicted by alcoholism or addiction to illegal drugs. The legislature therefore decided to levy a special tax, with all proceeds marked for rehabilitative services for the homeless. However, the legislators determined that direct taxes on alcoholic beverages would be resented by the citizenry. Lobbyists from the state's growing wine industry also objected to anything that would retard the industry's development. There were no breweries or distilleries within the state. Thus, a tax was eventually passed requiring newspapers and magazines of general circulation published in the state to be taxed at a rate of 20% on all advertising space sold for beer or distilled spirits promotions. For certain historical reasons, a high proportion of the advertising revenue of a particular small newspaper within the state came from beer and wine ads. The publisher of the small paper filed suit to have the tax declared unconstitutional. A major wholesale beer and liquor distributor located within the state and several out- of-state and distillers who sold and advertised their products in the state also joined in the suit as plaintiffs. If the tax is declared unconstitutional, what is the most likely reason? A The tax burdens interstate commerce by exempting advertisements for the local wine industry from the tax, while the ads of out-of-state brewers and distillers are subject to the tax. B The tax infringes on freedom of the press, which is guaranteed by the First and Fourteenth Amendments. C The tax is unconstitutional because it is not properly apportioned. D The tax violates the Equal Protection Clause of the Fourteenth Amendment, because it does not treat all alcoholic products equally.

B Tax on Newspaper Ads for Wine and Alcohol - Advanced

Question 5 A police officer learned from a reliable informant that a major drug deal was about to take place at a local restaurant. The officer obtained a search warrant for the restaurant and arrived with other uniformed officers to search the premises. While conducting the search, the officer searched several of the customers. While searching one of the restaurant's regular customers, the officer felt an object in the customer's pocket and pulled out a container filled with heroin. The customer was arrested and later convicted of possession of heroin. A state statute permits officers executing a search warrant to search persons on the premises if the officers reasonably expect danger to themselves or a risk of disposal or concealment of anything described in the warrant. If the customer challenges his conviction on the ground that his Fourth Amendment rights were violated, will he be successful? A Yes, because the statute is vague and overbroad. B Yes, because his presence in the place to be searched by the police does not negate the requirement of probable cause. C No, because the search was conducted pursuant to a valid search warrant. D No, because the search was authorized by statute

B Warrant and Probable Cause for Others on Premise - Intermediate

Question 7 A housing development contained one-, two-, and three-bedroom units. All units were suitable for occupancy, and the developers of the project filed the appropriate documents, including a Declaration of Restrictions that limited ownership and occupancy of the units to families or to groups of unrelated adults of not more than three in number. Each deed to the individual units also contained the restriction. One of the two-bedroom units was purchased by a woman and her boyfriend. They immediately moved into the unit with another unmarried couple who were friends of theirs. Other unit owners brought suit against the woman and her boyfriend to enjoin the occupancy by the other couple. If the other unit owners prevail, what will likely be the reason? A The litigants are private parties and the restriction was not enacted by the government. B Enforcement of the restriction is rationally related to a legitimate government interest. C Notice was not given by the woman and her boyfriend to the sellers of the unit that they intended to occupy the residence with another couple. D The restriction constitutes a lawful restraint on alienation

B Zoning Regulations - Advanced

8. A buyer entered into a written contract with a seller to purchase his commercial property for $100,000. The contract did not specify the quality of title to be conveyed, and made no mention of easements or reservations. The closing was set for November 25, three months from the signing of the contract. Shortly thereafter, the buyer obtained a survey of the property, which revealed that the city had an easement for the public sidewalk that ran in front of the store. Because this actually enhanced the value of the property, the buyer did not mention it to the seller. Subsequently, the buyer found a better location for her business. On November 1, the buyer notified the seller that she no longer intended to purchase the property. The seller told her that he intended to hold her to her contract. At closing, the buyer refused to tender the purchase price, claiming that the seller's title is unmarketable and citing the sidewalk easement as proof of that fact. In a suit for specific performance, will the seller likely prevail? (A) Yes, because the contract did not specify the quality of title to be conveyed. (B) Yes, because the buyer was aware of the visible easement and it enhanced the value of the property. (C) No, because an easement not provided for in the contract renders title unmarketable. (D) No, because the buyer gave the seller sufficient notice of her change in plans and yet he made no effort to try to find another purchaser.

B beneficial easement exception to the marketable title rule

6. A merchant had a serious cash flow problem and needed cash to buy the inventory required to fill orders. A friend offered to loan the merchant$ 50,000 if he would put up adequate collateral to assure her that she would not lose her money. To guarantee the loan, the merchant gave the friend a deed to a property worth $100,000 that he had inherited. The friend recorded the deed. The friend gave the merchant $50,000, and the merchant signed a promissory note agreeing to repay the $50,000 within eight months. The merchant continued to occupy the property, and the friend agreed to reconvey the property to the merchant as soon as he repaid the $50,000. The merchant was unable to pay the friend when the note came due. He asked the friend for an extension, but she refused. If the friend seeks to take possession of the property, may she do so? (A) Yes, because she has clear record title. (B) Yes, but only if she institutes foreclosure proceedings and is the successful purchaser at the foreclosure sale. (C) No, because $50,000 does not reflect the fair market value of the property. (D) No, the friend is limited to a contract claim against the merchant because the law does not recognize this type of security agreement.

B, Equitable mortgage

4. The owner in fee simple of a tract of land sold it to a farmer for$850,000. To finance the purchase, the farmer obtained a mortgage loan from a financing company for $600,000. The deed from the owner to the farmer was promptly and properly recorded, but due to an oversight the mortgage from the financing company was not immediately recorded. A few months later, the farmer approached the financing company about getting a second mortgage. The financing company turned him down, so he contacted a bank. Not having knowledge of the previous mortgage on the property, the bank agreed to loan the farmer $300,000 secured by a mortgage on the land, which it promptly and properly recorded. One day later, the financing company, having discovered that its original mortgage had not been recorded, properly recorded it. The jurisdiction's recording statute provides: "Any conveyance or mortgage of an interest in land, other than a lease for less than a year, shall not be valid against any subsequent purchaser for value, without notice thereof, whose conveyance is first recorded." The farmer struggled to keep up with his mortgage payments, and finally stopped making payments altogether on both mortgages. The bank began foreclosure proceedings, but did not include the financing company as a party. At the foreclosure sale, a buyer purchased the land, having no actual knowledge of the mortgage with the financing company. Soon after, the financing company declared its loan in default and sought to foreclose on the land. May the financing company foreclose against the buyer? (A) Yes, because the holder of a senior mortgage interest is unaffected by foreclosure of a junior interest. (B) Yes, because the holder of a junior mortgage interest is a necessary party that must be included in a foreclosure proceeding by the holder of a senior interest. (C) No, because its failure to promptly record extinguished its rights against all parties except the farmer, the original mortgagor. (D) No, because the buyer succeeds to the farmer's right of redemption.

B, Foreclosure, necessary parties

3. A chef purchased a restaurant for $100,000. As part of his financing, he obtained a purchase money mortgage from a bank for $60,000. Due to a clerical error by the bank, the mortgage was not recorded in the county recorder's office. A statute in the jurisdiction provides: "No conveyance of an interest in land, other than a lease for less than one year, shall be valid against any subsequent purchaser for value, without notice thereof, whose conveyance is first recorded." After the chef's restaurant had been in operation for five years, business dropped dramatically. To stay in business, the chef obtained a mortgage from a financing company for $30,000. The financing company was not informed by the chef of the mortgage held by the bank. The next day, the chef contacted the bank about renegotiating its mortgage. Checking its records, the bank discovered that the original mortgage was not recorded and immediately recorded it. Later that day, the financing company recorded its mortgage. A few days later, the chef and the bank agreed to a modification of their mortgage agreement to allow the chef to make lower monthly payments in exchange for a higher interest rate and a longer period of repayment. Despite this agreement, the chef was unable to make payments on the financing company mortgage. The financing company instituted a foreclosure action six months later, but failed to include the bank as a party to the foreclosure action. If the financing company takes title to the restaurant at the foreclosure sale, which of the following statements most correctly describes the bank's interest? (A) The bank's mortgage on the restaurant survives under its original terms. (B) The bank's mortgage on the restaurant survives under its modified terms. (C) The bank's mortgage is extinguished because when it was modified it became junior to the financing company's mortgage. The bank's mortgage is extinguished regardless of the modification because it had not recorded before the financing company obtained its mortgage interest

B, Foreclosure, purchase money mortgage, effect of modification on priority

8. Although the fair market value of each horse was $3,000, the horse breeder agreed to sell both horses together for a total price of $5,000. Under the agreement that the rancher wrote out and both parties signed, the horse breeder agreed to deliver one horse to the rancher on August 1, at which time the rancher agreed to pay the horse breeder $5,000. The horse breeder further agreed to deliver the other horse to the rancher's fiancée on August 12. On August 1, the horse breeder delivered the first horse to the rancher and, at the same time, the rancher gave the horse breeder a certified check for $5,000. On August 12, the horse breeder brought the second horse to the residence of the rancher's fiancée and told her that the horse was a gift from the rancher. The rancher's fiancée told the horse breeder that she loathed quarter horses and she refused to take the horse. The horse breeder brought this horse back to his farm and sent an e-mail to the rancher, informing him that his fiancée refused delivery and that he (the horse breeder) could not keep the horse. Two weeks later, after not hearing from the rancher, the horse breeder sold the horse to an interested party for $3,000. If the rancher sues the horse breeder, how much should the rancher recover? (A) $3,000, the value of the second horse. (B) $2,000, the difference between the value of the horse delivered to the rancher and what the horse breeder received from the rancher. (C) Nothing, because the rancher was not financially harmed. (D) Nothing, because the horse breeder performed his part of the contract.

B- Unjust Enrichment & Restitution

Question 2 A hiker in an isolated area encountered a cross-country skier who had broken her leg. The hiker created a makeshift sled and began pulling the skier to the nearest road. As the hiker was pulling her across the ice of a lake, the ice gave way and they went into the water. The hiker was unable to get out of the water and drowned. The skier was able to pull herself to shore and eventually was rescued. However, she suffered severe hypothermia and lost some of her toes to frostbite as a result of being in the water. Does the skier have a cause of action for damages against the Hiker's estate? A No, because the hiker had no duty to come to the skier's aid. B No, because the hiker did not survive the accident. C No, unless the hiker acted negligently in attempting to cross the ice. D No, unless the hiker acted with gross negligence in his attempt to cross the ice.

C (Duty to Rescue)

Question 1 An only daughter whose father died after a long illness arranged with a mortuary to bury him next to his wife's grave. The daughter selected the most ornate casket available because she wanted it to remain closed during the wake, and she could not bring herself to view the body before the funeral. At the cemetery, however, she decided to view the body just before it was buried. She was horrified to discover that the body in the casket was dressed in a clown costume and a bright orange wig. In fact, it was not her father but a popular circus entertainer who had died the same day as her father and had requested to be buried in his costume. Although the mortuary was able to retrieve her father's body and bury it, the daughter was greatly distressed by the episode and suffered nightmares as a result. However, she did not seek medical or psychiatric care because of it. The mortuary apologized for its error in switching the bodies, but insisted that the daughter pay all of the agreed-to charges for the funeral. If the daughter brings action against the mortuary to recover for her emotional distress, can she recover damages? A No, because the daughter did not have to obtain medical or psychiatric care. B No, because the daughter suffered no physical injury. C Yes, because of the known sensitivity of people concerning the death of a family member. D Yes, because the mortuary is requiring the daughter to pay the bill for the funeral expenses

C (NIED)

Question 3 A woman was driving carefully but with an expired driver's license, in violation of a statute requiring license renewal. When she stopped at a stop sign, another driver, who was speeding, crashed into her car. The woman suffered injuries and sued the other driver. What is the best reason why the fact that the woman had an expired driver's license will not affect her claim against the other driver? A The other driver's negligence occurred after the man's. B The driver should have known that there are some unlicensed drivers on the road. C The prevention of accidents of this sort is not the reason that drivers are required to renew their driver's licenses. D There is a greater chance of causing injury when a driver speeds than when a person drives with an expired license

C (Negligence Per Se)

Question 8 A dog whistle manufacturer's factory was located near a residential area. The manufacturer used the most effective methods for testing its whistles, but it was impossible to completely soundproof the testing area. A breeder of champion show dogs bought some property near the factory and raised and trained her dogs there. Although the whistles were too high-pitched to be perceived by human ears, they could be heard by the breeder's dogs. Consequently, the dogs often were in a constant state of agitation. In a suit by the breeder against the manufacturer, what is the likely outcome? A The breeder will prevail on a trespass theory, because the sound waves are entering onto the breeder's property. B The breeder will prevail on a nuisance theory, because the sound of the whistles is a substantial interference with the breeders use of her land. C The breeder will not prevail, because the sound of the whistles is not a substantial interference with the breeder's use of her land. D The breeder will not prevail, because the manufacturer has acted reasonably in testing its whistles

C (Nuisance)

Question 6 A child was injured when he was forcibly pulling on the safety harness in his highchair and the harness suddenly broke and snapped up, hitting him in his face. The highchair was manufactured overseas by a toy manufacturer and exported to the United States without the harness. When the highchair arrived in the United States, a company that makes safety equipment installed the harness on the highchair. The toy manufacturer had its factory representatives go to the safety company to inspect samples of the harness and conduct tests on a sample number of the highchairs after installation. The child's parents brought a negligence action against both the toy manufacturer and the safety company for his injuries. Evidence at trial shows that the toy manufacturer had conducted its normal tests with regard to the installations. The particular harness installed on the highchair, however, had some loose weaving that weakened the harness to the extent that it snapped from the child's continual pulling. Which of the following is the best defense the toy manufacturer could assert against the parents? A The safety company had guaranteed the reliability of its product, and, therefore, the toy manufacturer was under no obligation to inspect. B The harness was not designed to withstand the type of stress it suffered from the child's actions. C The toy manufacturer's inspection procedures were reasonable under the circumstances. D The safety company manufactured the harness, and, therefore, it must be held solely liable for any defects

C (Products Liability - Negligence)

Question 1 The governor of an arid Western state owned a vacation home and permitted his son to have a party there. At the end of the night the son failed to properly extinguish a bonfire that he and his friends had built, and within a few hours, wind-blown cinders had spread the fire to the trees east of the lodge. At the same time several miles away, a worker at a lumber mill was making emergency repairs to a pipe running between two mill buildings. He did not notice some of the sparks from his welding torch land in a pile of dried lumber and catch fire, and he failed to check the area after he was finished. By the time the fire was noticed by another employee, it was out of control. The wind blew both fires toward a landowner's hunting lodge. They merged a mile away and shortly thereafter totally consumed the lodge. For political reasons, the landowner did not bring a lawsuit against the governor or his son. He did, however, file a lawsuit against the lumber mill, alleging that its employee's negligence caused the destruction of his lodge. Evidence at trial established that either fire alone would have destroyed the lodge as well. Can the landowner recover from the lumber mill? A No, because the landowner's lodge would have been destroyed regardless of the conduct of the lumber mill's employee. B No, because the damage is indivisible and cannot be apportioned unless the landowner adds the other tortfeasor to the lawsuit. C Yes, because the negligence of the lumber mill's employee was a cause of the landowner's injury. D Yes, but the landowner can recover only 50% of his damages from the lumber mill

C (Substantial Factor Causation)

Question 9 A missile company was engaged in research and development of an interplanetary space shuttle, under contract with the United States government. Over a period of years, it developed the prototype of a huge, solid-fuel rocket engine for use in this program. To evaluate the performance of this engine, it conducted a static test of the engine at a remote desert test site. The rocket engine was mounted on a concrete test stand, with the thrust of the engine directed downward into the ground. When the engine was fired up, huge clouds of flame and smoke filled the air, and particles of debris from the rocket fell onto an adjoining farm. If the farmer files an action against the company for trespass, which of the following facts, if proved, would be most helpful to the company in avoiding liability? A. The farmer bought and operated his farm knowing that the company used the adjoining property for testing its rocket engines. B. Neither the company nor anyone in its employ set foot upon the farmer's land. C. The company had no reason to anticipate that the tests would cause any of the results that occurred. D. The rocket testing program is essential to national security, so that the company's conduct was completely privileged as a public necessity

C (Trespass)

Question 5 A law enforcement officer was transporting a prisoner on a plane to testify in a criminal case. Unknown to those on the plane, an assassin hired to kill the prisoner had bribed an airport baggage handler to sneak a timed-release crate of poisonous snakes into the cargo hold of the plane. Once the crate was triggered to open, the snakes were able to slither into the passenger compartment through gaps in the conduits between the cargo hold and the passenger compartment. In the ensuing panic caused by the snakes, the officer was struck in the head by a fire extinguisher that another passenger threw at a snake, and suffered a severe concussion. The officer filed suit against numerous parties, including the person who designed the conduit system on that type of plane. At trial, evidence established that the design for the conduit system that he used had been rejected in the industry because of the danger of pressure loss between the cargo hold and the passenger compartment. An industry-approved design that the designer could have used would have kept the snakes from getting into the passenger compartment of the plane. As between the officer and the designer, which party is likely to prevail? A The officer, because the designer is strictly liable for designing the conduit system of the plane. B The officer, because of the high degree of care owed to passengers of a common carrier. C The designer, because the assassin's actions were an unforeseeable intervening force. D The designer, because the officer was injured by another passenger rather than a snake

C (Unforeseeable Intervening Force Causation)

7. An investor owned two adjacent lots in a downtown area, one fronting directly on a public street and the other behind the first. The investor ran a small dry cleaning business on the lot next to the street, and had built a café on the rear lot. Because the rear lot had no access to any public street, the investor used the parking lot of the dry cleaning business, which extended from the street all the way back to the rear lot, for access to the café. The café was only open during the tourist season, from May through September. After several years, the investor sold the rear lot to a chef by a deed that granted an easement over the dry cleaning business's parking lot, to be used as an accessway to the café for the chef and her customers. The chef promptly recorded the deed. Two years later, in February, the investor sold the dry cleaning lot to a sub sandwich franchise. The deed did not mention the easement previously granted to the chef. The franchise immediately demolished the small dry cleaning building and constructed its own restaurant. An outdoor patio area completely blocked access to the chefs rear lot. Because the chefs café was closed for the winter, nothing came of the franchise's construction until April, when the chef returned to open her café for the summer season. The shortest alternate route over other parcels from the chef's lot to a public street would have to pass through several buildings and lots. If the chef brings an action to compel the sub sandwich franchise to demolish the outdoor patio, how should the court rule? (A) For the franchise, because its deed contained no mention of the access easement. (B) For the franchise, because construction of the outdoor patio extinguished the chefs rights to the access easement. (C) For the chef, because ownership of the easement gives her the right to use it for access to her lot. (D) For the chef, because she has no other access to her lot.

C - Easements

10. A racehorse breeder and a stable owner entered into a written agreement giving the stable owner the option to purchase all colts foaled out of a particular champion racehorse during the next three years. The agreement provided that price would be determined on the basis of weight, height, and bone structure at time of delivery. Six months later, the first colt was born to the horse, and it had all the markings of a champion. The stable owner immediately tendered $25,000 to the breeder for the colt, which was a good faith approximation of its value. However, the breeder refused to deliver the colt unless the stable owner paid $100,000. The stable owner sued the breeder. If the breeder defends on the ground that there is no enforceable contract obligating him to sell, what would the court most likely hold? (A) There is no enforceable contract because the stable owner was not obligated in any way under the signed writing. (B) There is an enforceable contract because, by signing the document, the stable owner impliedly promised to purchase. (C) Even if the writing was not an enforceable contract, the stable owner's good faith tender of $25,000 created an enforceable contract. (D) The agreement is enforceable as a firm offer between merchants under the UCC.

C - Mutual Assent; Offer and Acceptance; Termination by Offer, Revocation; Limitations on Offeror's Power to Revoke; Consideration;

2. On March 5, a seller who regularly deals in the sale of drill bits mailed a signed communication to the buyer for a chain of hardware stores, offering to sell to him 500 diamond core drill bits, priced at $300 each. The communication explicitly stated that it was a firm offer. The buyer received the offer on March 7. On March 8, the seller mailed a notice to the buyer revoking his March 5 offer. The buyer received this notice on March 10. On March 11, the buyer e-mailed the seller, stating that he considered the seller to be bound by his offer. The buyer also placed on order "per your offer" for 500 diamond core drill bits. The seller refused to tender delivery of the drill bits, and the buyer sues the seller for breach of contract. The seller defends on the ground that there was no contract because it had revoked its offer. Given these facts, how would the court likely rule? (A) No contract exists because the seller's notice of revocation had been received by the buyer before acceptance was attempted. (B) Having stated that it was extending a "firm offer," the seller's offer was irrevocable for three months as a matter of law. (C) The buyer has an enforceable contract with the seller. (D) Having received no consideration for its declaration that it had made a firm offer, the seller retained an unfettered election to revoke its offer at any time.

C - Mutual Assent; Offer and Acceptance; Termination by Offeror, Revocation; Limitations on Offeror's Power to Revoke

5. A salvage company offered for sale Confederate dollars that had been recovered when the company recently raised a shipwreck off the coast of South Carolina. A purchasing agent for a private west coast museum purchased the bills, but he had represented that he was buying them for himself in hopes of obtaining a lower price. After purchasing the bills, the agent carefully packaged them and had them shipped to his museum. While the bills were in transit, the museum burned to the ground and its owner decided that she would not rebuild because most of her collections had been destroyed. When the bills arrived after the fire, she opened the package only to discover that the bills were too brittle for shipping by this method - three bills had disintegrated in transit. Undaunted, the owner took the remaining nine bills and had them mounted behind a glass frame so she could display them in her study. While the bills were being framed, she read on the Internet that a large cache of similar bills had just been discovered, and the market price for such bills had just been cut in half. Frustrated but still undaunted, she hung the framed bills in her study. Unfortunately, the salt water had reacted with the pigments in the bills in such a way that shortly after they had been exposed to indirect sunlight, all of the color in the bills faded almost completely away. No other Confederate bills raised from the ocean before had similar reactions; these bills appear to have been printed using substandard dyes. Which of the following facts would give the museum the best basis for rescinding the contract with the salvage company? (A) The bills were too brittle for transport. (B) The discovery of a large cache of similar bills a few days after the sale. (C) The bills' unusual reaction to indirect sunlight. (D) The destruction of the museum before the bills arrived.

C - Mutual Mistake

4. A retailer entered into an oral contract with an office supply wholesaler to buy 100 file boxes for an upcoming back to school sale at the retailer's store. The wholesaler agreed to deliver the file boxes in two weeks at a cost of $4 per file box. A week later, the retailer phoned the wholesaler and asked if she could increase her order to 200 file boxes. The wholesaler agreed. The wholesaler delivered the 200 file boxes as promised, but the retailer accepted only 150 upon discovering that she lacked storage space for all 200. May the wholesaler recover damages with respect to the 50 file boxes that were not accepted? (A) Yes, because the retailer accepted $600 worth of file boxes. (B) Yes, because the modification was for less than $500. (C) No, because the contract as modified was for $800. (D) No, because the wholesaler is a merchant with respect to file boxes.

C - Statute of Frauds; Modification, Goods Priced at $500 or more;

1. A grandfather told his granddaughter that she could have his house because he was moving to a retirement home, and entered into a valid contract to convey it to her. He promised her that he would have another wing added to the house in the back before turning it over to her, and entered into a written contract with a builder to construct the addition for his granddaughter. Before the grandfather had entered into the contract with the builder, the granddaughter had paid $5,000 for a 60-day option to purchase another house because she was not sure she would like the addition. However, when her grandfather showed her the plans for his house prepared by the builder, she liked it very much and decided to let her option to purchase the other house lapse. Shortly thereafter, the local zoning authority increased the minimum lot line setbacks, making it impracticable to put the addition on the back of the house. The builder offered to put an addition above the existing floor rather than in the back, and the grandfather agreed. After the granddaughter's option had lapsed, she discovered that the addition was now going up rather than in the back. She angrily demanded that the builder either build the addition according to the original specifications that she approved or pay her damages. The builder refused and the granddaughter filed suit. Who is more likely to prevail? (A) The granddaughter, because she was an intended beneficiary of the contract whose rights had vested. (B) The granddaughter, because the subsequent agreement between her grandfather and the builder to modify the construction was unsupported by consideration. (C) The builder, because he may raise all defenses that he had against the grandfather against the granddaughter. (D) The builder, because the granddaughter is merely an incidental beneficiary of the contract between the grandfather and the builder and, as such, has no power to enforce the contract against the builder.

C - Third Party Beneficiary

3. An art gallery owner called one of her best customers to inform him that she had just acquired a painting by the collector's favorite artist and that she would sell it to him for $1,000. The collector, who had purchased over 100 paintings from this gallery owner, came down to the gallery that afternoon to see the painting. The collector stated that he was hesitant to purchase it because he wasn't sure the painting, which was a departure from the artist's usual style, would fit with the rest of his collection. Although the custom in the art sale trade is to prohibit returns other than for damage, the gallery had permitted the collector to change his mind and return several pieces over the years, no questions asked. With that in mind, the collector decided to go ahead and sign the contract for purchase, agreeing to pay $1,000 upon delivery of the painting. The contract itself did not mention the possibility of sending the painting back. Two days later, when the painting was delivered to him, the collector, having decided that the piece did not fit into his collection, refused to accept it. The gallery brings an action for breach. Is the gallery owner likely to prevail? (A) Yes, because the language of the written, signed contract controls. (B) Yes, because the custom of the trade controls. (C) No, because the parties' course of dealing will affect the terms of the contract. (D) No, because the gallery knew the collector was unsure about the purchase.

C - UCC Parol Evidence/Interpretation; Additional Terms Under Article 2

5. A landowner developed and built a large apartment house on property he owned. After the apartment house was completed, the landowner conveyed the building and the land to his son for life, remainder to his granddaughter; subject, however, to a bank's mortgage. The bank held a mortgage on the building for $17,000, payable in annual installments of $2,000 plus interest. The son began to manage the day-to-day maintenance of the building as well as to collect rents. Despite the son's best efforts, the vacancy factor reached an unanticipated high and the rental income was not sufficient to cover the annual mortgage payment. Who is responsible for the mortgage payments and in what amount? (A) The son pays all. (B) The granddaughter pays all. (C) The son pays the interest and the granddaughter pays the principal. (D) The granddaughter pays the interest and the son pays the principal.

C - Waste

3. A property owner owned a tract of commercial property that he conveyed in joint tenancy to his twin sons as a birthday present. Unfortunately, a few years after the conveyance, the property owner and his sons had a serious falling out over how to run the family business. The property owner no longer wished the sons to control valuable commercial property, and so he demanded that they return the deed with which he conveyed the property to them. The sons returned the deed, and the property owner destroyed it. A few months later, one of the twins learned that he was seriously ill and not likely to live much longer. He executed a quitclaim deed conveying "any interest I have in the commercial property conveyed to me and my brother from my father" to his daughter. The twin who conveyed the property subsequently died. Who owns the property? (A) The living twin. (B) The property owner. (C) The living twin and the deceased twin's daughter as tenants in common. (D) The living twin and the deceased twin's daughter as joint tenants.

C -Concurrent estates (joint tenancy)

6. A grantor executed a valid deed conveying a tract of land to a city for "the purpose of constructing a planetarium thereon." The city held the property for a number of years, but decided on another site for the planetarium. When presented an offer to purchase the property by a privately owned garbage collection company, the city accepted and conveyed the land to the company. Which of the following statements about the title of the tract of land is true? (A) The grantor's conveyance to the city created a fee simple determinable in the city and a possibility of reverter in the grantor. (B) Upon conveyance of the land to the company, the property reverted back to the grantor. (C) The company owns the land in fee simple absolute. (D) The company owns the land, but it will revert to the grantor or his successors in interest if the property is used for anything other than a planetarium.

C -Present Estate

2. A landowner owned a tract of land, and his brother owned the adjoining property. The landowner drew up an instrument that stated: "Upon my death, I wish the land to pass to my wife; but should she or her heirs ever attempt to sell the land, the right of first refusal is hereby granted to my brother, his heirs and assigns." The brother drew up a similar instrument granting the landowner the same rights over the adjoining property. Both instruments were executed with all proper formalities. The landowner died, and the wife enjoyed possession of the land. Subsequently, she offered to sell the land to a buyer for $200,000. The brother learned of this and demanded that he be allowed to exercise his option to purchase the land. The wife refused. The brother filed suit to compel the wife to sell him the land. Assuming that the state in which the property is located strictly follows the common law Rule Against Perpetuities and the common law rule against restraints on alienation, will the brother prevail? (A) Yes, because his arrangement with the landowner was reciprocal. (B) Yes, because he has the equivalent of a possibility of reverter in the land. (C) No, because the right of first refusal violates the Rule Against Perpetuities. (D) No, because a right of first refusal is an improper restraint on alienation.

C -Rule Against Perpetuities

Question 2 A locksmith knew that his friend had been having marital troubles. The friend had told the locksmith that he suspected his wife was having an affair with his rival. One afternoon, the friend, visibly upset, asked to borrow some of the locksmith's tools, telling him that he knew that his rival was going to meet up with his wife later that day. The locksmith gave his friend the tools, advising him not to do anything that he would regret later. The friend stated that it would be others who would have regrets. The friend went to his rival's apartment and picked the door lock with the locksmith's tools. He found his wife and rival in bed together. The friend stabbed his rival, seriously wounding him. A few minutes later the locksmith called the apartment to try to warn the rival that his friend might come over. After the friend was arrested, he agreed to plead guilty to aggravated battery and attempted voluntary manslaughter in exchange for testifying against the locksmith, who was charged as an accomplice to attempted murder. Can the locksmith be convicted of that charge? A Yes, because he recklessly disregarded a substantial risk to human life and was not provoked. B Yes, because his failed attempt to neutralize his assistance did not prevent the crime from occurring and therefore did not constitute an adequate withdrawal. C No, because he did not have the requisite intent to be liable as an accomplice. D No, because an accomplice cannot be found guilty of a more serious offense than that for which the principal has been convicted

C Accomplice - Intermediate

1. Fifty-one years ago, an owner conveyed land to a taker for "so long as the land is used solely for residential purposes; otherwise, the interest in land shall revert to the owner and his heirs." The taker used the land as her personal residence for 20 years, but 31 years ago, she began operating a children's day camp on the land. The owner knew of this operation, but he took no action. Two years ago, the aged taker decided to get out of the camp business. She closed her business and once again began to use the land solely as her personal residence. Also two years ago, the owner died, survived by his son and only heir. Now the son is laying claim to the conveyed land. The jurisdiction in which the land is located has a seven-year adverse possession statute and another statute that bars enforcement of possibilities of reverter 55 years after their creation. May the son validly claim title to the land? (A) Yes, because less than 55 years have elapsed since the creation of the possibility of reverter. (B) Yes, because the adverse possession period began to run when the taker returned the property to residential status, and the taker has not held for the requisite seven years. (C) No, because the adverse possession period began 31 years ago, and the taker has held the property for more than the requisite seven years. (D) No, because the owner did not assert his possibility of reverter; thus, no cause of action arose in the owner or his heirs.

C Adverse Possession (difficult)

Question 6 A homeowner was trimming his sidewalk-bordering hedge when a wasp began attacking him. The homeowner attempted to hit the wasp. During one of his swats, the homeowner struck a jogger in the face. The jogger, reacting to the unexpected blow to his head, reached into his pocket and pulled out a knife. The homeowner tried to shield himself by raising his arms in front of his body, but the jogger was able to stab the homeowner, seriously injuring him. If the jogger is prosecuted for aggravated battery, is it likely that he will be found guilty? A No, because he believed the homeowner was attacking him. B No, because he was adequately provoked by the homeowner. C Yes, because he used a deadly weapon. D Yes, because he intentionally created a reasonable apprehension of imminent bodily harm in the homeowner

C Aggravated Battery- Intermediate

Question 4 A plaintiff was injured as a result of a defendant's negligence. The plaintiff hired an attorney, who sent the plaintiff to see a physician for the purpose of examining the plaintiff prior to trial and assessing the extent of his injuries. During the course of the examination, the plaintiff made some statements to the physician indicating that he was not completely free from negligence when the accident occurred. The defendant seeks to call the physician to testify to the statements the plaintiff made to the physician. The attorney objects. How should the court respond? A Overrule the objection, because the plaintiff made an admission. B Overrule the objection, because a physician qualifies as an expert witness. C Sustain the objection, because the attorney-client privilege applies. D Sustain the objection, because the jurisdiction's physician-patient privilege

C Attorney-Client Privilege- Intermediate

Question 4 Which of the following statements most accurately states the prevailing view as to the burden of proving consent or lack thereof in a misdemeanor false imprisonment case? A The defendant must establish the victim's consent by a preponderance of the evidence. B The prosecution must establish that the victim did not consent by a preponderance of the evidence, otherwise the defendant's guilt must be proved beyond a reasonable doubt. C The prosecution must prove beyond a reasonable doubt that the victim did not consent even if the defendant remains silent on the issue. D The prosecution must prove beyond a reasonable doubt that the victim did not consent only if the defendant produces some evidence that the victim did not consent

C Burden of Proof - Intermediate

Question 10 A plaintiff was injured in an automobile accident when her car was hit by a pickup truck driven by the defendant. At trial of her personal injury action, the plaintiff alleges that the defendant was driving on the wrong side of the road in excess of the posted speed limit. The defendant denies these allegations and denies liability for the accident. The plaintiff seeks to introduce evidence that the defendant has a reputation in the community for being a daredevil and for being somewhat irresponsible. In fact, the plaintiffs witness would testify that the defendant is known by all his friends as "the Menace." Is the proffered testimony admissible? A Yes, because reputation evidence is a proper method of proving character. B Yes, because it is relevant. C No, it is inadmissible to show that the defendant was negligent on this occasion. D No, because the defendant did not introduce evidence of his reputation for carefulness

C Character Evidence- Advanced

Question 9 A resident of State A sued a resident of State B in federal district court in State B for breach of contract. Jurisdiction was based on diversity of citizenship. The plaintiff alleged that the contract was entered into in State C and was to be performed in State D. The plaintiff further alleged that the defendant failed to perform. While hearing this case, what substantive law should the federal district court apply? A The law that the State D state court would apply. B The law that the State C state court would apply. C The law that the State B state court would apply. D The law that the federal district court believes most logically applies.

C Choice of Law-Intermediate

Question 4 A man and a woman were arrested and charged with conspiring to blow up a federal government building. After being given Miranda warnings, they were questioned separately and each of them gave a written confession. The confessions interlocked with each other, implicating both of the defendants as being involved in every stage of the conspiracy. Subsequently, the woman attempted to retract her confession, claiming that it was false. At a preliminary hearing, the judge rejected her claim. Both defendants were tried together, and the prosecutor introduced both confessions into evidence. At trial, the woman testified that she was not involved in any conspiracy and that her confession was fabricated. Both defendants were found guilty by the jury. The woman challenges her conviction on appeal because of the admission of the man's confession. If the woman succeeds, what is the likely reason? A The man's confession was more incriminatory to her than her own confession. B The jury was not instructed to consider the man's confession as evidence only of his guilt and not of the woman's. C The man refused to testify at trial and therefore was not subject to cross-examination regarding his confession. D The man testified at trial and was subject to cross-examination but denied making the confession attributed to him

C Co-Defendant's Confession Trial 6th Amend. Confrontation Clause- Intermediate

Question 3 In an effort to protect the dwindling California condor population, Congress enacted the Condor Preservation Act, which made it illegal to take, possess, or sell any pan of a California condor. The constitutionality of the Act is challenged by a seller of gifts and artifacts, including artifacts made out of California condor feathers. Is the statute valid? A No, the statute violates due process because the absolute prohibition on sale is an effective taking under the Fifth Amendment Due Process Clause without just compensation. B No, because the statute is discriminatory as applied. C Yes, because the statute is rationally related to interstate commerce. D Yes, because the statute is designed to protect a dwindling national resource

C Commerce Clause - Intermediate

Question 2 Congress passed a law forbidding the United States mails to be used for the distribution of unsolicited advertising for contraceptives. This led to litigation in the federal courts regarding the constitutionality of the statute. What is the best argument against the constitutionality of the statute? A The statute offends certain rights that give rise to a constitutional right of privacy. B The statute constitutes a taking without due process of law. C The statute improperly infringes on the commercial speech protection of the First Amendment. D The statute unduly burdens interstate commerce

C Commercial Speech - Intermediate

Question 3 A state statute prohibited individuals from donating more than $1,000 per year to any group that lobbies for or against any matter up for consideration before the state legislature. A voter who wanted to donate $5,000 to a lobbying group challenged the statute on constitutional grounds in federal court. Is the court likely to uphold the statute? A Yes, because the statute is reasonably related to the state's legitimate interest in controlling such contributions. B Yes, because the statute does not restrict the core political speech right to donate directly to legislative candidates. C No, because the statute places a restraint on core political speech and association rights without sufficient justification. D No, because a state may not place any limits on the amount of money that may be contributed to

C Contribution as Political Speech & Association Rights - Intermediate

Question 7 An antiques purchaser who did not speak English sued a dealer for breach of contract, alleging that he had agreed to sell her an antique chair for $15,000 but had refused to accept her certified check when she came to pick up the chair. At the trial, the purchaser, through an interpreter, testified that she asked her brother to communicate to the dealer her offer to purchase the chair. She wishes to testify that her brother told her, "The dealer has agreed to sell you the chair for $15,000." The agreement was not reduced to writing and the brother died a few days after that conversation. If the jurisdiction has a typical "Dead Man Act," what effect will the Act have upon the admissibility of the purchaser's conversation with her brother? A It will render the conversation inadmissible because a civil action is involved. B It will render the conversation inadmissible because the purchaser is an interested party. C None, because the dealer is not a protected party. D None, because a civil action is involved

C Dead Man Acts - Advanced

Question 9 A law firm entered into a contract with an office furniture supplier to purchase a substantial amount of office furniture. The contract provided that the supplier would deliver and install the furniture within 30 days. Six months passed and the furniture had not been delivered, and the supplier informed the law firm that the furniture would not be available for at least another month. The law firm then filed a breach of contract action against the supplier in federal district court. The action sought recovery of lost revenues and profits that the law firm alleged were caused by the supplier's failure to deliver the furniture as agreed. The law firm properly served process on the supplier, but the supplier failed to file an answer or otherwise respond to the complaint. After several months passed, the law firm filed a motion requesting that the clerk of court make an entry of default, and the clerk did so. What procedure must the law firm follow to obtain a default judgment against the supplier? A File a motion to have the clerk of court enter the default judgment, which the clerk may do without the supplier receiving any further notice of the motion. B File a motion to have the clerk of court enter the default judgment, which the clerk may do as long as the supplier receives additional notice of the motion for default judgment. C File a motion to have the judge enter the default judgment, which the judge may do without the supplier receiving any further notice of the motion for default judgment. D File a motion to have the judge enter the default judgment, which the judge may do as long as the supplier receives additional notice of the motion for default judgment.

C Default Judgment- Advanced

Question 10 A patron at a private park was injured while operating a small boat he rented from the park. The patron subsequently filed a civil tort action against the park in federal district court, seeking to recover compensatory damages for the injury. Prior to trial the park took the deposition of a witness, another visitor to the park who saw the patron's accident. The park properly served notice of the deposition on all parties, and all parties were represented at the deposition. The deposition took place in the witness's home town, which is 140 miles from the park and the court where the action is pending. At the time of trial, the witness is at home, and the park's attorney seeks to read the witness's deposition testimony into evidence. The attorney for the plaintiff patron objects to the use of the witness's deposition. May the park present the deposition into evidence? A No, because the deposition testimony may be used only if the witness is unavailable to testify live at the trial. B No, because the deposition testimony may be used only to impeach the testimony of the deponent. C Yes, because of the witness's distance from the court. D Yes, regardless of the witness's distance from the court, because all parties were properly notified and had an opportunity to be represented at the deposition.

C Deposition Use at Trial-Intermediate

Question 10 A state requires that persons holding a state license to practice a particular profession reside in the state for at least one year before engaging in that practice. A practitioner who held the state license moved into the state and shortly thereafter contracted with a local business to provide professional services. As soon as he began practicing his profession, the state licensing board sought to sanction him for violating the one-year waiting period. Which of the following provides the strongest basis for the practitioner to challenge the waiting period? A The Privileges and Immunities Clause of Article IV. B The Contract Clause. C The Privileges or Immunities Clause of the Fourteenth Amendment. D The Commerce Clause. afety). Generally, state professional licensing laws fall within the exception

C Privileges and Immunities Clause of 14th Amendment- Advanced

Question 3 To combat rising insurance rates, a state formed a state-owned insurance company that operated exclusively within the state. The company provided insurance on the basis of premiums calculated according to a schedule of fees under the schedule, premiums for residents of a particular city were 25% higher than the premiums for any other municipality in the state. Forty percent of that city's residents were of Mexican descent compared with a state-wide Mexican-American population of approximately 15%. A Mexican-American citizen living in the city brings suit, alleging that the state insurance company's rate structure violates the Equal Protection Clause. What is the most likely result? A The citizen will prevail, because the higher rates have the effect of discriminating against the Mexican-American population. B The citizen will prevail, unless the state insurance company shows a compelling reason for the discrimination. C The state insurance company will prevail, unless the citizen shows that Mexican-American citizens pay higher rates than similarly situated non-Mexican-American citizens of that city. D The state insurance company will prevail, because discriminatory economic regulations are not a suspect classification

C Equal Protection - Discrimination - Intermediate

Question 1 As part of a program to meet the needs of a growing homeless population, a city council appropriated funds to construct a shelter and community center in the downtown area and sought bids from organizations interested in operating the center. The group submitting the lowest bid was a religious organization, which ran a number of shelters and food kitchens in adherence to the religion's central tenet of aiding the needy. While church members never actively preached to people using the shelters, it did make available reading materials about its religion. The only other bidder was a local nonprofit foundation not affiliated with any church. The nonprofit foundation concluded that it was underbid because it was subject to a state tax imposed on all facilities offering overnight lodging, whether run for profit or not for profit; the only exemption was for facilities run by an organized religion. If the nonprofit foundation seeks to challenge the application of the state tax to its operation of the homeless shelter, which of the following statements is most correct? A Imposing the tax on the church's operation of homeless shelters, which is mandated by the religious beliefs of its members, would improperly inhibit their free exercise of religion. B Maintaining the tax exemption for the church's operation of homeless shelters is a proper means of avoiding excessive government entanglement with religion. C Permitting a tax exemption for the church-run shelter and not for a shelter run by other not-for-profit institutions has the unconstitutional effect of advancing religion. D The tax exemption only for church-run shelters violates the Equal Protection Clause of the Fourteenth Amendment because the state does not have a compelling interest justifying application of the tax to some organizations that operate shelters and not others

C Establishment Clause -Intermediate

Question 7 Two partners who operated an electronics retail store hired a thug to intimidate the owner of a discount electronics store that was undercutting their prices and drawing customers away. They told the thug to rob the store owner but not to harm him; they only wanted to scare him out of town. The thug loitered near the discount store, waiting for it to close. When the lights in the store went out, an employee left by the back exit. The thug thought he was the owner and drew his gun, demanding money. The employee resisted, and in the ensuing struggle was fatally shot. The thug ran off, but both he and the partners were eventually arrested. In addition to conspiracy to commit robbery, are the partners guilty of any other crime(s)? A No other crimes. B Robbery. C Felony murder. D Robbery and felony murder

C Felony Murder Conspiracy- Advanced

Question 1 The defendant is charged with the criminal battery of the victim, a security guard in a drug store. At the trial, the prosecution introduces evidence that while the victim was attempting to question the defendant about a suspected shoplifting incident, the defendant committed a battery on the victim. The defendant claims self-defense, and offers into evidence a judgment for damages against the victim for battery against another person under similar circumstances three months earlier. Would evidence of the judgment be admissible? A Yes, but only in a subsequent civil trial. B Yes, for the purpose of establishing the victim's dangerous propensities. C No, because it is hearsay not within any recognized exception. D No, because it is irrelevant

C Hearsay -Advanced

Question 6 A plaintiff suffered injuries when her car was struck by the defendant's car. The police arrived on the scene and required the defendant to take a breathalyzer test. The defendant was cited for driving while intoxicated, tried in traffic court, and duly convicted. He received the maximum sentence for driving while intoxicated, which is 90 days' imprisonment in the county correctional facility. The plaintiff brought a civil action against the defendant, seeking compensation for her personal injuries. At the trial of the plaintiff's suit, the plaintiff's attorney offers a properly authenticated photocopy of the court judgment showing that the defendant was convicted of driving while intoxicated. Is the evidence admissible? A Yes, as a public record. B Yes, as a final judgment offered to prove a fact essential to a point in controversy. C No, because the crime was punishable by imprisonment of at most 90 days. D No, because it is not the best evidence of the defendant's conviction

C Hearsay-Prior Convictions - Advanced

Question 9 A defendant was charged with the murder of a victim. During the course of the criminal trial, a witness testified on behalf of the defense that, at the time the murder took place, he saw someone who looked like the defendant dancing at a local nightclub. The defendant is eventually acquitted of the charge. Following the acquittal, the appropriate survivors of the victim bring a wrongful death action against the defendant. As pan of her defense, the defendant wishes to introduce the testimony given at the criminal trial by the witness, who the defendant shows is now incarcerated in a prison in another state. Is the testimony of the witness admissible? A Yes, because the witness testified under oath at another hearing related to the same subject matter. B Yes, because the defendant is a party to both proceedings. C No, because the plaintiffs were not parties to the criminal proceeding. D No, because the witness can be subpoenaed to testify

C Hearsay-Prior Testimony - Advanced

Question 7 A corporation operated several factories that were emitting toxic chemicals into the air. The Environmental Protection Agency ("EPA") sued the corporation in federal court for violation of several environmental laws, and the jury found in favor of the EPA. One of the corporation's factories was located near a town, and some residents believed that their health had been harmed by the factory's emissions. One resident sued the corporation in federal district court, alleging damages stemming from the factory's violation of the environmental laws. The resident asserts issue preclusion (collateral estoppel) to establish the factory's violation of the environmental laws. If the court permits the resident to use issue preclusion to establish the violation, what is the likely reason? A A nonparty in the first case is always allowed to use issue preclusion (collateral estoppel) offensively or defensively against someone who was a party in the first case. B There was a valid, final judgment on the merits in the first case, and the same cause of action is involved in the resident's lawsuit. C The court determined that it is fair and equitable to allow the resident to use issue preclusion (collateral estoppel) offensively. D A party can use issue preclusion (collateral estoppel) offensively when the defendant has been found to have violated a federal law.

C Issue Preclusion/Collateral Estoppel-Intermediate

Question 9 A college student was the sole lifetime beneficiary under a large trust administered by a banker. The student received a large monthly distribution from the trust, and whenever he ran shot he simply called the banker for extra funds, because the trust provided that the student was to receive whatever he needed from income or principal. The student's roommate found out about the trust arrangement and decided to see if he could make it pay off for him. The roommate sent an e-mail to the banker, which appeared to be from the student, and which asked for several thousand dollars to cover medical expenses. The e-mail further stated that, since he was in the hospital, the student would send his roommate to pick up the cash. The next day, the roommate showed up at the banker's office and obtained the money on the promise that he would take it to the student. The roommate absconded with the funds. When the roommate obtained the cash from the banker, what crime did the roommate commit? A False pretenses B Embezzlement C Larceny by trick D Larceny

C Larceny by Trick-Advanced

Question 2 A plaintiff brought a conversion action against a defendant, alleging that the defendant had wrongfully taken a necklace owned by the plaintiff's mother, who had recently died intestate. The defendant's defense is that the mother, who had lived next door to the defendant, had freely given her the necklace because she had often given rides to and run errands for the mother, who did not drive and had trouble getting around. The defendant is cross-examined by the plaintiff's attorney, who challenges the defendant's claim that the mother had given her the necklace. The defendant wishes to testify that, before handing her the necklace, the mother told her, "You've always been so good to me that I want you to have this necklace." Is the defendant's proposed testimony admissible? A No, because it is hearsay not within any exception. B No, under the state's Dead Man Act. C Yes, because it is nonhearsay. D Yes, because the declarant is unavailable to testify

C Nonhearsay, Legally Operative Facts Advanced

Question 8 A 17-year-old high school student decided it would be great fun to scare his teacher by playing some sort of prank on her. He told his 16-year-old friend about his desire, and the friend, knowing the student's propensity for doing such stunts, encouraged him to go through with his plan, giving him a toy rubber snake to put in the teacher's desk. After opening her desk and having the rubber snake pop out, the teacher jumped back and fell, injuring her hip. A state statute provides that "anyone who recklessly causes bodily injury to the person of another is guilty of battery in the third degree." What is the best theory for charging the friend with a battery in the third degree? A The friend is an accomplice because he encouraged and provided assistance to the student so that he could commit the crime. B The friend is a co-conspirator because he agreed with the student to carry out the plan. C The friend is a principal and committed acts of recklessness of his own that constituted the crime. D By supplying the instrumentality of the crime, he is vicariously liable for his friend's actions

C Parties to a Crime -Advanced

Question 9 A woman was arrested, given Miranda warnings, and questioned about an armed robbery. After she asked to speak with an attorney, the police stopped questioning her about the robbery. Several hours later, the police gave the woman a fresh set of Miranda warnings and began to question her about a different robbery. She did not repeat her request for an attorney and instead made several incriminating statements about the robbery. At the woman's trial for the robbery for which she made incriminating statements, the prosecution seeks to have her statements introduced into evidence. If the woman's attorney objects on appropriate grounds, how should the court rule? A Overrule the objection, because the police did not badger the woman into confessing. B Overrule the objection, because the woman did not renew her request for an attorney after receiving fresh Miranda warnings. C Sustain the objection, because the police did not honor the woman's request. D Sustain the objection, because a confession obtained in violation of a defendant's Miranda rights but otherwise voluntary may be used against the defendant

C Police Responsibility to Honor Request for Attorney - Advanced

Question 10 A city council and park board announced joint plans to teardown some old buildings and erect a park. Before the contracts were made, in order to garner the greatest political benefit from such projects, the city council adopted an ordinance requiring that 35% of the work force of contractors working on city-funded projects be residents of the city. One of the contractors working on the park project employed several people from the city, but he and most of his employees came from a town in a neighboring state that was a few miles west of the city. When the city projects inspector discovered that the contractor did not employ the required 35%, he told the contractor that if he did not hire a sufficient number of city workers within 20 days the contractor would forfeit the opportunity to work on the project. The contractor immediately filed an action in federal court seeking to have the employment requirement declared unconstitutional. Of which party should the court should rule in favor? A The city, because it is acting as a "market participant" here. B The city, because there is a rational basis for favoring city residents here. C The contractor, because the requirement interferes with his rights under the Privileges and Immunities Clause of Article IV. D The contractor, because the requirement interferes with his Contract Clause rights

C Privileges & Immunities Clause - Advanced

Question 5 A state statute made it unlawful to sell milk for home consumption in containers less than one quart in size. Violation of the statute was a misdemeanor, punishable by a $500 fine and loss of the retail business license. A convenience store within the state specialized in sales of pints of milk to walk-in lunchtime buyers. During a routine inspection, a state inspector discovered that the store was selling milk in pints and immediately revoked the owner's retail license pursuant to the statute. What is the owner's best argument in a suit to defeat the revocation of her retail license? A The action of the state agency impaired her contract with wholesale distributors of pint canons of milk. B The action of the state agency was a denial of equal protection. C The action of the state agency was a denial of procedural due process. D The action of the state agency was a denial of substantive due process

C Procedural Due Process- Intermediate

Question 1 As part of a new environmental policy, a state legislature enacted a statute creating a farmland development agency. The agency was organized along the lines of a private corporation. The agency's purpose was to restore and cultivate polluted land in the state, making the land suitable for farmers. Once the agency cultivated the land, it would sell the land to new farmers at a reduced price. The agency began cultivating and selling farmland in one county where the pollution levels were high, but did not operate in a neighboring county with slightly lower pollution levels. A group of residents in the second county bring an action in federal court against the agency director, seeking to compel the agency to begin operations in their county. What will be the probable outcome of this litigation? A The action will be dismissed, because the state is immune from litigation under the Eleventh Amendment. B The action will be dismissed, because the agency is organized as a private business and thus does not engage in any state action. C The state will prevail, because it has a rational basis for not operating in the neighboring county. D The citizens will prevail, because the state cannot show a compelling state interest in not operating in their county.

C Rational Basis - Intermediate

Question 3 A defendant is on trial for murder. The only evidence linking the defendant to the crime is some blood found at the scene. The lead detective testifies that an officer took a vial containing a blood sample that had been retrieved by a crime scene technician and drove off with it. The officer is now dead. Next, the prosecution presents as a witness a crime lab chemist. The chemist will testify that he took a vial of blood that contained a label identifying it as having been retrieved from the subject crime scene, and that he performed tests that established a match between that blood and a blood sample taken from the defendant. Is the testimony of the chemist admissible? A Yes, because there has been proper authentication. B Yes, because the chemist qualifies as an expert witness. C No, because there is insufficient evidence of chain of custody. D No, because he did not take the original blood sample at the scene of the crime

C Real Evidence, Chain of Custody - Intermediate

Question 9 A buyer entered into a written contract with a farmer to purchase the farmer's dairy farm. The contract contained a provision that the farmer's "land and inventory are valued at $175,000." The contract also contained a provision that stated, "This contract represents the entire agreement between the parties. No other promises or representations have been made." In a fraud action against the farmer, the buyer alleges that he purchased the farm only because the farmer had assured him that the land and inventory were worth $350,000, when they were in fact worth only $175,000. The buyer seeks to testify that during negotiations the farmer had said repeatedly that the value of the land and the inventory was $350,000, but at the advice of the farmer's attorney, he was going to list the value at $175,000 in the contract for tax purposes. The farmer's attorney objects. How should the judge rule? A The evidence is inadmissible hearsay. B The evidence is inadmissible under the parol evidence rule. C The evidence is neither hearsay nor violative of the parol evidence rule. D The evidence is admissible hearsay, unaffected by the parol evidence rule

C Relevance -Advanced

Question 2 Three drivers were in an automobile accident in a city in State A. The drivers were citizens of State A, State B, and State C. The State B driver filed a tort action against the other two in a State A state court, seeking $300,000 from both of the other drivers. After extensive discovery over a period of one year and three months, the State B driver settled and dismissed her claims against the State A driver. After that dismissal, only the $300,000 claim against the State C driver remained. May the remaining State C driver properly remove the action to federal district court? A Yes, if he files a notice of removal within 30 days, because there is complete diversity of citizenship and the requisite amount in controversy, and there is no longer a defendant who is a citizen of the forum state of State A. B Yes, at any time, because complete diversity of citizenship and the requisite amount in controversy were satisfied from the time the action was filed. C No, because removal on the basis of diversity of citizenship jurisdiction is not available after an action has been pending in state court for more than one year. D No, because the State A and State C drivers did not remove the action within 30 days of the action being filed.

C Removal- Intermediate

Question 6 A patient sued her doctor in federal court for medical malpractice. During the trial, the evidence heavily favored the doctor, but the jury returned a verdict in favor of the patient. Judgment was entered for the patient on November 1. No motions were filed during the course of the trial. The doctor believes that a reasonable jury could not have found in favor of the patient and wants to set aside the verdict. On November 20, the doctor files a renewed motion for judgment as a matter of law. Will the doctor's motion be successful? A Yes, because he filed the motion within 28 days of entry of judgment. B Yes, because the evidence was heavily in his favor. C No, because he cannot use this motion unless he moved for judgment as a matter of law at some time during the trial. D No, because renewed motions for judgment as a matter of law must be filed no later than 14 days after entry of judgment.

C Renewed Motion for Judgment as a Matter of Law-Intermediate

Question 6 Acting with probable cause, the police arrested a man in connection with the armed robbery of a liquor store. After being given Miranda warnings, the man confessed to the robbery but denied his involvement with several other recent armed robberies of businesses in the area. He was formally charged with the one robbery and put into a cell with a paid informant working undercover for the police. The informant had been instructed to find out what he could about the other robberies but not to ask any questions. The informant began talking about a convenience store robbery in which a bystander was shot and seriously injured by the robber, and he deliberately misstated how it happened. The man, unaware that his cell mate was an informant, interrupted to correct him, bragging that he knew what really happened because he was there, and proceeded to make incriminating statements about the robbery. The man was subsequently charged with armed robbery and attempted murder in the convenience store robbery. At a motion-to-suppress hearing on that charge, if the man's attorney moves to exclude the statements made to the informant, should the motion be granted? A Yes, because the informant deliberately elicited incriminating statements in violation of the man's Sixth Amendment right to counsel. B Yes, because the informants conduct constituted custodial interrogation in violation of the man's Fifth Amendment privilege against self-incrimination. C No, because the man had not yet been charged with the robbery of the convenience store when he made the statements to the informant. D No, because the informant's conduct did not constitute interrogation

C Right to Counsel - Advanced

Question 2 A defendant who had been arrested for running an illegal narcotics operation hired the top criminal defense attorney in the area to represent him. Before his trial, he was notified that virtually all of his money was going to be seized by the government because he had earned it by selling drugs. The government then entered a restraining order to that effect, preventing him from making payments on services for which his attorney had billed him. May the government seize the funds earmarked for the defendant's attorneys' fees? A No, because the seizure violates the defendant's Sixth Amendment right to counsel. B No, because the attorney had already provided the services and billed his client for those services. C Yes, because the funds were obtained through selling illegal narcotics. D Yes, because an attorney may not accept funds obtained through illegal activity

C Right to Counsel -Intermediate

Question 6 A man was tried in state court for possession of heroin. The prosecution offered in evidence five rolled-up toy balloons containing heroin, which police officers had found on a table in the man's apartment. At a hearing on the defense motion to suppress, testimony was presented that established that the police had put the apartment under surveillance, and had watched a police informant go to the door of the apartment, hand four balloons of heroin to the man, and leave. The police had then knocked on the apartment door, identified themselves as police officers, and demanded entrance. Having heard nothing for 30 seconds, the police had then broken down the door and entered the apartment, discovering the heroin. The police had intended to arrest the man for the purchase of heroin, a felony. When they had gotten inside the apartment, they discovered that the man had left by a back exit. He was later arrested at the nearby newsstand. The trial court denied the motion to suppress, and the case is on appeal following the man's conviction for possession of heroin. How should the appellate court rule? A Affirm the conviction on the ground that the error, if any, in admitting the heroin was harmless error. B Affirm the conviction on the ground that the police complied with the "knock and announce rule" even though no one was there to admit them. C Reverse the conviction on the ground that the man's Fourth Amendment rights (as applied to the states by the Fourteenth Amendment) have been violated. D Reverse the conviction on the ground that the "knock and announce rule" was not satisfied when the police announced their presence and identity to an empty residence

C Search & Seizure - Warrantless Forcible Entry - Advanced

Question 7 While executing a search warrant at the home of a suspected arsonist, the police heard a knock on the door. A plain clothes officer the door and found a young woman standing outside with a backpack in her hand. The woman asked for the owner of the home by name and was told that the owner was not available at the moment. The woman replied, "Give him this, and tell him thanks for the $8,000," and she gave the officer the backpack. The officer opened the backpack and found eight hi-tech delay timers. The officer then identified himself as a police officer and placed the woman under arrest. He performed a quick patdown of the woman's outer clothing. In her jacket pocket, the officer found a package that appeared to be (and later proved to be) heroin. In addition to arson-related charges, the woman was charged with possession of heroin. At trial, the woman's attorney moved to have the heroin excluded from evidence. How should the court rule? A For the woman, because the warrant only entitled the police to search the premises. B For the woman, because the heroin was seized during an unlawful arrest. C For the state, because the heroin was seized during a lawful arrest. D For the state, because searching the backpack was within the scope of the warrant

C Search Incident to Lawful Arrest - Intro

Question 2 A defendant was convicted after a jury trial of violation of federal statutes prohibiting the sale of automatic weapons to foreign nationals. It was established at trial that the defendant had purchased a number of stolen United States Army heavy machine guns and attempted to ship them abroad. The trial court expressly based its imposition of the maximum possible sentence for the conviction on the defendant's refusal to reveal the names of the persons from whom he purchased the stolen weapons. His counsel argues that this consideration is reversible error. If the defendant appeals the sentence imposed, what should the appeals court do? A Reverse the trial court, because the consideration of the defendant's silence violates his Fifth Amendment privilege against self-incrimination. B Reverse the trial court, because the consideration of collateral circumstances in sentencing violates his due process rights. C Affirm the trial court, because the right to remain silent granted by the Fifth Amendment does not include the right to protect others from incrimination. D Affirm the trial court because citizens must report violations of the criminal statutes

C Self-Incrimination and Incrimination of Others - Intermediate

Question 7 The United States Surgeon General was cited for contempt for refusing to answer questions as part of a Senate investigation regarding an issue in the Food and Drug Administration. His contempt citation will be dismissed if he can show which of the following? A As a member of the executive branch, he is immune from prosecution. B If he answered the questions, he could be subject to dismissal from his position as Surgeon General. C The questions do not relate to any matter concerning which the Senate may legislate. D The questions do not relate to any matter concerning current or planned legislation

C Separation of Powers - Advanced

Question 5 A homeowner, domiciled in State A, hired a contractor, domiciled in State B, to build an addition to his residence. Dissatisfied with the contractor's work, the homeowner refused to pay the contractor. The contractor promptly commenced a contract action against the homeowner by filing a complaint in the federal court for State B, seeking damages in excess of $75,000. The contractor's lawyer went the homeowner's residence in State A and attempted to hand the summons and complaint to a man who was painting the front porch of the residence. The lawyer told the man that he was serving legal papers. When the man refused to accept the papers, the lawyer touched the papers to the man's arm and placed them on a nearby bench on the porch. The lawyer assumed that the man was the homeowner. In fact, he was the homeowner's neighbor, whom the homeowner had hired to paint the porch while he was vacationing. What is the homeowner's best argument that service was improper? A Process was served in State A. B Process was served by the contractor's lawyer. C The man was the homeowner's neighbor. D The man refused to accept the papers.

C Service of Process-Intermediate

Question 7 In an effort to curb underage drinking, the police staked out a liquor store near a college campus that was suspected of selling liquor to minors in violation of state law. They recorded a liquor store clerk being told by a minor that he was an underage student at the local university and that he would pay triple the marked price for a case of beer. The clerk readily agreed to the deal and took the money, but both parties were arrested before they could complete the transaction. In a jurisdiction following the unilateral approach to conspiracy, which of the following statements is correct? A The minor can be convicted of solicitation and the liquor store clerk can be convicted of conspiracy to violate the statute. B Both the minor and the liquor store clerk can be convicted of conspiracy to violate the statute. C The minor cannot be convicted of either solicitation or conspiracy, but the liquor store clerk can be convicted of conspiracy to violate the statute. D The minor cannot be convicted of either solicitation or conspiracy, and the liquor store clerk cannot be convicted of conspiracy

C Strict Liability Drinking Unilateral Conspiracy- Advanced

Question 4 A citizen of State A asserted a state law claim of $80,000 against a citizen of State B in the federal district court. The State B citizen then brought a third-party impleader claim against another citizen of State B. After learning of the third-party action, the State A citizen decided to bring his own related state law claim against that person for $90,000, and amends his original complaint accordingly within the period for amending a complaint as a matter of right. Does the court have subject matter jurisdiction over the claim in the amended complaint? A No, because the amount in controversy is insufficient. B No, because there is no diversity of citizenship. C Yes, because the requirements of diversity jurisdiction have been met. D Yes, because the court has supplemental jurisdiction.

C Subject Matter Jurisdiction/Diversity- Intermediate

Question 7 A witness was stopped at an intersection when she saw a car run a red light, strike the victim in the crosswalk, and proceed through the intersection. The witness gave a very detailed description of the driver to the police officer at the scene. Based on this description, the police apprehended the defendant and charged him with several criminal counts for the accident that seriously injured the victim. The witness testified at the trial, but the defendant was acquitted. The victim then filed a civil suit against the defendant to recover for her injuries. Before the trial of the victim's suit, the witness died. In her suit against the defendant, the victim offers into evidence the police report containing the witness's description of the driver. The defendant objects. How should the court rule on the admissibility of the report? A Admissible, because the report is relevant, and it is not hearsay. B Admissible, because the report falls within the business records exception to the hearsay rule. C Inadmissible, because the report is hearsay not within any exception. D Inadmissible, because the report is not the best evidence

C Traffic Accident Report - Advanced

Question 6 After a state supreme court overturned the conviction in a murder case for failure to give proper Miranda warnings, a reporter asked the murder victim's father to comment on the case as he exited the supreme court building. The father made the following statement: "Each one of the so-called supreme court justices is worse than a murderer, because they make it possible for more sons and daughters to be murdered. I'd like to see every one of them strung up, like they should have done to the creep who was set free, and if someone will give me a rope I'll go in there and do it myself." A state statute proscribes, with criminal penalties, "the making of any threat to the life or safety of a public official for any act the official performed as pan of the official's duties in office." Which of the following is correct regarding the statute? A The victim's father could constitutionally be punished under the statute, but only if the state supreme court justices heard the threats he made. B The victim's father could constitutionally be punished under the statute. C The victim's father could not be constitutionally punished under these circumstances, but the statute is constitutional on its face. D The statute is unconstitutional on its face

C True Threat vs. Political Speech- Advanced

Question 8 The plaintiff is a resident of City A, located in the Northern District of State A. The defendant is a resident of City B, located in the Eastern District of State. The defendant negligently ran a red light in City B, resulting in a collision between his car and the delivery van that the plaintiff was driving. The result: 100 pounds of cork that the plaintiff was delivering wound up striking him in the back of the head, causing severe injuries. The plaintiff sued the defendant in the Northern District of State A. The defendant moves to dismiss based on improper venue. How should the court rule on the defendant's motion to dismiss? A Grant the motion, because the Northern District of State A is not a place of proper venue. B Grant the motion, because the court does not have personal jurisdiction over the defendant. C Deny the motion, but it should transfer the case to the Eastern District of State B. D Deny the motion, because the Northern District of State A is a place of proper venue.

C Venue- Advanced

Question 2 While driving in the Western District of State A, a citizen and resident of State B caused an accident that injured a citizen and resident of the Southern District of State A. The State A citizen filed a negligence action against the State B citizen in the United States District Court for the Southern District of State A. The State B citizen filed a motion seeking to dismiss the action on the ground that the court lacked personal jurisdiction over him. The court denied the motion. The State B citizen then filed an answer as well as a motion to dismiss the action for improper venue. How should the court rule? A The court should grant the motion to dismiss, because the Southern District of State A is not a proper venue. B The court should grant the motion to dismiss, because the court lacks personal jurisdiction over the State B defendant. C The court should deny the motion to dismiss, because it was not timely asserted, and any objection to venue has been waived. D The court should deny the motion to dismiss, because the Southern District of State A is a proper venue.

C Venue- Intermediate

Question 2 A large sign fell from the ceiling of a department store and hit one of the store's customers, causing severe injuries. The store manager knew the accident would almost certainly result in litigation. Accordingly, after the injured customer was cared for and taken to the hospital by ambulance, the store manager conducted a brief investigation by observing the area and interviewing people who were in the area of the accident. The manager then created a digital document on his computer in which the manager summarized the information he obtained from the interviews in case litigation ensued. He never printed the document. A few months later, the customer filed an action against the store. During a deposition of the store manager, the customer's lawyer learned about the digital document on the managers computer that summarized the managers investigation. The customer's lawyer requested a copy of the digital document, but the store objected to the request on the grounds that the document is protected from discovery under the work product doctrine. Is the customer entitled to obtain a copy of the digital document? A Yes, because the work product doctrine protects only physical documents and tangible things. B Yes, because the document was not created by or at the direction of the store's attorney. C Yes, if the customer demonstrates substantial need for the document and an inability to obtain the substantially equivalent information by other means without undue hardship. D No, because the work product doctrine precludes any discovery of the document.

C Work Product- Intermediate

Question 4 The mayor of a town received several complaints from residents regarding the growing number of adult theaters and nude dancing establishments in a nearby town. To allay fears, the mayor asked the town's attorney what could be done to prevent or at least limit such establishments from setting up business in their town, which currently follows a zoning plan that provides for residential, commercial, and light industrial uses/ Which of the following most correctly describes the town's constitutional options? A The town may revise its zoning ordinance to prohibit adult theaters and nude dancing establishments because erotica is unprotected speech. B The town may revise its zoning ordinance to limit the location of adult theaters and nude dancing establishments only if this serves a compelling interest. C The town may revise its zoning ordinance to limit the location of adult theaters and nude dancing establishments to control the secondary effects of such businesses. D The town may not limit the location of either adult theaters or nude dancing establishments in any manner different from limitations on other commercial establishments

C Zoning- Intermediate

9. A landowner devised her campground in her will "to my niece, her heirs, and assigns, so long as it is used for camping and recreational purposes; if used for any other purpose during her lifetime, then to the Girl Scouts of America." Subsequently, the landowner died. The residuary clause of her will left all property not devised in the remainder of the will to her daughter and sole heir. Soon thereafter, the daughter died intestate, her only heir being her son. Last month, the niece entered into a contract to sell the campground to a buyer for its reasonable market value. After the buyer received the title report called for in the contract, he refused to proceed with the purchase, claiming that the niece could not convey good title. The niece, the Girl Scouts of America, and the buyer then execute a new contract calling for the former two parties to sell the property at the same price to the latter. The jurisdiction follows the common law Rule Against Perpetuities, and a statute provides that future estates and interests are alienable, and may be devised or inherited, all in the same manner as possessory estates or interests. Should the buyer proceed with the new purchase transaction? (A) Yes, because good title can now be conveyed by the sellers. (B) Yes, if the Girl Scouts of America promises never to use its right of entry should the buyer use the property for other than camping and recreational purposes. (C) No, because the daughter's son has not been included as a party selling the property. (D) No, because no one can convey good title to the property during the niece's lifetime.

C present and future estates, marketable title

2. A buyer purchased a parcel of land from a seller for$500,000. The buyer financed the purchase by obtaining a loan from the seller for $300,000 in exchange for a mortgage on the land. The seller promptly and properly recorded his mortgage. Shortly thereafter, the buyer gave a mortgage on the land to a creditor to satisfy a preexisting debt of $100,000 owed to the creditor. The creditor also promptly and properly recorded its mortgage. Within a year, the buyer stopped making payments on both mortgages, and the seller brought an action to foreclose on his mortgage. The creditor was not included as a party to the foreclosure action. The seller purchased the property at a public foreclosure sale in satisfaction of the loan. The creditor subsequently discovered the sale and informed the seller that it was not valid. Who has title to the land? (A) The seller, because he gave a purchase money mortgage and the creditor's mortgage was for a preexisting debt (B) The seller, because the public foreclosure sale extinguished the creditor's interest (C) The seller, but he must redeem the creditor's mortgage to avoid foreclosure (D) The buyer, because the seller's foreclosure action was invalid without the inclusion of the creditor as a necessary party

C, Foreclosure, effect on priorities of interests

1. To secure a loan of $100,000 from a bank, the owner in fee simple of a parcel of land conveyed a deed of trust for the land to the bank. The deed of trust contained a "power of sale" clause, permitted by the jurisdiction, which allowed the bank to sell the property in the event of default without the necessity of a judicial foreclosure action. After several years, the owner defaulted on his loan payments to the bank. The bank informed the owner that it was exercising its power of sale. After appropriate notices, the bank conducted a public sale of the land. The bank was the sole bidder and obtained the property for $80,000, which was $10,000 less than the outstanding balance on the loan plus the expenses of the sale. One month later, the owner notified the bank that he wanted to pay off the loan and extinguish the deed of trust, and was prepared to tender $80,000 to do so. The bank insisted that the owner must tender $90,000 to pay off the loan. If a court in the jurisdiction will require the bank to accept only $80,000 under the circumstances above, what is the likely reason? (A) The owner had the power to revoke the trust as long as he was alive. (B) The bank did not have the authority to bid on the property at other than a judicial foreclosure sale. (C) The owner was exercising a statutory power rather than an equitable power. (D) The bank does not have the power to clog the equity of redemption.

C, Foreclosure, redemption in equity

7. A zoologist decided to install an artificial koi pond in his backyard. He asked his neighbor, a landscaper, for advice on doing the installation himself. After the zoologist dug a trench for the drainpipe, the neighbor complained that her garage, located only 20 feet from the trench, had developed numerous cracks in the foundation and walls. What is the zoologist's best defense to an action by the neighbor for the damage to her garage? (A) The zoologist gave notice to the neighbor of his intention to excavate and she made no objection. (B) A professional would have excavated in the same manner. (C) The zoologist's excavation was done in a proper manner, and there would have been no subsidence except for the fact that the garage was there. (D) The garage was made of solid concrete and thus was unusually heavy.

C, Right to lateral and subjacent support of land, support of buildings on land

9. A department store buyer and a manufacturer of food processors entered into a written contract whereby the manufacturer would sell to the buyer 50 of its top-of-the-line models for $100 each. When the delivery arrived on May 15, several days early, the buyer noticed that the food processors were a different model that did not have all of the features as the top-of-the-line model that was ordered. The buyer contacted the manufacturer and told him that he was rejecting the food processors that were delivered to him and expected the manufacturer to send 50 top-of-the-line models immediately. The manufacturer replied that because of a backlog of orders that had not yet been filled, the top-of-the-line models could not be delivered until August 15. Because the department store had contracted with a restaurant to deliver three top-of-the-line models by May 31, the buyer delivered three of the nonconforming food processors along with a promise to replace them with three top-of-the-line models in mid-August. The buyer returned the remaining food processors to the manufacturer. How much could the department store recover from the manufacturer for the three food processors that it delivered to the restaurant? (A) Nothing, because they were resold to another. (B) Nothing, because it accepted them knowing they were defective. (C) The difference between the market price of the top-of-the-line models and the existing food processors' actual value. (D) The difference between the existing food processors' actual value and the cost of the food processors that the department store must provide to the restaurant in mid-August.

C- Contract Damages

5. The owner of 10 acres of undeveloped grassland decided to open a public park. The entrance to the park was to be by a street that he planned to construct on the edge of his property. The owner's plan was approved and recorded, but construction of the street has been delayed awaiting permits. After the plans for the park were announced to the public, a family purchased an adjacent plot, from another party, planning to build a home. The family applied for a building permit that would allow them to gain access to their plot from the planned yet still undeveloped street, which was referenced in their deed. The owner of the grassland objects to the granting of the building permit on the ground that he never granted any rights to anyone to use the street. There are no applicable statutes dealing with this situation. The family brings an appropriate action to determine their rights to build an access connecting the adjacent plot and the street. What is the best argument for the grassland owner in this action? (A) The Statute of Frauds prevents the introduction of evidence that might prove the necessity for the family to use the street. (B) The family would be unjustly enriched if they were permitted to use the street. (C) The family's right must await the action of appropriate public authorities to open the street as a public street, because no private easement arose by implication. (D) The family's right to use the street is restricted to the assertion of a way by necessity, and the facts preclude the success of such a claim.

C- Easement, Public Street

5. A photography buff wrote a letter to his brother-in-law offering to sell him his camera for $1,500, because he knew that he had admired it for a long time. The day after the brother-in-law received the letter, he mailed a letter back to the photography buff agreeing to purchase the camera equipment for $1,500. The next day, after describing the camera to a friend who was very knowledgeable about photographic equipment, the brother-in-law learned that the camera was second-rate and not worth more than $1,200. He immediately telephoned the photography buff and told him that he had no interest in buying the camera. The photography buff received his brother-in-law's letter agreeing to purchase the camera equipment a day after receiving the phone call. If the photography buff brings an action against his brother-in-law for breach of contract, and the brother-in-law defends on the grounds that no contract was formed, how should the court rule? (A) For the brother-in-law, because the description of the subject matter of the contract was too indefinite to be enforced. (B) For the brother-in-law, because the photography buff received the telephone call before he received the letter. (C) For the photography buff, because his brother-in-law's letter accepting the offer was effective when mailed. (D) For the photography buff, because the contract is for the sale of goods over $500 in value and his brother-in-law's attempted rejection of the offer was oral.

C- Mutual Assent; Offer and Acceptance; The Acceptance; When Effective; The Mailbox Rule

7. A building contractor entered into a contract with the local college to remodel a residence hall during the summer. As specified by the contract, the work had to be completed before the fall semester began at the beginning of September. Because the contractor received a great deal of other maintenance business from the college, his price of $400,000 was significantly lower than other contractors and he was not going to demand payment until the work was completed. By the end of the first week in August, the contractor had completed 75% of the project and had expended $350,000 in labor and materials. At that time, however, a labor dispute between the contractor and his employees prompted most of the workers to walk off the job. Because prospects for a quick settlement of the dispute were doubtful, the contractor informed the college that he would not be able to meet the completion deadline. A week later, the college obtained another contractor who was able to finish the project by the end of August. The college paid him $150,000, which included a substantial amount of overtime for his workers. The increase in value of the residence hall due to the remodeling was $425,000. The original contractor, who had not been paid, files suit against the college, which files a counterclaim against him. What should the contractor recover from the college? (A) Nothing, because the contractor breached the contract. (B) $200,000 in restitutionary damages, which is the difference between its expenditures and the amount the college paid the other contractor to complete the work. (C) $250,000 in restitutionary damages, which is the contract price minus the amount the college paid the other contractor to complete the work. (D) $275,000 in restitutionary damages, which is the difference between the value of the completed remodeling and the amount the college paid the other contractor to complete the work.

C- Restitution Damages

1. A landlord leased a building to a baker for 10 years, commencing January 1, at a monthly rental of $1,700. The lease stated in part, "The tenant may not sublet or assign this lease without first receiving written permission from the landlord to do so. Any attempt to sublet or assign the lease without first receiving written permission shall constitute a breach entitling the landlord to terminate this lease." Five years later, an investor approached the baker and offered to purchase the bakery if the baker would agree to sublet the premises to him. The baker agreed and executed a sublease on July 1 of that year. The investor took possession the same day. On July 3, the baker approached the landlord and asked for written permission to sublet the premises to the investor. The landlord said he had no real objection to the sublease and would execute the document requested by the baker, but only if the investor would sign a five-year extension of the existing lease. The investor refused to extend the lease, but remained in possession of the building. At no time did the landlord accept rent from the investor. After notice was given to all parties and the applicable grace period in the lease had elapsed, the landlord brought an appropriate action against the baker and the investor to evict them from the premises and to declare the lease terminated because it had been breached. How should the court rule in this action? (A) Against the landlord, because his withholding consent is an invalid restraint on alienation. (B) Against the landlord, because his conditional consent operated as a waiver of the term of the lease requiring the landlord to give written permission for subletting. (C) For the landlord, because the baker has breached the lease. (D) For the landlord, because his oral consent to sublet is not enforceable under the Statute of Frauds.

C-Landlord Tenant

Question 6 A homeowner looked out his front window one day and saw a neighbor standing on a narrow ledge on the second story of the house across the street. He also saw a ladder lying on the ground beneath where the neighbor was stranded. The homeowner ran out and picked up the ladder and placed it against the side of the house. However, he set it atop a patch of ice. As the neighbor climbed down the ladder, a rotten rung broke and he fell to the ground and was injured. If the neighbor sues the homeowner for damages for his injuries, will he recover? A Yes, because the homeowner's action caused the injury to the neighbor. B Yes, because the homeowner assumed the duty of aiding the neighbor. C Yes, because it was foreseeable that the neighbor would be injured as a result of the homeowner's negligent conduct. D No, because the homeowner's negligence did not cause the injury to the neighbor

D (Actual Cause-Cause in Fact)

Question 4 A state statute required that any freight train operating within the city limits be able to stop within 200 yards of applying its brakes. No fixed speed limit was established or particular type of braking mechanism required, but through either lowered speed or braking power, the 200-yard limit was required of all trains. Another statute prohibited vehicles from being within the railroad crossing when the lights on the warning signs are flashing or when the gates are lowered. One day, as a freight train was entering the city limits, the engineer saw a car stalled at a street crossing ahead. He immediately applied full braking power, but was unable to stop the train before it had hit and demolished the car. The driver of the car had gotten clear before the impact, but brought suit against the freight line for property damage to the $25,000 car. At trial, the parties stipulated that the car was stalled within the crossing while the warning lights were flashing. Evidence at trial established that the distance from the point at which the engineer applied the train's brakes to the point of impact was 150 yards, and from the braking point to the point at which the train finally stopped was 225 yards. No other evidence of negligence was presented by the driver. At the end of the driver's case, the freight line moved for a directed verdict. Should the court grant the motion? A No, because the freight line was negligent per se. B No, because the freight line was strictly liable for its violation of the braking statute. C Yes, because the driver's car was on the freight line's tracks in violation of the crossing statute. D Yes, because the freight line's violation of the braking statute was not the cause in fact of the accident

D (But For Cause in Fact)

Question 1 A college student took his roommate's notebook computer without permission because he needed to write a term paper that was due the next day. While the computer was sitting open on the student's desk overnight, a water pipe in the ceiling began leaking and water dripped down on the computer, rendering it inoperable. A computer repair service estimated that it would cost $500 to repair all the damaged components. At the time it was damaged, the computer was $700. If the roommate sues the student for the damage caused to the computer, what will be the extent of his recovery? A Nothing, because the damage occurred through no fault of the student. B Loss of use damages for the time it was in the Student's possession. C $500 in damages. D $700 in damages.

D (Conversion Damages)

Question 6 The owner of a collection of old anvils lent it to the local museum and hired professional movers to transport the anvils to the second floor of the museum, where they would be displayed. The movers used a rope and pulley apparatus to lift the anvils on the outside of the building to a second-story window. While one of the largest anvils was being lifted, it slipped and fell, crashing to the ground. However, the anvil was not even dented. If the owner brings a negligence action against the movers for allowing the antique anvil to fall, what can the owner recover? A Nominal damages. B Punitive damages. C Both nominal damages and punitive damages. D Neither nominal damages nor punitive damages

D (Damage to Property)

Question 4 A man drove into the parking lot of a bank and was about to pull into an empty spot when a woman cut in front of him with her automobile and took his parking place. The man and woman got out of their cars, and the man started to yell at the woman. After a heated argument, a fight broke out between them. A customer came out of the bank at that moment and saw that the man was getting the better of the woman in the fight. The customer ran to his car, took a gun from the glove compartment, pointed it at the man and said, "Stop this minute or I'll shoot." If the man asserts a claim against the customer based on assault, who will prevail? A The man, because the customer threatened him with deadly force. B The man, because the customer was unaware of who the aggressor was. C The customer, if the man was the original aggressor by starting the fight with the woman. D The customer, if it was apparent that the man was about to inflict serious bodily harm to the woman.

D (Defense of Another)

Question 7 A trainer of homing pigeons brought several of them to a park that he often used for training. He had trained this group of pigeons carefully and was confident that they would readily find their way home. When they were released, one of the pigeons inexplicably turned in the opposite direction from home. Several blocks away at the other end of the park, it collided with a radio-controlled model airplane that its owner had just purchased and was trying out for the first time. The collision sent the airplane out of control; it dipped low across a highway and was struck and run over by a truck. The airplane owner sued the pigeon trainer for the destruction of his airplane. The parties stipulated to the above facts and the airplane owner presented evidence of his damages. The trainer then moved for a directed verdict. Should it be granted? A No, because the trainer's pigeon caused the destruction of the airplane. B No, because the jury could find negligence on the trainer's part under the doctrine of res ipsa loquitur. C Yes, because the truck, rather than the pigeon, was the direct cause of the airplane's destruction. D Yes, because the trainer took reasonable care in training his pigeons.

D (Duty & Non-Dangerous Animals)

Question 4 A state contains major deposits of natural gas. In an effort to support this industry, and at the same time save its citizens substantial sums for the cost of heating their homes and businesses, the legislature enacted a substantial tax on out-of- state suppliers of natural gas. In addition, the state required state- licensed public utilities to buy no less than 75% of their natural gas needs from sources within the state as long as their needs could be met. An out-of-state supplier ("the supplier") brought suit against the state challenging this statute. What is the best constitutional argument that the supplier could make? A The statute violates the Due Process Clause of the Fourteenth Amendment. B The statute violates the Equal Protection Clause of the Fourteenth Amendment. C The statute violates the Privileges and Immunities Clause of Article IV. D The statute violates the Commerce Clause

D Commerce Clause/State Tax - Intermediate

Question 10 A motorist was negligently driving close to the shoulder of a highway when his vehicle skidded and hit a support column of a bridge that crossed over the highway. The impact from the car caused structural damage to the support column, which caused the bridge to drop 18 inches. The sag of the bridge was clearly visible from the highway. The motorist died in his heavily damaged car as a result of the accident. A rescuer, who had been five miles away at the time of the accident, came on the scene and pulled his car off the road to see if he could render assistance. Shortly thereafter, a trucker approached the scene of the accident. The trucker saw that an accident had occurred, and had adequate time to slow down or stop, but he proceeded ahead without reducing speed under ordinary circumstances, his truck could have passed easily under the bridge, but the 18-inch drop caused the top of the truck to strike the bridge. A chunk of concrete fell from the bridge, striking the rescuer in the head and seriously injuring. If the rescuer sues the motorist's estate, who will prevail? A The motorist's estate, because the trucker's actions caused the rescuer's injuries. B The motorist's estate, because the rescuer was five miles away when the initial accident occurred and therefore not a foreseeable plaintiff. C The rescuer, because he stopped to render assistance. D The rescuer, because the motorist's negligence was a proximate cause of the rescuers injuries.

D (Proximate Cause and Cause in Fact; indirect cause case - independent intervening force)

Question 3 On the way home from a nightclub, a passenger began yelling at the designated driver claiming that he was not taking the best route back to her house. The driver disagreed and contended that his route was the quickest. The passenger impulsively grabbed the steering wheel, causing the car to swerve and strike a pedestrian, injuring him. At trial, the pedestrian established that the driver's license had expired the day before the accident. The driver's traffic record qualified him for an automatic renewal of his driver's license, but he had forgotten to submit it in time. A statute in the jurisdiction makes it an offense to drive a vehicle on any public road in the state without a valid driver's license. Will the pedestrian prevail? A Yes, because the driver violated a statute by driving without a valid license. B Yes, because the driver failed to control his passenger. C No, because the driver did not start the argument. D No, because the passenger's action was the proximate cause of the injury

D (Proximate Cause and superseding intervening cause)

Question 8 A small print shop on the second floor of an older two-story industrial building kept its front door locked for security reasons, so most visitors gained access to the shop by using the shop's freight elevator in the rear of the building. This elevator did not have a call-button for use by the public, so anyone wanting to use it had to wait for one of the shop's employees to send down the elevator. A city fire inspector inspecting the building informed the shop foreman that he was going to inspect the lower floor, the trash area, and the elevator shaft. The foreman said that he would turn the elevator off until the inspector called up from downstairs that he was finished with his inspection. About 10 minutes later, as the inspector was under the elevator in the shaft, the freight elevator suddenly stalled descending. The inspector tried to get out of the way, but his leg was severed by the elevator. The inspector sued the print shop for the loss of his leg. At the time of the accident, there were three employees in the shop, but all deny that they pushed the button that would send the elevator to the first floor. Evidence at trial indicated that the foreman had turned the switch to "off" as he agreed to do. The inspector's expert testified that there was nothing wrong with the elevator that would cause it to malfunction and operate when it was turned off. Is the inspector likely to prevail? A No, because the evidence indicated that the foreman had turned the switch to "off." B No, because, as a fire inspector, the inspector was merely a licensee and the foreman took steps to protect him from injury. C Yes, because the foreman must have forgotten to turn the switch to "off." D Yes, because the printing shop's elevator could not be controlled from the first floor

D (Res Ipsa Loquitor & Respondeat Superior)

10. A man borrowed $5,000 from his colleague to purchase stock and agreed in writing to repay the loan on or before August 1. On August l the man notified his colleague that he would be unable to pay back the $5,000. He told her that he could send her a check for $2,500 and that, in addition, he could give her an antique diamond ring that had been recently appraised at $2,200. The colleague liked the ring and agreed to accept it plus $2,500 in cash as payment for the loan. On August 5 a courier delivered the ring and a certified check for $2,500 to the colleague. She took the check but told the courier to return the ring to the man. The man received the ring back the same afternoon. Meanwhile, the colleague deposited the check in her bank, and the next day filed suit against the man for $2,500. The man consulted an attorney as to whether he has a valid defense against his colleague's suit. Assuming there are no Statute of Frauds issues, what advice should the attorney give the man? (A) The man has no defense against his colleague's suit, because the amount of the debt was undisputed. (B) The man has no defense against his colleague's suit, because she properly exercised her right to enforce the original agreement by refusing tender of the ring. (C) The man has no defense at law, but he may successfully defend in equity under a specific performance theory because the ring is unique. (D) The man has the option of defending in equity under a specific performance theory or waiting until his colleague obtains a judgment against him and then suing her for breach.

D - Accord

6. On August l the owner of a hardware store noticed that he was running low on half-inch carriage bolts and their corresponding nuts. He called a screw manufacturer and ordered 1,000 half-inch carriage bolts and nuts to be delivered by August 15. The screw manufacturer e-mailed the store owner a confirmation of the order that same day. On August 15, the 1,000 bolts were delivered, but the nuts were missing. The store owner called the manufacturer and was told that they had been temporarily out of nuts when they had filled his order, and had reduced the amount he owed to reflect this, as they had done in the past with him in similar circumstances. The store owner protested and the manufacturer offered to send the nuts by overnight carrier so that he would get them the next day. May the store owner cancel the contract? (A) Yes, because he was entitled to a perfect tender. (B) Yes, because the time for performance has passed. (C) No, because the one-day delay is not material. (D) No, because the manufacturer has a reasonable amount of time within which to cure.

D - Breach/Perfect Tender Rule/Cure; Seller's Right to Cure

1. A wealthy sportsman purchased a large old wooden sailing ship for $200,000. Although the boat was a classic, the sportsman wanted it to be modernized and made more comfortable. To that end, the sportsman entered into a written contract with a marine architect-engineer to draw up and then execute the modernization plans. At the time the parties entered into the agreement, the sportsman told the architect-engineer that his modernization plan would be subject to the approval of the sportsman's sister, that they would, in fact, have no deal unless the plans meet with her approval. The architect-engineer agreed to this. He finished his drawings and submitted them to the sportsman, who was enthusiastic about the designs. The sportsman's sister, a famous yachtswoman, was engaged in a trans-Pacific yacht race at the time and was not expected home for a number of weeks. Cheered by the sportsman's enthusiasm, the architect-engineer went ahead and modernized the ship according to his designs. When he finished the work, he submitted a bill to the sportsman, who refused to pay, pointing out that his sister had never approved the designs. If the architect-engineer sues the sportsman, which of the following issues of contract law is most likely to be decisive in determining the outcome of the case? (A) Statute of Frauds (B) Parol evidence rule (C) Rules of construction (D) Conditions precedent

D - Condition precedent

8. A woman had a developmentally disabled brother who lived in a group home. The woman ran errands for her brother, took him to the park, and generally made his life pleasant and comfortable. The siblings' grandmother wanted to encourage her granddaughter to continue helping her brother. Therefore, she called her granddaughter and told her that if she continued to take care of her brother for the next five years, the grandmother would give the granddaughter her condominium in Hawaii. The granddaughter continued to take care of her brother. However, two years after their conversation, the grandmother sold her condominium in Hawaii and told her granddaughter that she would not be able to give her the condominium as a gift, as she had promised. The granddaughter continued to care for her brother. If five years after their initial agreement, the granddaughter brings suit against her grandmother for breach of contract, which of the following will not be relevant to the grandmother's defense? (A) The contract involved the transfer of an interest in real property. (B) The contract involved services that could not be performed within a year. (C) The granddaughter was caring for her brother before the agreement was entered into. (D) The grandmother sold the condominium before the end of the five years.

D - Consideration; Promissory Estoppel or Detrimental Reliance; Statute of Frauds

7. A grantor executed and delivered a deed to his daughter conveying his ranch as follows: "To my daughter for life, but if my daughter dies survived by her husband and children, then to my daughter's husband for life, with the remainder in fee simple to my daughter's children; but if my daughter dies survived by her husband and no children, then to my son in fee simple." A few months later, the daughter married; as a wedding gift, the grantor quitclaimed his interest in the ranch to the daughter's husband. Assume that the jurisdiction does not follow the doctrine of destructibility of contingent remainders. A year later, the daughter died without children and without a will. The applicable law of intestate succession provides that the husband is the daughter's only heir. The son claims that the husband has no interest in the land. Title to the ranch is held by whom? (A) The husband, because of the doctrine of merger. (B) The husband, because the daughter died intestate and her fee simple passed to him as her intestate heir. (C) The son, because the interest granted to the daughter's spouse is void under the Rule Against Perpetuities. (D) The son, because the daughter, although survived by her spouse, died without children.

D - Contingent remainders/doctrine of merger

8. A homeowner owned a parcel of land on which she built a single-family residence. When the house was completed, the homeowner leased the house to a tenant for a three-year term. Because there was no dishwasher in the kitchen, the tenant bought a professional-grade dishwasher from an appliance company and had it installed in the space provided around the built-in cabinets in the kitchen. There was no provision in the lease agreement regarding kitchen appliances. To make the purchase, the tenant signed a security agreement with the appliance company granting it a security interest in the dishwasher in exchange for financing. The appliance company did not file or record its security interest in the dishwasher. By the end of the lease term, the tenant was in serious default on his loan payments to the appliance company. The tenant planned to remove the dishwasher and take it with him when he moved out within the next few weeks. The homeowner filed an action against the tenant claiming ownership of the dishwasher and joined the appliance company as a party. Which party has a superior claim to the dishwasher? (A) The homeowner, because there was no objective intention that the dishwasher become a fixture. (B) The tenant, because removal of the dishwasher will not cause substantial damage to the real estate. (C) The homeowner, because the dishwasher was annexed to the real estate and the appliance company never recorded. (D) The appliance company, because it has a valid security interest in the dishwasher even though it was not recorded.

D - Fixtures

4. A homeowner and a contractor duly executed a contract providing that the contractor was to construct a residence on a specified lot. No date was included in the contract for completion of the home. After the contractor completed 5% of the residence, a tornado demolished the construction but left the lot undamaged. Which of the following states the probable legal consequences of the tornado damage? (A) The contract is void because the subject matter of the contract was destroyed through no fault of the parties. (B) The contractor's duty of performance is discharged because of impossibility. (C) The contractor remains obligated to construct the residence, but he is entitled to a quantum meruit recovery for the work done prior to the tornado. (D) The contractor remains obligated to perform under original contract without any compensation for the work done prior to the tornado.

D - Intermediate: Risk of Loss/Casualty; Discharge by Impossibility

10. A large appliance store entered into a written agreement with a television manufacturer for 20 high definition televisions for $300 each. The shipment arrived on time and was paid for with a cashier's check by the appliance store. However, when the appliance store tested the TVs, it found that none of them met the high definition specifications in the contract, even though they functioned fine otherwise. The appliance store notified the manufacturer that it was rejecting the TVs because they were nonconforming, but the manufacturer did not want them returned. What may the appliance store recover? (A) Nothing. (B) The money it has already paid for the TVs, and it gets to keep them. (C) The difference between the value of the TVs and the price it could have obtained for them on the market. (D) The money it paid for the TVs, but it may sell them for the manufacturer's account.

D- Nonconforming Goods Damages

5. After reaching an oral agreement on the terms of representation, a law firm, at its clients' behest, instituted a class action lawsuit against a tobacco company for $100 million. Prior to signing the written contract outlining the parties' rights and responsibilities, including the fee arrangement, the firm's senior partner told the clients' representative in a moment of goodwill and generosity that if they won or the tobacco company settled, he would turn over half of the attorneys' fees in the case to a particular nonprofit group that funds research on lung cancer and other respiratory illnesses. After the law firm won the case and collected its fee of $33 million, it had second thoughts about turning over half of it to the nonprofit group. If the nonprofit group sues the law firm in an attempt to collect the $16.5 million, which of the following is the law firm's best defense to such action? (A) The promise to turn over half of the attorneys' fees was discharged by novation. (B) A partial assignment (in favor of the nonprofit group) is ineffective. (C) The promise to turn over half of the attorneys' fees was not in writing. (D) The law firm was simply attempting to confer a gift upon the nonprofit group.

D - Lack of Consideration

3. A man had rented a woman's home from her for seven years. When the time came to sign a new lease, the woman decided that because the man had always been a quiet tenant, she would continue to charge him only $350 per month rent instead of the $500 to $550 she could probably get otherwise. The new lease was for a period of five years, and by its terms, the man was specifically prohibited from assigning the lease without the woman's specific written consent. About a year later, the man got married and moved into his new wife's home. Instead of giving up his lease, the man sublet the property to a friend for $500 a month. The man did not get the woman's permission to sublease the property. If the woman brings an action to either eject the friend from the premises or to recover damages from the man for subletting the premises without her consent, what is the most likely result? (A) The woman will be able to recover damages and to eject the new tenant. (B) The woman will be able to eject the new tenant only, because she has suffered no money damages. (C) The woman will not be able to eject the new tenant because, although the man did not have the right to sublet, he had the power, but she will be entitled to recover the full rent paid by the new tenant because it would be unfair to let the man profit from his wrongful act. (D) The woman will have no cause of action for either ejectment or damages.

D - Landlord Tenant (Difficult)

8. The owner of a personal watercraft put an ad for its sale in the paper. Her neighbor saw the ad and told her that he wanted to buy the watercraft but had to arrange for financing. The owner suggested that they write a contract for sale then and there so that they would not have to waste any time while he got his financing. They orally agreed that the contract would not become binding unless the neighbor obtained financing, but the written contract did not mention this and appeared to be a fully integrated document. The neighbor could not obtain financing and the brings suit to enforce the written contract. Who will prevail? (A) The owner, because the contract was a fully integrated writing. (B) The owner, because parol evidence is not allowed to contradict a writing. (C) The neighbor, because the oral agreement that the contract would not be binding if the neighbor did not get financing was made contemporaneous with the writing. (D) The neighbor, because obtaining financing was a condition precedent.

D - Parol Evidence Rule- Supplementing, Explaining, or Contradicting Terms

5. The owner of an apartment building contracted with a painter to paint the porches of the apartments for $5,000. The contract was specifically made subject to the owner's good faith approval of the work. The painter finished painting the porches. The owner inspected the porches and believed in good faith that the painter had done a bad job. The painter demanded payment, but the owner told him that the paintjob was poor and refused to pay. The painter pleaded that he was desperately in need of money. The owner told the painter that she would pay him $4,500, provided he repainted the porches. The painter reluctantly agreed, and the owner gave the painter a check in the amount of $4,500. The painter went to his bank, indorsed the check "under protest" and signed his name, then deposited the check in his account. He never returned to repaint the porches. The painter sues the for $500, which he believes is still owed to him on his contract to paint the porches. Will he prevail? (A) Yes, because he indorsed the check "under protest." (B) Yes, but only if he repaints the porches. (C) Yes, because he performed the contract by painting the porches the first time. (D) No, even if he repaints the porches.

D - Promisor's satisfaction as Condition Precedent

8. A farmer executed a valid deed, conveying his orange grove: "To my brother for life, then to my niece and nephew. My brother shall have no right to convey any interest in the orange grove. Likewise, should my niece or nephew attempt to convey any interest in the orange grove prior to the date that their interest becomes possessory, that niece's or nephew's interest shall pass to the other." Subsequently, the brother transferred all of his interest in the orange grove to his friend. Several weeks after the brother's conveyance, the nephew found himself in need of cash and conveyed his interest in the orange grove to a loan shark in exchange for $20,000. The niece brings a suit to quiet title to the orange grove. Who will the court find has title to the orange grove? (A) The niece only. (B) The niece and the loan shark only. (C) The niece and the loan shark, subject to the brother's life estate. (D) The niece, subject to the friend's life estate pur autre vie.

D - Restraint on alienation

10. A developer owned a 100-acre tract of land, which he subdivided into 200 single-family residences. All deeds contained a covenant that the property would be used only for residential purposes. Two years after the last lot was sold, the developer purchased the 50-acre tract of land adjoining the subdivision. The developer constructed another group of 100 single-family residences on this land. To ensure their marketability, the developer desired to sell them like homes in the original development, and to that extent, intended that the deeds to the new homes contain the same restrictions as those in the original development and be enforceable in the same manner by owners in both developments. Will the developer be successful in this plan? (A) No, zoning restrictions will control. (B) No, because in order to have such restrictions, the entire project must have been built at the same time. (C) Yes, the restrictions will be equally enforceable for this new project as they were for the original development. (D) Yes, provided that a court considers this new project to be part of a common development scheme with the original development.

D - Servitudes

2. A large bookstore entered into a written agreement with the manager of a shopping center to rent space that the manager agreed to fix up to suit the bookstore's purpose. The bookstore explained to the manager that classes at the local college begin on August 15 and in order to sell textbooks to those students, the store must be up and running before that date. The contract between the parties provided that the store premises would be ready by August 10 at the latest. The manager immediately contracted with a builder to do the construction work in accordance with the bookstore's requirements. When the premises were not ready by August 10, the bookstore notified the manager that it would rent store space elsewhere. Five days later, on August 15, the manager sent the bookstore a letter by certified mail stating that the construction had been delayed because the builder he had hired had breached his contract by abandoning the job, and he had to employ another builder. However, the manager promised that the premises would be completed no later than August 25. The manager further stated that, under the circumstances, he believed he was entitled to this short period of time to complete the agreement because the renovations, done especially for the bookstore, had already cost him $15,000. On August 25, the manager tendered performance to the bookstore, but the bookstore refused to accept the lease. The manager sought another tenant for the space, without success. The shopping center brought suit against the bookstore for breach of contract. Which of the following facts will the court consider in determining which party will prevail? (A) A substitute tenant who was willing to rent the renovated premises could not be found. (B) The shopping center could not have foreseen that the builder would breach his contract. (C) The shopping center manager's letter of explanation arrived too late. (D) The contract provided that the store premises will be ready by August 10 "at the latest."

D - Time is of the essence clause

4. A developer owned a large urban property, which she subdivided into 10 lots. The developer conveyed Lot 1 to an architect by a deed that contained a restriction banning commercial use of the property. The developer subsequently conveyed Lots 2 through 7 to six separate purchasers. Each of the deeds to these purchasers also contained the restriction on commercial use. The architect left Lot 1 undeveloped, but the purchasers of Lots 2 through 7 all used their lots for commercial purposes. The developer subsequently conveyed Lot 8 to a florist. The florist's deed contained the restriction banning commercial use of the lot, but he decided that he wished to use Lot 8 commercially. The developer retains ownership of Lots 9 and 10. The florist wants to bring suit to establish his rights to use Lot 8 for commercial purposes. Which of the following best describes the parties the florist should join in his lawsuit? (A) The developer only. (B) The developer and the architect only. (C) The other commercial users only. (D) All landowners in the subdivision.

D -Covenants, Servitudes, Abandonment

4. A landowner devised her parcel of land to her daughter, her heirs, and assigns, "so long as the property is used for residential purposes, then to my niece, her heirs, and assigns." The remainder of the landowner's property passed through the residuary clause of her will to her grandson. The daughter lived on the land for 25 years; then, on her death, ownership passed to her husband. In the meantime, the niece had also died, leaving her entire estate to her son. The husband has leased the land to a developer, who has obtained the necessary permits to build a shopping center on it. The grandson and the niece's son both file quiet title and ejectment actions against the husband, and the cases are consolidated. How should the court, applying common law, rule as to ownership of the land? (A) For the husband. (B) For the niece's son. (C) For the grandson, because he received a right of reversion from his grandmother. (D) For the grandson, because he received a possibility of reverter from his grandmother.

D -Future Interests/Rule against perpetuities

Question 10 A woman's boyfriend came to her apartment with a mink coat in his arms and handed it to the woman. After the boyfriend told her that the coat was now hers, the woman asked him where he got the coat. The boyfriend answered, "From the Easter Bunny." After the boyfriend left, she tried on the coat and admired how good it looked on her in the mirror. The next day, the woman read in the newspaper that the home of a well-known socialite had been burglarized the night before. Among the missing items, according to the paper, was a mink coat. The woman took the coat from the closet and rifled through the pockets. She found a handkerchief with the monogram matching the initials of the well-known socialite. The woman decided to keep the fur coat, thinking to herself that the socialite could probably afford to buy another. Which of the following best describes the crime or crimes, if any, the woman has committed? A Accessory after the fact to burglary. B Larceny and accessory after the fact to burglary. C Receipt of stolen property. D The woman has not committed any crime

D Accessory after the fact - Advanced

2. A doctor and a lawyer owned adjoining parcels of land. Ten years ago, the doctor installed a swimming pool on her land. The doctor obtained the lawyer's oral consent to run plumbing from the pool across part of the lawyer's land. Last year, the lawyer sold his land to a buyer. The buyer wants to plant a garden on the land under which the doctor's plumbing runs, and wants to eject the doctor and quiet title. The statute of limitations for ejectment is seven years. With respect to the land under which the plumbing was laid what, if anything, has the doctor acquired? (A) The doctor has acquired title by adverse possession. (B) The doctor has acquired a prescriptive easement. (C) The doctor has acquired both title by adverse possession and a prescriptive easement. (D) The doctor has acquired neither title by adverse possession nor a prescriptive easement.

D Adverse Possession

Question 2 A horse breeder offered to sell a colt to his neighbor and they agreed on a purchase price. The horse breeder subsequently received a letter from the neighbor thanking him for the sale and summarizing their agreement. The letter contained the neighbor's alleged signature. When the horse breeder attempted to set up transfer of the colt, the neighbor denied that she agreed to purchase it. In a breach of contract action against the neighbor, the horse breeder offers into evidence the letter. The horse breeder testifies that he is familiar with the neighbor's handwriting and recognizes the signature on the letter as being hers. Assuming appropriate objection by the neighbor, who claims that she did not sign the letter, how should the trial court rule on the admissibility of the letter? A Exclude the letter for lack of foundation because lay opinion testimony regarding handwriting identification is not admissible. B Exclude the letter unless its authenticity is established by a preponderance of the evidence. C Admit the letter as authentic and instruct the jury accordingly. D Admit the letter but instruct the jury that it is up to them to decide whether the letter is authentic

D Authentication - Intermediate Admit the letter but instruct the jury that it is up to them to decide whether the letter is authentic. The court should admit the letter and instruct the jury that it is up to them to decide whether the letter is authentic. Before a writing may be received in evidence, it must be authenticated by proof showing that the writing is what the proponent claims it is. All that is necessary is proof sufficient to support a jury finding of genuineness. The authenticity of a document is a preliminary fact to be decided by the jury. Here, the horse breeder's testimony that he is familiar with the neighbor's handwriting and that he recognizes the signature on the letter to be that of the neighbor is sufficient to support a jury finding of genuineness. Thus, the letter should be admitted and authenticity should be left to the jury to decide. (A) is wrong because a lay witness who has personal knowledge of the handwriting of the supposed writer may state his opinion as to whether the document is in that person's handwriting. (B) is wrong because authentication of documentary evidence requires only enough evidence to support a jury finding that the matter is what its proponent claims it is. It is not required that the proponent establish its genuineness by a preponderance of the evidence. (C) is wrong because, as noted above, where there is a dispute as to the authenticity of a document, the issue of authenticity is a fact determination for the jury, not the judge, to decide.

Question 9 It was common practice in a particular state for a security interest in land to be structured as a deed absolute, which gave a lender absolute title to the borrower's property as security for the loan. The lender would reconvey only on complete payment of the loan by the debtor party, and could dispose of the land immediately without a foreclosure sale on default. A new governor of the state whose campaign platform was built around abolishing the deed absolute mortgage encouraged the legislature to enact a bill that immediately use of the deed absolute, declaring that all such deeds would be considered mere liens against the secured property. The law applied not only to loans made in the future, but also to the thousands of such loans in existence at the time the legislation was passed. As soon as the governor signed the legislation, lending institutions and individuals who had loaned money secured through deeds absolute challenged the constitutionality of the new law. What is the strongest argument that the challengers can make? A As applied to loans outstanding at the time the bill was enacted, the law is an ex post facto law, and such laws are banned by the federal Constitution. B Lenders using the deed absolute have been singled out by the governor and his followers in the legislature as political scapegoats, and such discrimination against the lenders violates the Equal Protection Clause. C Lenders had property rights in the secured property and such rights were summarily abrogated by the new law, constituting an unconstitutional taking of property without due process of law. D As applied to loans outstanding at the time the bill was enacted, the law impairs the contract rights of the lenders and such rights are guaranteed by the Contracts Clause of the federal Constitution

D Contracts Clause - Advanced

Question 3 A rancher purchased cattle from a breeder. As payment for the cattle, the rancher gave the breeder a promissory note that obligated the rancher to pay the breeder a lump sum of $80,000 18 months after the purchase. During the course of the year that he had the cattle, however, the rancher discovered that the cattle were diseased, so a year after the purchase the rancher filed an action against the breeder in federal district court for breach of warranty, alleging that the cattle were diseased at the time of sale the previous year. The breeder will file his answer within three weeks. May the breeder assert in his answer a counterclaim against the rancher seeking payment of the promissory note? A Yes, the breeder may assert the claim on the promissory note as a counterclaim or he may assert it in a separate action. B Yes, the claim on the promissory note is a compulsory counterclaim because it arises from the same transaction or occurrence as the rancher's initial claim. C No, because it does not arise from the same transaction or occurrence as the rancher's initial claim. D No, because the claim will not yet be mature when the breeder files his answer.

D Counterclaims-Intermediate

Question 2 A husband was very jealous of any contact his wife had with other men, and was particularly suspicious of his wife's relationship with their neighbor, a plumber. Early one morning, the shower in the couple's master bathroom sprang a leak while the wife was getting ready for the day. Fearing permanent damage to their house, and needing to get the repair done quickly, the wife quickly threw on some clothes and called the neighbor. The neighbor immediately went over to the couple's home, and went to the master bathroom. At the same time, the husband unexpectedly came home because he had forgotten something for work. The husband went to the master bedroom and saw the neighbor and his wife with her hair wet and clothes hastily put on. Enraged, he ran to his study, grabbed his gun from the desk drawer, and shot and killed the neighbor. The jurisdiction defines murder in the first degree to include premeditated and deliberate killings and all killings that take place during the commission of a dangerous felony. Premeditation and deliberation is defined as requiring some meaningful reflection prior to the killing. All other common law murder is classified as murder in the second degree. If the jury finds that the husband was unreasonable in his erroneous belief that the neighbor and his wife were together for the purpose of adultery, what crime did the husband commit when he killed the neighbor? A Voluntary manslaughter B Involuntary manslaughter C First degree murder D Second degree murder

D Crime of Passion- Intermediate

Question 8 An inventor sued a rival in federal court for violating the inventor's patent. At a deposition attended by lawyers for both parties, the rival gave convincing testimony that his product was completely different from that of the inventor. Just before trial, the rival began serving a four-year sentence in the state penitentiary. The rival's lawyer considered the inventor's suit to be a nuisance action, and was certain that the rival's deposition testimony was all that she would need to introduce at trial to win the case. The inventor's lawyer objects to having the deposition read into evidence, arguing that a party's deposition may not be used in this way. Will the court allow the deposition to be used? A No, because imprisonment does not make a party "unavailable." B No, because the rival is a party. C Yes, because the deposition of a party may be used for any purpose. D Yes, because the rival is unavailable.

D Deposition-Intermediate

Question 8 Intending to encourage long-time resident aliens to become American citizens, a state passed a law denying numerous state and municipal jobs to persons who had been resident aliens for longer than 10 years. Those already in the state had to apply for American citizenship within a year after the law took effect. Persons who had acquired resident alien status prior to achieving the age of majority had until age 30 to acquire such status or be automatically disqualified from obtaining such a job. A 40-year-old man who has been a resident alien in the state for 15 years applied for a job as a police emergency response telecommunications expert. He had not filed for citizenship within the one-year grace period. May the state constitutionally rely on the statute to refuse to hire the man? A Yes, because a police department performs an integral governmental function and the state law does not discriminatorily classify resident aliens by race or ethnicity. B Yes, because aliens are not entitled to the privileges and immunities of state citizenship. C No, because the law does not apply equally to all aliens. D No, because the reasons for application of the law to the man do not appear compelling.

D Equal Protection - Advanced

Question 1 A police officer often visited an art gallery on his lunch break and became friends with one of the gallery's salespersons, whom he found to be honest and forthright. After hearing rumors that the gallery's owner sometimes dealt in stolen artwork, the officer decided to investigate. Acting without a warrant, he snuck into a back room and discovered various crates addressed to and from a country that was a well-known conduit for stolen artwork. The officer questioned his salesperson friend about the crates. She told the officer that she never saw what was inside any of the crates, but that she assumed they contained artwork for the owner's private collection that he maintained at home. The officer obtained a search warrant for the owner's home based on the foregoing information. Upon executing the warrant, the officer found several pieces of stolen artwork at the owner's home. The owner was subsequently tried for receiving stolen property. The salesperson was to be called to testify about what she knew about the gallery owner's activities related to the procurement of artwork. Is the salesperson's testimony admissible? A No, because it is the fruit of the poisonous tree. B No, because she is an unindicted co-conspirator. C Yes, because she is unindicted and therefore has no privilege against compelled self-incrimination. D Yes

D Exclusionary Rule, Fruit of Poisonous Tree - Intermediate

Question 10 An insurance company held a $500,000 fire insurance policy on a warehouse located in State A that recently burned down. Three different parties claimed all or part of the policy proceeds. The insurance company filed an action under the Federal Interpleader Statute, naming the three different parties as defendant claimants. The insurance company is incorporated in State B and has its principal place of business in State A. The three claimant parties are citizens of different states: State A, State C, and State D. The insurance company filed the action in State D, and the State A claimant moves to dismiss the case for improper venue. Should the court grant the State A claimant's motion? A Yes, because State A is the only proper venue. B Yes, because venue is proper where a substantial part of the property that is the subject of the action is situated. C No, because venue is proper anywhere in the United States. D No, because venue is proper where any claimant resides.

D Federal Interpleader Statute- Advanced

Question 9 A woman elected to a state legislature is prosecuted in federal court for violating the civil rights of a member of the janitorial staff of the state legislature's building in connection with a sexual assault that occurred in her office. The woman raises as a defense the claim that the allegedly wrongful acts were committed in the course of legislative business, and she is immune from scrutiny by the federal courts. Which of the following is the strongest constitutional argument in favor of the woman's defense? A The application of federal civil rights statutes to state legislators would violate the Due Process Clause. B A federal court must follow state law respecting the scope of legislative immunity. C State legislators enjoy the protection of the Speech and Debate Clause of the United States Constitution. D Because of the doctrine of federalism, federal law generally cannot be applied to state legislators acting in the course of their official duties.

D Federalism/ Intergovernmental Immunities-Advanced

Question 5 A pedestrian sued the driver of a car that hit him, alleging that the driver ran a stop sign. The driver denies this, maintaining that the pedestrian carelessly darted into the street. At trial, the driver calls her husband to testify for her. The husband offers testimony that the pedestrian invariably looks both ways before crossing the street. The driver objects to the admission of this evidence. How should the court rule on the driver's objection? A Sustained, because there is no evidence to corroborate the husband's testimony. B Sustained, because it seeks to prove conduct in conformity with the character evidence. C Overruled, because the pedestrian's character for exercising due care in issue. D Overruled, because it tends to establish the pedestrian's habit

D Habit as Character Evidence - Intermediate

Question 3 A defendant was on trial for burglary, and he took the stand in his own defense. On direct examination, the defendant vigorously denied having committed the burglary. Also on direct examination, the defense attorney asked the defendant questions about his employment history in an attempt to portray him to the jury as a "solid citizen" who would not commit a burglary. The defendant stated that his last regular employment was as a bookkeeper for a corporation. On cross-examination, the prosecutor asked the defendant if he had embezzled funds from the corporation. The defendant denied that he had embezzled from the corporation or from anyone else. The prosecutor then wanted to call a police officer to the stand to testify that when she arrested the defendant for embezzlement, the defendant admitted to the officer that he had embezzled money from the corporation. Assuming that the defendant has not yet been tried on the embezzlement charges, may the prosecutor call the officer to the stand? A Yes, but only for purposes of impeachment. B Yes, both for impeachment of the defendant and as substantive evidence. C No, because the defendant has not yet been convicted of embezzlement. D No, because the evidence would be extrinsic

D Impeachment by Extrinsic Evidence - Intermediate

Question 3 Congress passed a law allowing widespread oil exploration on federal lands in the western United States. A large deposit of oil sand was discovered in one western state and Congress authorized an oil sand refining plant to be built on federal park land within the state. The refinery was built in compliance with federal pollution regulations. Pursuant to state law, the plant manager allowed the state to inspect the plant before putting it into operation. Because state refinery standards were more strict than the federal standards (in order to better protect state citizens from pollution associated with refineries), the refinery did not pass the inspection, and the state inspector refused to give the manager a permit to run the refinery. The refinery manager nevertheless began to run the refinery and was fined by the state. Which of the following is the manager's best defense against imposition of the fine? A The state does not have a compelling interest in regulating the refinery, because it is within a federal park. B The state regulation is invalid because Congress has preempted the field of pollution control. C The state pollution regulation is invalid because it is inconsistent with the state's compelling interest in providing jobs. D The state law violates the principles of intergovernmental immunity as applied to the Manager

D Intergovernmental Immunities - Intermediate

Question 2 A student activist at a state-supported university filed a defamation suit in federal court against the school, seeking a cease and desist order. The suit alleged that several university officials had been making statements to the local press about the student, falsely accusing her of both bizarre sexual activities and former felonious crimes. In addition, the student alleged that she had found herself generally harassed on campus by university personnel and that the university's actions against the student were intended to silence her criticisms. Within a week after the suit was filed, the defamations and harassments ceased. When the case came before the federal court four months later, both sides stipulated that the defamations and harassments had ceased. What action should the district court take? A Dismiss the suit for lack of standing. B Dismiss the suit for mootness. C Dismiss the suit, because the case is not ripe. D Hear the case on the merits

D Judicial Review - Intermediate

Question 8 A state adopted a law requiring that all clothing sold within the state be treated with fire-retardant material. The law empowered a state agency to issue rules specifying the required treatment. A manufacturer of clothing that sold its products in the state brought suit in federal court for a declaratory judgment as soon as the law was adopted, alleging that the statute violated the United States Constitution. What should the court do? A Entertain the suit, because a federal question is involved. B Entertain the suit, because an action for declaratory relief is a proper method of deciding constitutional questions. C Abstain from the assertion of jurisdiction until a state court has the opportunity to construe the statute. D Dismiss the suit.

D Judicial Review - Ripeness - Advanced

Question 9 A town with a population of 30,000 merged with a city of 60,000. To protect voting rights of the citizens of the former town, a proposal was made that for a period of 20 years, beginning at the date of the merger, the city council of the merged city would consist of six persons. Each formerly separate municipality would be divided into three council districts. Each district from the former town would have approximately 10,000 residents, and each district from the former city would have 20,000 residents. A mayor would be elected at large. Before this proposal was placed on the ballot, the state attorney general issued an advisory opinion stating that the proposal was not in violation of any state statutory or constitutional provision. The proposal was placed on the ballot and was carried by large majorities in both the town and the city, and the districts were carved out. Three taxpayers filed suit to enjoin the holding of an election with council districts of such disparate proportions. The suit reached the state supreme court, which ruled that the governmental formula was constitutional under both the state and United States Constitutions. The plaintiffs wish to take the case to the United States Supreme Court. How should the Supreme Court proceed? A Rely on the attorney general's opinion and not hear the case on its merits. B Not hear the case, because it was decided below on an independent state ground. C Not hear the case, but remand it to federal district court. D Hear the federal issues involved, but decline to rule on state issues

D Judicial review - Advanced

Question 1 The plaintiff corporation is incorporated in State A and has its principal place of business in State B. The defendant corporation is incorporated and has its principal place of business in State C. It also has branch offices and does substantial business in all 50 states. The cause of action is based on a breach of contract that was to be performed in State D, which has a long arm statute authorizing jurisdiction in such cases. In which of these states may the plaintiff corporation bring its action? A In State C only, because that is the only state in which the defendant corporation is at home. B In State D only, because that is where the cause of action arose and is thus the only proper venue. C In States A or B only, because those are the only two states in which the plaintiff corporation resides for venue purposes. D In States C or D only, because only those two states have personal jurisdiction and proper venue.

D Long Arm Statute - Corporation- Intermediate

Question 10 The jurisdiction divides murder into degrees, with all murders being second degree murder unless the prosecution can prove premeditation and deliberation, in which case the killing would be first degree murder. The jurisdiction also uses the M'Naghten rule for insanity. A physician prescribed an experimental drug for the defendant's severe allergies. The physician told the defendant that the medication was experimental, but failed to inform the defendant that the manufacturer had reported a small risk that the medicine caused severe delusions. After taking the medicine the first day, the defendant began to believe that his next door neighbor was spreading false rumors about him to a few of his other neighbors. While still under the influence of the medication, the defendant grabbed a knife from the kitchen, went to his neighbor's house, and rang the doorbell. When the neighbor the door, the defendant plunged the knife into his neighbor's chest, killing him instantly. May the defendant be convicted of first degree murder? A No, because the defendant was temporarily insane under the M'Naghten rule. B No, because the defendant's intoxication was involuntary. C Yes, because the defendant's intoxication was voluntary. D Yes, because the defendant murdered his neighbor without justification and with premeditation

D M'Naghten Insanity - Advanced

Question 8 A pedestrian sued a driver for personal injuries in federal court, properly invoking diversity of citizenship jurisdiction. In the complaint, the pedestrian alleged that the driver ran a red light and struck the pedestrian while the pedestrian was in the crosswalk. Concurrent with the accident, a police report was prepared on which the name and address of a witness to the accident was listed, but neither party requested a copy of the report from the police department. Thus, when the pedestrian submitted an interrogatory to the driver for the names and addresses of persons with knowledge of the accident known to the driver, the driver truthfully omitted the name of the witness. When asked during his deposition whether he knew of any witnesses, the driver again truthfully answered "l don't know of any." At trial the jury found for the driver. In one of the special interrogatories answered by the jury, the jury found that the driver had the green light and that the pedestrian was crossing against the light. Six months and a day after a final judgment was rendered in favor of the driver, the pedestrian's attorney was contacted by the witness, who stated that the driver ran a red light and that the pedestrian had the "walk" sign when he attempted to cross the road. The pedestrian's attorney immediately moved for relief from judgment based on newly discovered evidence, and the trial judge granted the motion. On appeal the court should: A Affirm the trial judge's decision, because whether to deny or grant a motion for relief from judgment is strictly a matter within the trial judge's discretion. B Affirm the trial judge's decision, because the driver should have discovered the identity of the witness. C Reverse the trial judge's decision, because more than six months had passed since the judgment was rendered when the trial judge granted the motion. D Reverse the trial judge's decision, because the pedestrian could have discovered the witness's identity with reasonable diligence.

D Motion for a New Trial-Intermediate

Question 9 A critically acclaimed movie that had received a number of awards opened in a small town. The film had portrayals of nudity and scenes involving sexuality, but its advertising was very tasteful and concentrated on its critical acclaim and its receipt of seven Academy Award nominations. Nevertheless, when the movie opened in the small town, there was a public outcry against it, including picketing. The town, which had been founded in the late nineteenth century by a fundamentalist religious group, remained very conservative and highly religious, and was the only community in the state where a consensus of the community would find the movie to be obscene. The town prosecutor went to the local court seeking an injunction to halt the showing of the movie. The theater owner refused to voluntarily stop showing the film and appeared in court to defend against the proposed injunction. What is the owners best defense? A The proper "community standards" should be those of the entire state rather than of the town. B The film has some redeeming social value. C The Establishment Clause of the First Amendment prevents the state from enforcing a particular set of religious beliefs. D The film has proven artistic merit

D Obscenity/Artistic Merit - Advanced

Question 9 A citizen of State A filed a complaint alleging negligence by two parties, a citizen of State B who lives in the District of State B, and a citizen of State C who lives in the District of State C. The complaint was filed in the United States District Court of State A. However, the accident took place in State D, the plaintiff was treated by an emergency room physician in State D, and all of the witnesses reside in State D. Neither defendant has ever been to State A. Two months after they filed their respective answers, which did not address any problems with personal jurisdiction or venue, the defendants filed a joint motion to dismiss the case for lack of personal jurisdiction, or, in the alternative, to transfer the case to the District of State How should the court rule? A Dismiss the case, because it would be unconstitutional for the State A court to issue a ruling against either defendant for lack of minimum contacts. B Dismiss the case, both because it would be unconstitutional for the State A court to hear the case for lack of minimum contacts, and because the District of State A is an improper venue. C Deny the motion in its entirety, because the defendants have consented to personal jurisdiction and venue by not raising these issues in a timely manner. D Grant the motion in part and transfer the case to the District of State D in the interests of justice.

D Personal Jurisdiction & Venue- Advanced

Question 7 The plaintiff, a citizen and resident of the Southern District of State A, filed a negligence action against a citizen and resident of Canada, alleging that the Canadian resident caused the accident in which the plaintiff was injured while driving in the Western District of State A. The action was filed in the United States District Court for the Southern District of State A. The defendant filed a motion seeking to dismiss the action on the grounds of improper venue and lack of personal jurisdiction. How should the court rule? A Dismiss the action, because the Southern District of State A is not a proper venue. B Dismiss the action because the court lacks personal jurisdiction over the Canadian defendant. C Exercise its discretion by transferring the action to a proper venue rather than dismissing it, even though the Southern District of State A is not a proper venue. D Deny the motion to dismiss because it has personal jurisdiction over the Canadian defendant, and the Southern District of State A is a proper venue.

D Personal Jurisdiction - Advanced

Question 1 The Federal Communications Commission ("FCC") issued a lengthy set of regulations regarding personal radar detectors. The regulations deal with the safety of such detectors and the frequencies on which they may operate, so as not to interfere with FCC-licensed radio and television stations or with radar used by commercial airliners and private aircraft. May a state constitutionally ban the use of radar detectors on its roads? A No, because the regulation of radio transmissions is within the purview of the FCC rather than the states, and state laws that attempt to regulate devices such as radar detectors are preempted. B No, because such a ban would burden interstate commerce. C Yes, because such a ban would operate only within the state. D Yes, because the state has a legitimate interest in regulating the use of radar detectors in order to promote safe driving

D Preemption - Intermediate

Question 10 A state statute makes criminal "all speech-making, picketing, or public gathering of any sort on the steps of the supreme courthouse Monday through Friday, between the hours of 8:30 am and 4:30 pm, when court is in session." A citizen is upset about a supreme court decision that was just released and stands on the steps of the courthouse at noon, while court is in session, handing out leaflets and exhorting passersby to vote the state supreme court justices out of office. If the citizen is prosecuted for violation of the statute, which of the following best describes the applicable burden of proof? A The state will have to show that there was a compelling need for the statute and that no less restrictive alternatives existed to meet that need. B The state will have to show that the statute was narrowly tailored to serve an important government interest and leaves open alternative channels of communication. C The citizen will have to show that there was no compelling need for the statute and that less restrictive alternatives were available to accomplish the same goals. D The citizen will have to show that there was no reasonable basis for enacting the statute

D Public Forum- Advanced

Question 6 An insured purchased a life insurance policy on his life, naming his brother as beneficiary. Fifteen years ago, the insured traveled overseas on what was supposed to be a six- month trip, but has not been heard from since. The brother contacted the insurance company, which refused to pay the claim on the basis that there was no evidence that the insured was dead. The brother filed suit against the insurance company to collect the proceeds under the policy. The jurisdiction in which the action has commenced has a statute that states that a person is presumed dead if missing from the jurisdiction for seven years, and if no one in the jurisdiction has heard from the person in those seven years. Assume that no other evidence is admitted at the trial on the issue of the insured's death. Which of the following is the most accurate statement? A The jury will be permitted to find that the insured is alive. B The jury will be permitted to find that the insured is dead. C The judge must rule as a conclusive presumption that the insured is dead. D The jury must find that the insured is dead

D Rebuttable Presumptions - Advanced

Question 3 The defendant was involved in a three-car accident with two other drivers. The first driver's car was totaled and had an estimated replacement cost of more than $75,000. The second driver's car was also totaled, and it had an estimated replacement cost of $30,000, and her medical bills were $50,000. The first driver sued the defendant in federal court and served a request for admission on him, asking that he admit he was exceeding the posted speed limit by 50 miles per hour. The defendant admitted that fact. Thereafter, the second driver also sued the defendant in federal court in a separate action. May the second driver use the admission by the defendant in her case? A Yes, because the admission is not considered to be hearsay. B Yes, because the mutuality requirement for issue preclusion has largely been eliminated. C No, because the issue was never actually litigated. D No, because admissions may be used in the current litigation only.

D Request for Admissions-Advanced

Question 1 A worker was fired by her employer. The worker filed an action in federal district court, alleging that she was fired on the basis of race. The employer has an internal memo that suggests that race in fact was the impetus for the worker's firing. Must the employer provide a copy of the memo to the worker as part of the employer's required initial disclosures? A Yes, because the employer must produce all documents in its possession that are relevant to any claim or defense in the action. B No, but the employer must provide the worker a description of the memo, and the worker may obtain a copy using a request for production of documents. C No, because the employer must produce in its required initial disclosures only information about witnesses having discoverable information. D No, because the employer must produce in its required initial disclosures only documents and other information that it may use to support its allegations and defenses in the action.

D Required Initial Disclosures

Question 5 A citizen of State A ran a red light at a traffic intersection, striking a pedestrian, a citizen of State B, who was lawfully in the middle of the crosswalk at the time. The accident occurred in State B. Despite the pedestrian's extensive injuries, the driver was more concerned with the damage to his new automobile. The driver sued the pedestrian in a state court of State B for $90,000 for damage to his car. The driver promptly lost, and then decided to bring suit in federal court, claiming diversity jurisdiction. May the driver successfully bring his claim in federal court? A Yes, because the requirements for diversity jurisdiction are met. B Yes, because a state court decision does not bind a federal court. C No, because all claims merged with the state court case. D No, because the claim is barred.

D Res Judicata (Claim Preclusion)/Merger & Bar -Intermediate

Question 4 A city's water board election laws provide that, although members of the board are elected at large, one member of the board is required to live within each of the five designated water districts within the city. The city's population was more or less evenly distributed among the districts when this election law was enacted. A resident and registered voter of the city investigated the district residency requirement and discovered that most of the city's newer residents had moved into the same two water districts, so that the city's population was no longer evenly distributed among the five water districts. Instead, 80% of the city's residents lived within its central and eastern water districts, while the other 20% of the city's residents were scattered among its three other, more rural, districts. If the resident files suit in federal court challenging the constitutionality of the residency requirement, how will the court most likely rule? A The residency requirement is unconstitutional because it impairs the voters' equal protection rights, in that it gives the voters in the less populous districts more effective representation on the water board. B The residency requirement is unconstitutional because it violates the candidates' equal protection rights. C The residency requirement is constitutionally permissible because the water board members do not exercise legislative power. D The residency requirement is constitutionally permissible because the water board members are elected at large

D Residency requirement for elections/Voter's rights- Intermediate

Question 9 A defendant who was indigent was charged with a crime for which the maximum punishment is six months in prison and a fine of $500. At a hearing, the defendant told the judge that he wanted to plead not guilty and that he wished to represent himself. The judge told the defendant that the court would appoint an attorney to represent him if he needed counsel. He further explained that the state has a policy of appointing private attorneys to defend indigents, and that if such defendants are acquitted or imprisoned, there is no charge for the court-appointed lawyer. If, however, the defendant is sentenced to probation, the defendant must pay 'reasonable attorneys' fees,' which it is presumed the defendant will be able to pay out of job earnings while on probation. The defendant told the judge that paying for an attorney would be difficult, and that he still wished to defend himself. The judge believed that the defendant was competent to defend himself, but nevertheless appointed an attorney with criminal defense experience to defend him. The defendant fully cooperated with the attorney, who did a highly competent job, but the evidence heavily favored the state. The defendant was convicted, but the attorney's plea for leniency was effective, and the defendant received a suspended sentence and probation. Two weeks later, the defendant received a bill for $500 for legal services, a figure that represented about half the sum a lawyer not appointed by the court would have charged for similar work. Although the defendant would be able to pay the bill over a long period of time via installments, he was angry that he had been billed at all, and believed that he could have gotten probation if he had been allowed to argue his own case. He consulted another attorney, and asked her to appeal both his conviction and the imposition of the legal fees. If the second attorney files an appeal, what is the appellate court most likely to do? A Affirm both his conviction and the imposition of fees, because there is no reversible error, since the trial attorney competently represented the defendant and the state has a right to recoup costs from those able to pay. B Affirm the conviction because there was no reversible error, but reverse the imposition of fees, because the defendant could have gotten probation for himself as easily as the attorney did. C Reverse the conviction, because the defendant was denied the right to defend himself, but affirm the imposition of fees because the attorney was competent and the state has a right to recoup costs from those able to pay. D Reverse both the conviction and the imposition of fees, because the defendant was denied the right to defend himself

D Right to counsel - Pro Se Representation -Advanced

Question 6 An attorney in one state represented a plaintiff for personal injuries sustained due to the negligence of the defendant, which occurred on January 20. The statute of limitations for the plaintiff's cause of action is governed by the law of a different state. That statute of limitations is only three months. In other states, the statute of limitations for similar causes of action ranges from two years to five years; in the attorney's home state it is three years. The plaintiff first went to see the attorney on June 15, after the statute of limitations had expired. The attorney, based on his experience in other states, believed that the case was well within the statute of limitations. He drafted a complaint, signed it, and filed it in federal court on June 20. The case was immediately dismissed based on the fact that the statute of limitations had expired May the attorney be sanctioned? A No, because he did not know that the plaintiff's case was not warranted by existing law. B No, because he could have argued that the statute of limitations should be reversed. C Yes, because the attorney's signature makes him strictly liable for any defects in the complaint. D Yes, because he should have known that the plaintiff's case was not warranted by existing law.

D Sanctions-Intermediate

Question 4 A police officer was given a tip about a blonde male living in a nearby trailer park who was selling narcotics. The officer immediately drove to the trailer park and obtained from the manager the names of six blonde males who had trailers or mobile homes in the trailer park. At the first lot, the officer knocked on the defendant's door, announced that he was a police officer, and asked to talk to the defendant. The defendant's girlfriend, who did not live there but had been visiting, told the officer that the defendant would not be back for some time. The officer, believing that the girlfriend lived there, told her that he suspected that the defendant was dealing drugs and asked her if he could look around a little. The girlfriend said, "Sure, why not?" and let the officer in. After seeing nothing in the main living area, he went into the small back bedroom and opened several small storage compartments. In the corner of one of the compartments, he found an opaque bag. On opening it, he observed that it contained what appeared to be marijuana and confiscated the bag. Shortly thereafter, the defendant was arrested and charged with possession of narcotics with intent to distribute, a felony. On a motion by the defendant's attorney to suppress the introduction of the marijuana into evidence, how is the court likely to rule? A For the defendant, because his girlfriend did not live in the trailer. B For the defendant, because the search exceeded the scope of the consent. C Against the defendant, because mobile homes fall within the automobile exception to the warrant requirement. D Against the defendant, because the officer reasonably believed that the defendant's girlfriend lived in the trailer

D Search by Consent - Intermediate

Question 5 At a waterfront bar, a college student sought to provoke a fight with a merchant seaman by making insulting remarks. Eventually the seaman had had enough and threw a punch that connected to the student's jaw and sent him sprawling to the floor. The seaman then told the student that he wanted no further trouble. Getting up off the floor, the student pulled a knife out of his pocket and charged at the seaman. Three other students were standing between the seaman and the exit door. The seaman tried to dodge, but was cut on the forearm by the student's knife. The seaman immediately drew a gun and shot the student, killing him. The seaman was charged with murder. Which of the following points raised in the seaman's defense will not be helpful for his defense? A The student had no reason to fear serious bodily injury when he drew the knife. B The student's drawing of the knife constituted an escalation of the fight. C Three college students were standing between the seaman and the door, so there was no clear route of retreat. D The student's comments were motivated by a desire to provoke the seaman

D Self-defense, Provocation & Escalation - Intermediate

Question 6 A state set up an intrastate message routing system to carry messages to and from the various state agency offices located throughout the state. This proved to be cheaper and more efficient than the United States Postal Service. The message service worked so well that the state offered the messenger service to its employees as a fringe benefit. Moreover, it expanded delivery options beyond state offices to any address in the state and permitted the employees to use the service for personal correspondence as well as for official business. Are the state's actions constitutional? A Yes, because the messenger service operates entirely within the state borders. B Yes, because the Commerce Clause does not prohibit states from acting as a market participant. C No, because the Equal Protection Clause prohibits this singling out of state employees for special benefits. D No, because it violates the federal postal monopoly

D Separation of Powers- Advanced

Question 5 A woman was the subject of a murder investigation. The investigation continued for more than tm years, with the woman frequently being called in for questioning. Finally, the woman was indicted for the murder. The woman's lawyer filed a motion to dismiss all charges against her, arguing that the excessively long investigatory period violated the woman's constitutional right to a speedy trial. Despite the pending motion, the woman decided that she wanted to "get it over with," and she told the judge that she wished to plead guilty. The judge then explained the charges to the woman and asked her if she understood them. She replied, "Yes." The judge then asked the woman if she understood that she was not required to plead guilty. She responded in the affirmative. Finally, the judge described the maximum sentence and asked the woman if she understood that she could receive the maximum sentence, which was life imprisonment. She again responded, 'Yes," and maintained that she still wished to plead guilty. The judge accepted the woman's plea and sentenced her to 30 years' imprisonment in the state penitentiary. Six months later, the woman filed a motion to set aside the guilty plea. Which of the following provides the best argument that the woman has a constitutional basis for relief? A The judge did not rule on the pending motion to dismiss before accepting her guilty plea. B The judge did not attempt to determine if the woman had actually committed the murder. C The judge did not determine whether the files in the prosecutor's office contained any undisclosed exculpatory evidence. D The judge did not determine whether the woman understood that she had a right to a trial by jury

D Set Aside a Guilty Plea. 6th Amendment Right to Jury Trial- Intermediate

Question 6 When the defendant learned that his former wife, to whom he was paying $l ,000 per month in alimony, was dating someone else, the defendant encouraged her to get married "for the sake of the children." The former wife said that she would consider it, but she also expressed concern that her boyfriend might already be married. The defendant told his former wife that he would have an acquaintance run a computer check on the boyfriend that would reveal whether he was currently married. However, the defendant did not bother with the computer check; instead he called the boyfriend and offered him $5,000 if he would propose to the defendant's former wife. The defendant then told his former wife that, according to official records, the boyfriend was single. The defendant's former wife and the boyfriend went through a wedding ceremony shortly thereafter. The boyfriend, however, was already married to someone else, a fact that would have been disclosed by a routine check of official records. If the defendant is charged with being an accessory to bigamy, a strict liability offense in the jurisdiction, should he be found guilty? A Yes, because the defendant aided, abetted, and encouraged the marriage. B Yes, because even though the defendant did not know that the boyfriend was married, bigamy is a strict liability offense. C No, unless the defendant's action in failing to check the records was a breach of duty to his former wife. D No, because the defendant did not have the mental state necessary for aiding and abetting

D Strict Liability Bigamy Accomplice-Advanced

Question 8 A Delaware corporation with its business headquarters in Illinois conducts bar preparation courses for law school graduates. In addition to its national headquarters, the corporation rents local administrative headquarters in at least one city in each state. Its instructors are law school professors and other lawyers who are paid an hourly, or occasionally biannual, fee, but each instructor is considered an independent contractor and prepares his or her own lecture outlines and respective study outlines that are provided to students. Some instructors lecture only in one or more cities in the same state, while other instructors lecture in cities throughout the United States. A professor who is a resident of the state of Virginia gives lectures on Constitutional Law in more than 20 states, including California. The printer that publishes all of the bar preparation company's instructional materials is located in New York and ships the materials by commercial carriers to the respective state offices of the bar preparation company. The printer does not solicit business and does not have officers or agents outside of New York. Which of the following taxes is most likely valid? A A Virginia income tax on all of the Virginia professor's income. B An Illinois income tax on all of the Virginia professor's income. C A New York use tax on instructional materials shipped to and sold in California. D A California ad valorem property tax on each item of instructional material received in that state

D Taxation - Advanced

7. A rancher entered into a written contract to buy a farm from a farmer for $100,000. The contract stipulated for closing on September 30. In addition, the contract contained the following provision: "The taxes shall be prorated as agreed to by the parties at a later date." Upon the signing of the contract, the rancher gave the farmer a check for $10,000 as a down payment. On September 28, the rancher notified the farmer that he would not be able to close on the farm until October 2, because the closing on his current home, the proceeds from which were to be applied to his purchase of the farm, was unavoidably delayed due to his buyer's illness. Meanwhile, the farmer had difficulty finding a home she liked as well as the farm. She decided that she would rather not sell the farm and wished to avoid the contract with the rancher. On October 2, the rancher showed up at the closing with the $90,000 to tender to the farmer. The farmer did not show up. The rancher sues for specific performance. In whose favor will the court most likely rule? (A) The farmer, because the tax provision is an essential term of the contract, and it is not specific enough to satisfy the Statute of Frauds. (B) The farmer, because the rancher materially breached by not tendering performance on September 30. (C) The rancher, because of the operation of the doctrine of equitable conversion. (D) The rancher, because time was not of the essence.

D Time is of the essesnce

Question 8 The defendant was arrested, given Miranda warnings, and charged with burglary. At the police station, he telephoned his mother and asked her to come to the station to post bail. Instead, his mother immediately called the family attorney. In the meantime, the police had begun questioning the defendant. Although he never told the police to stop the questioning, his answers were at first vague or clearly unresponsive. During the course of the questioning, the family attorney phoned the station and told the police that she had been hired to represent the defendant and would be there in half an hour. The police did not inform the defendant of the attorney's call. Ten minutes later, the defendant admitted to committing the burglary, and signed a statement to that effect prepared by the police. The attorney arrived a few minutes later and advised the defendant to remain silent, but he told her that he had already signed a confession. How should the court rule on the attorney's pretrial motion to exclude the confession as evidence at trial? A Grant the motion, because the police had a duty to inform the defendant that an attorney was coming to represent him. B Grant the motion, because the defendant has been deprived of his Sixth Amendment right to counsel. C Deny the motion, because the defendant's statement admitting the crime was voluntary. D Deny the motion, because the defendant waived his Miranda rights

D Valid Miranda Waiver - Advanced

8. A landowner and his neighbor own adjoining properties. The landowner decided to build an addition on his home. Prior to construction of the addition, surface waters flowed across the neighbor's property to the landowner's property and then to a drainage ditch. After the addition was completed, surface waters no longer flowed to the ditch but accumulated on the neighbor's property. The standing water attracted so many mosquitoes that the neighbor was unable to enjoy her backyard. If the neighbor prevails in a lawsuit against the landowner, it will most likely be because the jurisdiction follows which legal doctrine? (A) Riparian doctrine. (B) Prior appropriation doctrine. (C) Common enemy theory. (D) Natural flow theory.

D, Nature of Riparian Right

9. A developer subdivided his rural parcel of land into 10 lots and rented them out to tenants. To supply the domestic needs of all 10 homes, the developer drilled a well on the land. The well provided an adequate water supply and none of the tenants ever had reason to complain of water shortages. Subsequently, the owner of property adjacent to the developer built a home on her property, drilling a well to supply her water needs. Although all of the owner's water usage was domestic, she drew a large quantity of water from her well. Six months later, the well on the developer's land ceased producing enough water to adequately supply his tenants. Both wells draw percolating water. The jurisdiction applies the reasonable use doctrine in determining rights to underground water. If the developer sues the owner, asking that the court enjoin her from interfering with the developer's supply, what is the court likely to order the owner to do? (A) Cut back her water use sufficiently so that the developer's tenants can be adequately supplied. (B) Pay money damages to the developer. (C) Transfer water from the owner's well to the developer's property. (D) Nothing.

D, Nature of riparian Right, reasonable use theory

6. A homeowner and a local builder entered into a written contract that called for the builder to build a second story onto the top of the homeowner's one-story residence. When scheduling conflicts arose, the builder asked the homeowner if they could substitute his buddy, an out-of-town builder who had comparable experience and skills, to perform the local builder's part of the contract. All of the parties agreed to the substitution. Unfortunately, the out-of- town builder made a major blunder that will be quite expensive to correct. Is the local builder liable to the homeowner for the cost of correcting the defect? (A) Yes, because the substitution in and of itself does not relieve the local builder of liability on the underlying contract. (B) Yes, because the local builder did not give any consideration on which to base a release. (C) No, because the local builder transferred his duties to the out-of-town builder. (D) No, because the local builder was discharged through a novation.

D- Novation


संबंधित स्टडी सेट्स

Abdomen - Appleton and Lange Questions, Abdomen - BRS Gross Anatomy 7th Ed, Abdomen-Big Picture Review Questions, Abdomen-Lippincott Review Question (Study w/Definition First)

View Set

Chapter 13, Chapter 14, Chapter 15

View Set

Guerrero - TExES 150 School Librarian Test Terms

View Set

chapter 3: neuroscience and behavior

View Set

Ch14 Commercial Vehicle Batteries

View Set

ECONMT - NEC 2014 Study Card Quiz

View Set

NSG 330 Ch 45- Management w/ Oral & Esophageal Disorders

View Set